複素数の底力                               戻る

 2022年度より実施の新学習指導要領で、数学 I・II・III・A・B・Cの6分野構成に復活し、
「ベクトル」・「複素数平面」・「式と曲線」が新設の数学Cにまとめられた。
(数学Cは、2012年度に旧学習指導要領から廃止されていた。)

 一見して難しそうな平面幾何の問題も、複素数を利用することにより簡明に示される場合
がある。このページでは、そのような体験をまとめていきたいと思う。

 特に、回転や拡大・縮小に関係する場面において、複素数は絶大な力を発揮する。

 以下に、目次をまとめたので、興味あるところから読み進めていただき、「複素数の底力」
を実感していただけたら、幸甚に思います。


≪ 目 次 ≫

ガウス平面   2点を通る直線(→一般   線分の中点
三角形の重心   共線条件   メネラウスの定理
回転と相似   基本図形の表現   定点通過問題
軌跡   直角三角形の条件   正三角形の条件
長方形の条件   トレミーの不等式(→複比   平行と垂直
ブラマグプタの定理   プトレマイオスの定理   相似条件
ナポレオンの定理   円周角の定理   垂線の足(→一般
シムソンの定理   モーリーの定理   パスカルの定理
単位円の割線   一般の垂線の足   一般の2点を通る直線
法線に垂直な直線   線分の垂直2等分線   ヘッセの標準形
2直線の交角   2直線の交点   三角形の垂心
2直線の交点を通る直線   3直線が1点で交わる   三角形の外心
4点の共円問題   複比   トレミーの不等式(複比)
九点円   フランケ点   三角形の内心
三角形の傍心   フォイエルバッハの定理   擬似重心
重心座標   面積   ド・モアブルの定理 
変換   相似変換    反転 
1次分数変換 アフィン変換



ガウス平面

 方程式の解に負の数の平方根(虚数)が現れることが意識され始めたのは、16世紀頃と
言われるが、当時は、ありえない数と考えられていたらしい。18世紀になって、ノルウェーの
ヴェッセル(1745〜1818)とスイスのアルガン(1768〜1822)は、それぞれ独立に、1797年
と1806年に複素数平面の考えを発表した。その後、1811年にガウス(1777〜1855)は、
複素数は、実数が数直線上の点で表されるように、座標平面上の点で表されると書き記し
ている。これが、ガウス平面と呼ばれる由縁である。

 複素数 z は、虚数単位 i (i2=−1)を用いて、 z=a+bi (a、bは実数) と表される。

 a は複素数 z の実部と言われ、 a=Re(z) と表す。
 b は複素数 z の虚部と言われ、 b=Im(z) と表す。

 複素数 z=a+bi に、(a,b)という座標を持つ点を対応させることにより、ガウス平面(複
素数平面とも言われる。)が定められる。横軸は、実軸、縦軸は虚軸と言われる。

     

 複素数 z の絶対値は、 |z|= で定義される。|z|は、原点Oと点 z の距
離のことである。

 複素数 z の偏角θは、 arg z で表される。2πの整数倍を無視すれば、ただ1通りに定
まる。


問題 z1=1+i、z2=1+i のとき、複素数 z1/ z2 の絶対値と偏角を求めよ。

(解) |z1|=2、|z2|= なので、 |z1/ z2|=2/

 arg(z1)=π/3、arg(z2)=π/4 なので、

  arg(z1/ z2)=π/3−π/4=π/12  (終)


問題 次の複素数 z の絶対値と偏角を求めよ。ただし、−π<θ<π とする。

 z=(1+cosθ−i・sinθ)/(1+cosθ+i・sinθ)

(解) 倍角の公式より、

z=(2cos2(θ/2)−i・2sin(θ/2)cos(θ/2))/(2cos2(θ/2)−i・2sin(θ/2)cos(θ/2))

 =(cos(θ/2)−i・sin(θ/2))/(cos(θ/2)+i・sin(θ/2))

 =(cos(θ/2)−i・sin(θ/2))2

 =cosθ−i・sinθ

 =cos(−θ)+i・sin(−θ)

 よって、 |z|=1 、arg z =−θ  (終)


問題 x の2次方程式 ax2+bx+c=0 の解 α について、a>b>c>0 ならば、

  |α|<1 であることを示せ。

(解) α が実数解のとき、すなわち、b2−4ac≧0 のとき、

 α=(−b±√(b2−4ac))/(2a) より、

 |α|≦b/(2a)+b/(2a)=b/a<1

α が虚数解のとき、すなわち、b2−4ac<0 のとき、

 α=(−b±i・√(4ac−b2))/(2a) より、

 |α|2=b2/(4a2)+(4ac−b2)/(4a2)=c/a<1 より、 |α|<1  (終)


 この問題は、掛谷の定理の特別な場合で、「方程式の解のある性質」において、その別解
が与えられている。


 ガウス平面においては、2つの線分のなす角が、複素数を用いて簡単に表される。しかも、
2つの線分は交わっていなくともよい点が面白い。

   左図において、2つの線分のなす角(正の向き)θ は、

         
  
  で与えられる。


 偏角について、読者のために練習問題を置いておこう。

練習問題 平行四辺形ABCD内に1点Pを、∠PAB=∠PCB となるようにとる。

     

このとき、∠PBC=∠PDC が成り立つことを示せ。

(解) 平行四辺形ABCDの対角線の交点を原点Oにとり、A(α)、B(β)、P(p)とおくと、

 C(−α)、D(−β) である。

    

 題意より、∠PAB=∠PCB なので、

  arg{(p−α)/(β−α)}=arg{(β+α)/(p+α)}

よって、

 arg[{(p−α)/(β−α)}・{(p+α)/(β+α)}]=arg{(p2−α2)/(β2−α2)}=0

から、 (p2−α2)/(β2−α2)=λ (λは正の実数) とおける。

 このとき、

(p2−α2)/(β2−α2)=(p2−β2+β2−α2)/(β2−α2)=−(p2−β2)/(α2−β2)+1

より、 (p2−β2)/(α2−β2)=1−λ は実数となる。

よって、 arg{(p2−β2)/(α2−β2)}=0 から、

 arg{(p−β)/(−α−β)}=arg{(−α+β)/(p+β)}

が成り立つので、 ∠PBC=∠PDC と言える。  (終)


 複素数 z の共役複素数は、=a−bi で定義される。点 z と点 は、実軸に関して線
対称で、

 |z|2=z

  z が実数 ⇔ z=

  z が純虚数 ⇔ z+=0 かつ z≠0

が成り立つ。


問題  次の問いに答えよ。

(1) α、βがともに0でない複素数で、βが実数のとき、βはαの実数倍と書けることを
   示せ。

(2) α、β、γ、δが互いに異なる0でない複素数で、γ、δ、δ+γがすべて実
   数であるとき、α−βの偏角とγ−δの偏角との間には、どんな関係が成り立つか。
    ただし、偏角θの範囲は、0≦θ<2πとする。

(解)(1) 題意より、 β=α で、すなわち、 β/α=/ から、

    β/α=k (kは実数) とおける。したがって、βはαの実数倍と書ける。

(2) (1)より、 γ=mα 、δ=nβ (m、nは実数) と書ける。

 また、、δ+γが実数なので、 δ+γ=α+β

 γ=mα 、δ=nβ を代入して、 nβ+mα=nα+mβ より、

 (n−m)β=(n−m)α となり、 n=m または、 βが実数

 n=m のとき、 γ=mα 、δ=mβ から、 γ−δ=m(α−β)

  よって、(γ−δ)/(α−β)の偏角は、0 または π

 βが実数のとき、 β=kα (kは実数) より、 γ=mα 、δ=nkα なので、

 γ−δ=(m−nk)α 、 α−β=(1−k)α

 よって、 (γ−δ)/(α−β)が実数より、(γ−δ)/(α−β)の偏角は、0 または π

以上から、何れにしても、(γ−δ)/(α−β)の偏角は、0 または π  (終)


(別解)(2) ()(γ−δ)=γ−(δ+γ)+δ において、

 条件より、γ、δ、δ+γがすべて実数なので、()(γ−δ)は実数となる。

 よって、γ−δはα−βの実数倍となり、(γ−δ)/(α−β)の偏角は、0 または π

となる。  (終)


(コメント) 最初から、別解のように計算すべきでした...。


 和歌山県立医科大学(2016)で、次のような問題が出題されている。

問題  異なる複素数α、βに対して、(z−α)/(z−β)が純虚数となる z は、どんな図形
    を描くか。

(解) arg(z−α)/(z−β)=arg(α−z)/(β−z)=∠βzα=±π/2 より、

   求める図形は、線分αβを直径とする円(ただし、α、βは除く)を描く。  (終)


問題  a、b、c は実数の定数とし、F(z)=az2+bz+c とする。このとき、任意の複素数
    zに対して、F(z)+F()は実数であることを示せ。

(解) F(z)+F(
   =a(z22)+b(z+)+2c
   =a((z+2−2z)+b(z+)+2c

 z+、z は実数なので、 F(z)+F() は実数である。  (終)


(別解) F()= より、F(z)+F() が実数は明らか。  (終)


問題  四角形ABCDにおいて、 AB2+CD2=AD2+BC2 が成り立つとき、2つの対角
    線は直交することを示せ。

(解) ガウス平面において、 A(0)、B(β)、C(γ)、D(δ)とおく。条件より、

 |β|2+|δ−γ|2=|δ|2+|γ−β|2

すなわち、 δγ=γβ なので、 γ()=−(β−δ)

 よって、 γ/(β−δ)=−/() が成り立つので、 γ/(β−δ) は純虚数

 したがって、 対角線ACと対角線BDは直交する。  (終)


問題  ガウス平面において、原点中心の半径1の円周上に異なる3点α、β、γがある。
    このとき、z=(β−γ)/(α−β)とおくと、(α/γ)z の共役複素数は、z であること
    を示せ。

(解) 3点α、β、γは、原点中心の半径1の円周上にあるので、

 =1/α 、=1/β 、=1/γ

 このとき、 w=(α/γ)z とおくと、 =(/=(γ/α)

ここで、=()/()=(α/γ)(β−γ)/(α−β)=(α/γ)z なので、

 =(γ/α)(α/γ)z=z となる。

 以上から、(α/γ)z の共役複素数は、z である。  (終)


 絶対値に関する問題を整理しておこう。(令和5年8月10日付け)

問題  |α|=|β|=|γ|=1 のとき、

 |α+β+γ|=|1/α+1/β+1/γ|=|αβ+βγ+γα|

が成り立つことを示せ。

(解) |α|=|β|=|γ|=1 より、 α=β=γ=1 なので、

 1/α+1/β+1/γ==(α+β+γ)の共役複素数

よって、 |α+β+γ|=|1/α+1/β+1/γ| が成り立つ。

 また、1/α+1/β+1/γ=(αβ+βγ+γα)/(αβγ) なので、

 |1/α+1/β+1/γ|=|αβ+βγ+γα| が成り立つ。


問題  4つの複素数 α、β、γ、δ に対して、

 (|α|2+|β|2)(|γ|2+|δ|2)=|α+β2+|αδ-βγ|2

が成り立つことを示せ。

(解) |α+β2=(α+β)(γ+δ)

  =|α|2|γ|2+αδ+βγ+|β|2|δ|2

|αδ−βγ|2=(αδ−βγ)(

=|α|2|δ|2−αδ−βγ+|β|2|γ|2

より、 |α+β2+|αδ-βγ|2

 =|α|2|γ|2+|α|2|δ|2+|β|2|γ|2+|β|2|δ|2

 =(|α|2+|β|2)(|γ|2+|δ|2

 以上から、等式は成り立つ。  (終)


問題  次の問いに答えよ。

(1) |α|=1 のとき、 |(α−β)/(1−β)|=1 を示せ。

(2) |α|>1 のとき、 |(α−β)/(1−β)|と 1 の大小を調べよ。

(解)(1) |(α−β)/(1−β)|=|(α−β)/(α−β)|・1/||=1

(2) |α−β|2−|1−β|2

 =|α|2−αβ+|β|2−(1−αβ+|α|2|β|2

 =|α|2+|β|2−1−|α|2|β|2

 =(|α|2−1)(1−|β|2

よって、 |β|>1 のとき、

  |α−β|<|1−β| から、 |(α−β)/(1−β)|<1

|β|=1 のとき、

  |α−β|=|1−β| から、 |(α−β)/(1−β)|=1

|β|<1 のとき、

  |α−β|>|1−β| から、 |(α−β)/(1−β)|>1  (終)


問題  α、β、γ を複素数とし、A=α+β+γ、B=α+βω+γω2
    C=α+βω2+γω とおく。ただし、ωは、1の3乗根である。このとき、

  |A|2+|B|2+|C|2=3(|α|2+|β|2+|γ|2

が成り立つことを示せ。

(解) =−ω−1=ω2、ω3=1、ω=1 に注意して、

|A|2=|α|2+α+αβ+|β|2+βγ+γ+|γ|2

|B|2=(α+βω+γω2)(ω2ω)

 =|α|2+αω2+αω+βω+|β|2+βω2γω2γω+|γ|2

|C|2=(α+βω2+γω)(ω+ω2

 =|α|2+αω+αω2βω2+|β|2+βω+γω+γω2+|γ|2

より、

|A|2+|B|2+|C|2

=3(|α|2+|β|2+|γ|2

 +(αβ+βγ+γα)(ω2+ω+1)

=3(|α|2+|β|2+|γ|2)  (終)


 α、β、γ を複素数とし、A=α+β+γ、B=α+βω+γω2、C=α+βω2+γω
に対して、次の因数分解が成り立つことは有名である。この因数分解が前問に使えないかと
思案したが、...挫折!

問題  α3+β3+γ3−3αβγ=ABC が成り立つことを示せ。

(解) ABC

 =(α+β+γ)(α+βω+γω2)(α+βω2+γω)

ここで、ω+ω2=−1、ω3=1 に注意して、

(α+βω+γω2)(α+βω2+γω)

 =α2+(β(ω+ω2)+γ(ω+ω2))α+β2+βγ(ω+ω2)+γ2

 =α2+β2+γ2−αβ−βγ−γα

なので、

ABC=(α+β+γ)(α2+β2+γ2−αβ−βγ−γα)=α3+β3+γ3−3αβγ

が成り立つ。  (終)


 前問と同様に、次の事実も興味深い。

問題  次の等式が成り立つことを示せ。ただし、ωは、1の3乗根である。

(1) α3−β3=(α−β)(α−βω)(α−βω2

(2) α3+β3=(α+β)(α+βω)(α+βω2

(解) ω+ω2=−1、ω3=1 に注意して、

(1) (α−βω)(α−βω2)=α2−(ω+ω2)αβ+β2ω3=α2+αβ+β2

 より、

 (α−β)(α−βω)(α−βω2)=(α−β)(α2+αβ+β2)=α3−β3

(2) (α+βω)(α+βω2)=α2+(ω+ω2)αβ+β2ω3=α2−αβ+β2

 より、

 (α+β)(α+βω)(α+βω2)=(α+β)(α2−αβ+β2)=α3+β3  (終)


問題  α≠0 のとき、次の不等式が成り立つことを示せ。また、等号成立も調べよ。

  |α+1/|≧2

(解) |α+1/2−4=|α|2+1/|α|2−2≧2−2=0 より、

 |α+1/|≧2

 等号成立は、|α|=1/|α| すなわち、 |α|=1 のときに限る。  (終)


問題  ωを1とは異なる1の3乗根とする。
    3つの複素数 x、y、z との間に、x+yω+zω2=0 が成り立つとき、△xyz の形状
  を答えよ。

(解) 題意より、ω≠1 、ω3=1 で、ω2+ω+1=0 である。

 x+yω+zω2=0 から、 x+yω−z(1+ω)=0 より、 x−z=−(y−z)ω

 |ω|=1 、argω=π/3 なので、 |x−z|=|y−z| 、arg∠yzx=−π/3

よって、△xyz は、頂角の大きさがπ/3の2等辺三角形なので、正三角形となる。  (終)


(参考) △xyz が正三角形となるための必要十分条件は、x+yω+zω2=0 であることが
    知られている。(→ 「正三角形の条件」)

 △αβγ が正三角形となるための必要十分条件としては、

 α2+β2+γ2−αβ−βγ−γα=0

が知られているが、

 α2+β2+γ2−αβ−βγ−γα=(α+βω+γω2)(α+βω2+γω)

と因数分解されることと関係があり、興味深い。


問題  ガウス平面上に、原点Oを内部に持つ凸n角形がある。その頂点を表す複素数を
   反時計回りに z1、z2、・・・、z とおく。任意の自然数 k (1≦k≦n−1)に対して、
  |zk+1−zk|>|zk| 、|zk+1−zk|>|zk+1
 が成り立つとき、n<6 であることを示せ。

(解) △Ozkk+1 において、条件より、∠zkOzk+1>∠Ozk+1k、∠zkOzk+1>∠zk+1k

よって、 2∠zkOzk+1>∠Ozk+1k+∠zk+1kO から、

 3∠zkOzk+1>∠Ozk+1k+∠zk+1kO+∠zkOzk+1=180° なので、 

 ∠zkOzk+1>60° が成り立つ。

 したがって、

 Σk=1n-1 ∠zkOzk+1>60°×n で、 Σk=1n-1 ∠zkOzk+1=360° から、

 n<6 であることが分かる。  (終)


問題  ガウス平面上の3点α(≠0)、β、γについて、

 βとγが0とαを結ぶ直線に関して対称な点であるための必要十分条件は、

 γ=α であることを示せ。

 

(解) argα=θとして、β、γを−θ回転させる。このとき、β/αとγ/αは実軸に関して対称であるので、

 γ/α=/ が成り立つ。よって、γ=α である。  (終)


(追記) 令和5年12月7日付け

問題  ガウス平面上に2点A(α)、B(β)が下図のように与えられたとき、商 α/β を表す
   点Cを図示せよ。ただし、E(1)とする。

  

(解) △OAB∽△OCE となるように点Cをとればよい。

   (終)


(追記) 令和5年12月8日付け

問題  a、bは複素数とする。次の問いに答えよ。

(1) 任意の複素数 z に対して、常に z+az+b+1 が実数になるための必要十分条件
  を求めよ。

(2) 任意の複素数 z に対して、常に z+az+b+1≧0 が成り立つような|a|の値の範
  囲を求めよ。

(解)(1) 題意より、 z+az+b+1=zz+1 が成り立つから、

 (a−)z=(−b) すなわち、任意の複素数 z に対して、常に (a−)z は実数

そのための必要十分条件は、 a= である。

(2) まず、z+az+b+1が実数なので、(1)より、 a= すなわち、b=

このとき、z+az++1≧0 から、 (z+)(+a)≧|a|2−1

すなわち、 |z+2≧|a|2−1 が任意の複素数 z に対して、常に成り立つためには、

 |a|2−1≦0 すなわち、 −1≦|a|≦1 である。  (終)


 上記の問題(1)を簡略化して次の問題を得る。

問題  a、bは複素数とする。任意の複素数 z に対して、常に az+b が実数になるため
   の必要十分条件を求めよ。

(解) 題意より、 az+bz が成り立つから、(a−)z=(−b) すなわち、

 任意の複素数 z に対して、常に (a−)z は実数となり、そのための必要十分条件は、

 a= である。  (終)


問題  a、bは実数とする。2次方程式 x2+2ax+b=0 の2つの解を α、βとする。この
   とき、α、βを直径の両端とする円Cが原点Oを内部に含むとき、(a,b)の存在範囲を
   図示せよ。

  

(解) 解と係数の関係より、α+β=−2a 、αβ=b で、判別式をDとすると、

 D/4=a2−b である。

原点Oが円Cの内部に含まれるための必要十分条件は、 |α+β|<|α−β|

 すなわち、 2|a|<|α−β|

ここで、(α−β)2=(α+β)2−4αβ=4a2−4b=4(a2−b)

よって、2つの解α、βがともに実数 すなわち、a2−b≧0 のとき、

 |α−β|=2√(a2−b) と書けるので、 |a|<√(a2−b)

両辺を平方して、 a2<a2−b より、 b<0 ただし、b≦a2

 2つの解α、βがともに虚数 すなわち、a2−b<0 のとき、

 |α−β|=2√(b−a2) と書けるので、 |a|<√(b−a2

両辺を平方して、 a2<b−a2 より、 b>2a2 ただし、b>a2

 以上から、(a,b)の存在範囲を図示すれば、下図を得る。

  

 ただし、境界線は含まない。


(追記) 令和5年12月10日付け

問題  正方形ABCDの中心がP(1+2i)で、点A(a+bi)とする。このとき、次の問いに答えよ。

(1) 点B、C、Dを表す複素数を求めよ。

(2) 両端を除く線分AB、CDがともに虚軸と交わるような点Aの存在範囲を求めよ。

  

(解)(1) 線分ACの中点がPなので、明らかに、C(2−a+(4−b)i) である。

また、((a−1+(b−2)i)×i+1+2i=3−b+(a+1)i から、B(3−b+(a+1)i)

線分BDの中点がPなので、D(b−1+(3−a)i) である。

(2) 両端を除く線分AB、CDがともに虚軸と交わるためには、

 a(3−b)<0 かつ (2−a)(b−1)<0

であればよい。このとき、点(a,b)の存在範囲を図示すれば、下図を得る。

  

 ただし、境界線は含まない。  (終)



2点を通る直線

 ガウス平面において、2点 z1、z2 を通る直線上の点 z は、

  z=mz1+nz2 (m+n=1)

と表される。

 mn>0のとき、点 z は内分点で、mn<0のとき、外分点となる。

線分の中点

 2点 z1、z2 を結ぶ線分z12の中点は、 (z1+z2)/2 で表される。


三角形の重心

 △z123 の重心は、 (z1+z2+z3)/3 で表される。


 実際に、△ABCにおいて、A(α)、B(β)、C()とし、線分ABの中点P()、線分BC

の中点Q()、線分CAの中点R()をとる。

   

 =(αβ)/2 で、AQとCPの交点をG()とすると、

 =k(αβ)/2=(k/2)α+k(β/2)

 Gは、A(α)とQ(β/2)を結ぶ直線上にあるので、 k/2+k=1 より、 k=2/3

  よって、 =(αβ)/3

 同様にして、BRとCPの交点をG’(g’)とすると、 g’=(αβ)/3 となり、G=G’

 よって、△ABCの3中線は1点Gで交わり、Gは重心となる。

 A(1)、B(2)、C(3)のとき、平行移動して、A’(α)、B’(β)、C’()とすると、

  α13 、β23

△A’B’C’の重心は、 (αβ)/3=(12−23)/3 となるので、平行移動して、

 △z123 の重心は、 (12−23)/3+3(z1+z2+z3)/3 で表される。


問題  ガウス平面上の3点A(2+2i)、B(−4+2i)、C(−1−i)について、次の問いに
    答えよ。

(1) 線分ABの中点Mを表す複素数を求めよ。
(2) 線分BCを 1 : 2 に内分する点Pを表す複素数を求めよ。
(3) 線分CAを 1 : 2 に外分する点Qを表す複素数を求めよ。
(4) △ABCの重心Gを表す複素数を求めよ。

(解)(1) {(2+2i)+(−4+2i)}/2=−1+2i

(2) {2(−4+2i)+(−1−i)}/3=−3+i

(3) {2(−1−i)−(2+2i)}/(−1+2)=−4−4i

(4) {(2+2i)+(−4+2i)+(−1−i)}/3=−1+i  (終)


問題  △ABCの辺BC、CA、ABをそれぞれ m:n (ただし、m+n=1)に内分する点を
    P、Q、Rとするとき、△ABCと△PQRの重心は一致することを示せ。

   

(解) ガウス平面において、 A(α)、B(β)、C(γ)とおくと、

 P(nβ+mγ) 、Q(nγ+mα) 、R(nα+mβ)

となる。このとき、

△ABCの重心は、 (α+β+γ)/3

△PQRの重心は、

 {(nβ+mγ)+(nγ+mα)+(nα+mβ)}/3=(α+β+γ)/3

 よって、△ABCと△PQRの重心は一致する。  (終)


問題  △A111、△A222、△A333 は、同じ向きの正三角形とする。
    △A123、△B123、△C123 の重心をそれぞれP、Q、Rとするとき、
    △PQRは正三角形であることを示せ。

    

(解) ガウス平面において、A(α)、B(β)、C(γ) (k=1、2、3) とする。

△A111 は、同じ向きの正三角形なので、 α1+β1ω+γ1ω2=0 ・・・ (1)

△A222 は、同じ向きの正三角形 α2+β2ω+γ2ω2=0 ・・・ (2)

△A333 は、同じ向きの正三角形 α3+β3ω+γ3ω2=0 ・・・ (3)

 このとき、P(p)、Q(q)、R(r) とすると、

 p=(α1+α2+α3)/3 、q=(β1+β2+β3)/3 、r=(γ1+γ2+γ3)/3

なので、(1)(2)(3)より、 p+qω+rω2=0 が成り立つ。

 よって、△PQRは正三角形である。  (終)


共線条件

 相異なる3点 α、β、γ が同一直線上にあるための必要十分条件は、

   (γ −α)/(β−α) が実数



(コメント) ∠βαγ=0 または π より、3点 α、β、γ は同一直線上にある。

上記の条件を数式化すれば、

  (β−γ)+(γ−α)+(α−β)=0

となる。

#△αβγの外心を与える式の分母なんですね...。半径が∞で、円が直線になるという
 感覚でいいのかな?何となく意味深。

(証明) 2点α、βを通る直線の方程式は、

  (z−α)/(β−α)=()/(

すなわち、 ()(z−α)−(β−α)()=0

点γを通るので、 ()(γ−α)−(β−α)()=0

 (γ−α)−(γ−α)−(β−α)+(β−α)=0

 (β−γ)+(γ−α)+(α−β)=0  (証終)


(追記) 令和5年8月13日付け

 岡山大学前期理系(2019)で、次のような問題が出題されている。

問題  次の3つの等式 zw 、 |z−1|=1 、|z−w|=2 を満たす複素数
    z、w について、以下の問いに答えよ。ただし、z≠0 とし、z の偏角を θ と表す。

(1) 複素数平面において、3点 0、z、w は一直線上にあることを示せ。
(2) z と w を θ を用いて表せ。
(3) θは、0≦θ<π/2 の範囲を動くとする。このとき、w のとりうる値について、その虚
  部の最大の値を求めよ。

(解)(1) zw から、 w/z=/ なので、 w/z は実数

 よって、 w/z=k (kは実数) とおけるので、 w=kz

 したがって、3点 0、z、w は一直線上にある。

(2) r を実数として、 z=r(cosθ+i・sinθ) とおく。

 |z−1|=1 より、 r=2cosθ となるので、 z=2cosθ(cosθ+i・sinθ)

 さらに、arg(z−w)=θ または θ+π なので、|z−w|=2 より、

 z−w=±2(cosθ+i・sinθ) と書ける。

よって、 w=z±2(cosθ+i・sinθ)=2cosθ(cosθ+i・sinθ)±2(cosθ+i・sinθ)

すなわち、 w=2cosθ(cosθ±1)+i・2sinθ(cosθ±1)

(3) (2)より、 (wの虚部)=2sinθ(cosθ±1) で、最大の値を求めるので、

 (wの虚部)=2sinθ(cosθ+1) (0≦θ<π/2 ) の場合を調べればよい。

 y=sinθ(cosθ+1) とおくと、

 y’=cosθ(cosθ+1)−sin2θ=2cos2θ+cosθ−1=(2cosθ−1)(cosθ+1)

0≦θ<π/2 で、y’=0 を解くと、 θ=π/3

 π/3の前後で、y’の符号が+から−に変わるので、yは、π/3で極大かつ最大となる。

よって、 (wの虚部)の最大値は、 2sin(π/3)(cos(π/3)+1)=3/2  (終)



 直線の方程式の応用として、次のメネラウスの定理も有名だろう。

メネラウスの定理  △ABCにおいて、A(α)、B(β)、C()とする。

 直線AB上の点P()、直線BC上の点Q()、直線CA上の点R()をとるとき、

  3点P、Q、Rが一直線上にあるための必要十分条件は、

    (AP/PB)(BQ/QC)(CR/RA)=-1

(注) APBP、・・・ 等は有向線分のため、「=-1」となる。

   

(証明)(必要であること)

 (AP/PB)=s 、(BQ/QC)=t 、(CR/RA)=u とおくと、

 Pは、ABを s : 1 に分ける
 Qは、BCを t : 1 に分ける
 Rは、CAを u : 1 に分ける

ので、 =(α+sβ)/(s+1) 、=(1/(t+1))β 、=(u/(u+1))α

 このとき、 α=((u+1)/u) 、β=(t+1) を第一式に代入して、

   =((u+1)/(u(s+1)))+(s(t+1)/(s+1))

 3点P、Q、Rが一直線上にあるので、 (u+1)/(u(s+1))+s(t+1)/(s+1)=1

すなわち、 u+1+us(t+1)=u(s+1) より、 stu=-1 となる。

 よって、 (AP/PB)(BQ/QC)(CR/RA)=-1 が言える。

(十分であること) 直線AB上の点P()、直線BC上の点Q()、直線CA上の点R()を
          とるとき、

  (AP/PB)(BQ/QC)(CR/RA)=-1

が成り立つとする。2点Q、Rを通る直線が、直線ABと交わる点をP’(p’)とする。

AP’/P’B)=s’ とおくと、(必要であること)の証明から、 s’tu=-1 となる。

 よって、 s=s’ となり、P=P’ から、3点P、Q、Rが一直線上にある。  (証終)


回転と相似

 ガウス平面において、複素数の積が回転と相似拡大を表すので、煩雑な回転等の問題が
楽になる。

 i を掛ける ・・・ 原点中心、90°の回転を表す。

例 1×i=i 、i×i=−1 、−1×i=−i 、−i×i=1

 このことから、

 1に i を掛ける ・・・ 1を原点中心、90°の回転で、i に移る。
 i に i を掛ける ・・・ i を原点中心、90°の回転で、−1 に移る。
−1に i を掛ける ・・・ −1 を原点中心、90°の回転で、−i に移る。
−i に i を掛ける ・・・ −i を原点中心、90°の回転で、1に移る。

 したがって、以上から、 (−1)×(−1)=1 が説明される。


 α=(1+i)/2を掛ける ・・・ 原点中心、60°の回転を表す。


 ω=(−1+i)/2を掛ける ・・・ 原点中心、120°の回転を表す。


問題  △ABCの外部にある点Pを、Aを中心に−2∠Aだけ回転した点をQとする。さらに、
    QをBを中心に−2∠Bだけ回転した点をR、RをCを中心に−2∠Cだけ回転した点
    をSとする。このとき、S=P であることを示せ。

       

(解) △ABCの外接円を単位円とし、A(e^(θi))、B(e^(φi))、C(1)として示しても一般
   性は失われない。ただし、0<θ<φ<2π とする。

 このとき、 ∠A=π−φ/2 、∠B=θ/2 、∠C=φ/2−θ/2 である。

 P(p) 、Q(q) 、R(r) S(s) とおくと、題意より、

 q−e^(θi)=(p−e^(θi))e^((φ−2π)i)=(p−e^(θi))e^(φi)

なので、 q=pe^(φi)−e^((θ+φ)i)+e^(θi)

 同様にして、

 r−e^(φi)=(q−e^(φi))e^((−θi) より、

 r=qe^((−θi)−e^((φ−θ)i)+e^(φi)

 上式に q=pe^(φi)−e^((θ+φ)i)+e^(θi) を代入して、

 r=pe^((φ−θ)i)+1−e^((φ−θ)i) となる。

 同様にして、 z−1=(r−1)e^((θ−φ)i) より、

 z=re^((θ−φ)i)−e^((θ−φ)i)+1

 上式に、 r=pe^((φ−θ)i)+1−e^((φ−θ)i) を代入して、

 z=p+e^((θ−φ)i)−1−e^((θ−φ)i)+1=p

 したがって、 S=P である。  (終)


(コメント) 3つの回転を合成すると、元に戻るという美しい結果ですね!


 ガウス平面において、3点 1、ω、ω2 を考える。3点で、正三角形1ωω2が作られる。

     

 ω−1 の絶対値は、 で、偏角は、5π/6 である。

 ω2−ω の絶対値は、 で、偏角は、3π/2 である。

 1−ω2 の絶対値は、 で、偏角は、π/6 である。


 したがって、複素数 (ω−1)(ω2−ω)(1−ω2) の絶対値は、3 で、偏角は、

  5π/6+3π/2+π/6=2π+π/2

となり、 (ω−1)(ω2−ω)(1−ω2)=3i は純虚数となる。


 z=(ω−1)(ω2−ω)(1−ω2) が純虚数であることは、次のようにしても分かる。

すなわち、 =(ω2−1)(ω−ω2)(1−ω)=−(ω−1)(ω2−ω)(1−ω2)=−z

より、z+=0 となって、z は純虚数である。


 上記と同様のことが、単位円周上の任意の3点 α、β、γ (反時計回り) についても

  複素数 z=(α−β)(β−γ)(γ−α)/(αβγ)

は、純虚数であることが示される。

     

 実際に、

 z=(α−β)(β−γ)(γ−α)/(αβγ)

  =(α−β)(β−γ)(γ−α)=(1−β)(1−γ)(1−α)

 よって、 =(1−α)(1−β)(1−γ)=(β−α)(γ−β)(α−γ)

 すなわち、 =−(α−β)(β−γ)(γ−α)=−z より、 z+=0 となって、

z は純虚数である。


 同様にして、単位円周上の任意の4点 α、β、γ、δ (反時計回り) について

  複素数 z=(α−β)(β−γ)(γ−δ)(δ−α)/(αβγδ)

は、実数であることが示される。

     

 実際に、

 z=(α−β)(β−γ)(γ−δ)(δ−α)/(αβγδ)

  =(α−β)(β−γ)(γ−δ)(δ−α)

  =(1−β)(1−γ)(1−δ)(1−α)

 よって、

 =(1−α)(1−β)(1−γ)(1−δ

  =(β−α)(γ−β)(δ−γ)(α−δ)

 すなわち、 (α−β)(β−γ)(γ−δ)(δ−α)=z より、

z は実数である。


(追記) 令和5年2月21日付け

 よおすけさんからの問題提供です。今回は、東京工業大学理系後期(1991)の問題です。

問題  原点Oを中心とする半径2の円Kの内部に、一辺の長さが2で対角線の交点がOと
    なるような正方形ABCDをとる。K上の点Pにおいて、線分POと角θで交わる2本の半
    直線を引く。

  

 このとき、PがK上どのような位置にあっても、これら2本の半直線が正方形ABCDを通るよ
うなθの最大値を求めよ。

(解) ガウス平面において、A()、B(i)、C(−2)、D(−i)としても一般性を失
   わない。また、対称性から、P(2e^(iφ)) (ただし、0≦φ≦π/4) としてよい。

  

 直線ADと円との交点をP”とおく。原点Oと直線ADの距離は1なので、∠OP”A=π/6と
なる。

 このことから、題意より、θ≦π/6 であることが分かる。

 よって、最大値がθ=π/6を示すには、直線nが直線ADと一致するとき、直線mが辺AB、
BCと交わることを示せば十分である。

 原点Oから直線ADに下ろした垂線の足をHとすると、H((1−i)/)である。

 よって、点P”を表す複素数は、2e^(i(π/3))(1−i)/=(+1+(−1)i)/

点P”を中心に、原点Oを−π/6だけ回転した点をzとおくと、

 z=e^(i(−π/6))(−−1+(−+1)i)/+(+1+(−1)i)/

  =()i

よって、この点 z は、線分OB上にあり、直線mは辺AB、BCと交わることを示す。

 以上から、求める最大値は、θ=π/6 である。  (終)


(コメント) ちょっとスッキリしない証明になっていたので、修正しました。納得いくものになっ
      た筈です。「Pがどこにあっても」という条件を上手く使いこなすのは難しいですね!
      よおすけさんの補足に期待します。


 よおすけさんからのコメントです。(令和5年2月25日付け)

 1991年当時は、複素数平面が高校範囲外の頃なので、通常なら、ベクトル、実数範囲の
微分・積分で解くところを無理言って複素数で解いてもらって、感謝しています。

 上記解答の補足ですが、僕からは言うことはありません。


 上記では、微分を使わない解法だったので、今度は微分を活用する別解に挑戦しました。
考察する図は、次の図の方が分かり易かったですね。

(別解) ガウス平面において、A(−1+i)、B(−1−i)、C(1−i)、D(1+i)としても一般性
    を失わない。また、対称性から、P(2e^(iφ)) (ただし、0≦φ≦π/4) としてよい。

  

 線分ADをD方向に延長し、円Kとの交点をP’とすると、φ=π/6 である。

 ここで、θ=π/6 のとき、直線mは直線ADと一致し、直線nは線分BC、CDと交わるの

で、条件を満たす。よって、求めるθは、π/6以下であることが分かる。

(イ) π/6≦φ≦π/4 のとき、直線mが点Aを通る場合を考える。

 P(2e^(iφ))から、

 PA=−1+i−2e^(iφ)=−1−2cosφ+i・(1−2sinφ)

 PO=−2cosφ−2i・sinφ

よって、 cosθ=(cosφ−sinφ+2)/√(6+4(cosφ−sinφ))

 ここで、cosφ−sinφ=x とおくと、 x=cos(φ+π/4) で、

π/6≦φ≦π/4 より、 5π/12≦φ+π/4≦π/2 なので、

 0≦x≦(−1)/2 である。

このとき、 y=cosθ=(x+2)/√(6+4x) の増減を調べる。

 y’=(2x+2)/√(6+4x)3 より、0≦x≦(−1)/2 において、単調増加である。

 よって、 /3≦y≦/2 より、 θ≧π/6 が成り立つ。

(ロ) 0≦φ≦π/6 のとき、直線mが点Dを通る場合を考える。

 P(2e^(iφ))から、

 PD=1+i−2e^(iφ)=1−2cosφ+i・(1−2sinφ)

 PO=−2cosφ−2i・sinφ

よって、 cosθ=(2−(sinφ+cosφ))/√(6−4(sinφ+cosφ))

 ここで、sinφ+cosφ=x とおくと、 x=sin(φ+π/4) で、

0≦φ≦π/6 より、 π/4≦φ+π/4≦5π/12 なので、

 1≦x≦(+1)/2 である。

このとき、 y=cosθ=(2−x)/√(6−4x) の増減を調べる。

 y’=(2x−2)/√(6−4x)3 より、1≦x≦(+1)/2 において、単調増加である。

 よって、 /2≦y≦/2 より、 θ≧π/6 が成り立つ。

 (イ)(ロ)から、結局 θ≧π/6 となり、θ=π/6 のとき、2つの半直線m、nは、Pが

円Kの周上のどこにあっても必ず、正方形ABCDと交わる。

 よって、求めるθの最大値は、π/6 である。  (終)


(コメント) θがπ/6を超える場合は、円Kの周上の点から半直線m、nを引いた場合に、
      正方形ABCDと交わらない場合が起こり得るということですね!



基本図形の表現

 複素数と絶対値によって、基本的な図形が得られる。

○ |z−α|=|z−β| ・・・ 線分αβの垂直2等分線

○ |z−α|=r ・・・ 中心α、半径 r の円

 両辺を平方して、 zz−α+α−r2=0 と書ける。

○ |z−α|=n|z−β| (nは実数)

  ・・・ 線分αβを n : 1 に内分・外分する点を直径の両端とする円(アポロニウスの円)

○ (α−γ)/(β−γ)=i ・・・ ∠αγβ=90°の直角2等辺三角形αγβ


 読者のために、練習問題を残しておこう。

問題 異なる複素数 α、β、γ について、(α−β)2+(β−γ)2=0 が成り立つとき、
   △αβγの形状を求めよ。

(解) (α−β)/(γ−β)=±i より、∠αβγ=90°の直角2等辺三角形αβγ (終)


問題 異なる3つの複素数α、β、γの間に、等式 (γ−α)/(β−γ)=i が成り立つ
   とき、△αβγの形状を求めよ。

(解) αを直角の頂点とする直角2等辺三角形  (終)


問題 ガウス平面上の3点A(α)、B(β)、C(γ)について、α−β=(−1+i)(γ−β)
    のとき、△ABCの形状を求めよ。

(解) (α−β)/(γ−β)=(−1+i) より、

  |(α−β)/(γ−β)|=2 で、 arg{(α−β)/(γ−β)}=2π/3

 よって、 AB : AC=2 : 1 で、∠ABC=120°の三角形  (終)


問題 ガウス平面上の異なる3点A(α)、B(β)、O(0)について、β/α=(1+i)/2
   が成り立つとき、△OABの形状を求めよ。

(解) |β|/|α|=1 より、 |α|=|β| で、

 arg(β/α)=π/3 より、∠AOB=60°なので、△OABは正三角形である。  (終)


問題 四角形ABCDにおいて、等式 AB2+CD2=AD2+BC2 が成り立つとき、2つの
   対角線は垂直に交わることを示せ。

(解) A(0)、B(β)、C(γ)、D(δ)とおくと、

 |β|2+|δ−γ|2=|δ|2+|γ−β|2 を展開して、

 |β|2+|δ|2−γδ+|γ|2=|δ|2+|γ|2−βγ+|β|2

すなわち、  γδ=βγ より、 γ/(β−δ)=−/(

 よって、 γ/(β−δ) は純虚数となるので、 AC⊥BD である。  (終)


問題 △OABの外側に、Oを頂点とし、辺OA、OBのそれぞれを1辺とする直角2等辺三角
   形OAC、OBDを作る。線分CDの中点をMとするとき、 AB=2OM 、AB⊥OM で
   あることを示せ。

          

(解) ガウス平面において、 O(0)、A(α)、B(β) とおく。このとき、題意より、

 C(-iα)、D(iβ) となる。よって、M(m) とおくと、 m=(β-α)i/2 である。

 このとき、 AB=|β−α|=2|m|=2OM で、

 m/(β-α)=i/2 より、 arg{m/(β-α)}=π/2 なので、 AB⊥OM  (終)


問題  ガウス平面の原点OとOとは異なる2点A、Bで、△OABを考える。辺OA、OBを斜
    辺として三角形の外側に直角2等辺三角形OPA、OQBを作る。
     辺ABの中点をMとするとき、MP=MQ、MP⊥MQ が成り立つことを示せ。

        

(解) A(α)、B(β)、P(p)、Q(q)、M(m) とおくと、題意より、

  −p=i・(α−p) より、 p=(1−i)α/2

 同様に、 β−q=−i・q より、 q=(1+i)β/2

 m=(α+β)/2 なので、

 MP=p−m=−(i・α+β)/2 、MQ=q−m=−(α−i・β)/2

よって、 MP=i・MQ となり、 MP=MQ、MP⊥MQ が成り立つ。  (終)


(コメント) 上記問題の相似回転による別解は、こちらを参照。


問題  四角形ABCDの外側に各辺を斜辺とする直角2等辺三角形PAB、QBC、RCD、
    SDAを作る。このとき、 PR=QS 、 PR⊥QS であることを示せ。

     

(解) A(α)、B(β)、C(γ)、D(δ)、P(p)、Q(q)、R(r)、S(s) とおくと、

 α−p=i・(β−p) より、 (1−i)p=α−i・β すなわち、 

   p=(α+β+i・(αーβ))/2

 β−q=i・(γ−q) より、 (1−i)q=β−i・γ すなわち、 

   q=(β+γ+i・(βーγ))/2

 γ−r=i・(δ−r) より、 (1−i)r=γ−i・δ すなわち、 

   r=(γ+δ+i・(γーδ))/2

 δ−s=i・(α−s) より、 (1−i)s=δ−i・α すなわち、 

   s=(δ+α+i・(δーα))/2

よって、

PR=(γ+δ+i・(γーδ))/2−(α+β+i・(αーβ))/2

  =(γ+δ−α−β+i・(γーδ−α+β))/2

同様に、

QS=(δ+α+i・(δーα))/2−(β+γ+i・(βーγ))/2

  =(−γ+δ+α−β+i・(γ+δーα−β))/2

 =i・(γ+δ−α−β+i・(γーδ−α+β))/2

なので、 QS=i・PR が成り立つ。

 したがって、 PR=QS 、 PR⊥QS である。  (終)


定点通過問題

問題  ガウス平面内の2定点A、Bをとり、直線ABの上側に任意に点Cをとり、△ABCを
    考える。辺CA、BCを1辺として三角形の外側に正方形PQAC、RSCBを作る。
     このとき、点Cによらず、直線QRはつねに定点を通ることを示せ。

 

(解) A(α)、B(β)、C(γ)、Q(q)、R(r)とおくと、

 q−α=i・(γ−α) より、 q=α+i・(γ−α)

 γ−β=i・(r−β) より、 r=β−i・(γ−β)

 このとき、 (q+r)/2=(α+β+i・(β−α))/2 より、線分QRの中点は、点Cによら

ずに一定となるので、直線QRはつねに定点を通る。  (終)


(コメント) 線分ABを実軸上にとり、A(−a)、B(a)とすると、 (q+r)/2=i・a となって、
      定点の場所が瞬時に分かるかな...。


問題  △ABCの底辺BCに平行な直線が辺AB、ACと交わる点をそれぞれD、Eとおく。
    線分BEとCDの交点をFとおくと、直線AFは、底辺BCの中点を通ることを示せ。

    

#チェバの定理より、 (AD/DB)(BM/MC)(CE/EA)=1 が成り立ち、DEとBCが平行

 であるから、 AD/DB=EA/CE すなわち、 (AD/DB)(CE/EA)=1

 よって、 BM/MC=1 すなわち、 BM=MC から、Mは辺BCの中点である。

 これを、敢えて複素数を用いて解いてみよう。

(解) ガウス平面において、A(0)、B(β)、C(γ)とおく。題意より、実数k(0<k<1)に

対して、 D(kβ) 、C(kγ) とおける。

 直線BEの方程式は、 (z−β)/(kγ−β)=()/(k

すなわち、 (k)z−(kγ−β)=β(k)−(kγ−β)=k(βγ)

同様にして、直線CDの方程式は、 (k)z−(kβ−γ)=−k(βγ)

2式より、を消去して、 (k2−1)(βγ)z=k(k−1)(β+γ)(βγ)

 条件より、 k≠±1 、βγ なので、 z={k/(k+1)}(β+γ)

よって、2直線の交点は、AB、ACが張る平行四辺形の対角線上にあることから、直線AF

は、辺BCの中点を通ることが分かる。  (終)


(コメント) 「2直線の交点を通る直線」の公式を用いれば、次のように解くことも出来る。

 ガウス平面において、A(0)、B(β)、C(γ)とおく。題意より、実数k(0<k<1)に

対して、 D(kβ) 、C(kγ) とおける。

 直線BEの方程式は、 (k)z−(kγ−β)=k(βγ)

 直線CDの方程式は、 (k)z−(kβ−γ)=−k(βγ)

なので、2直線の交点を通る直線の方程式は、同時に0ではない実数 m、n を用いて、

  m((k)z−(kγ−β)−k(βγ))

 +n((k)z−(kβ−γ)+k(βγ))=0

とおける。この直線が、原点を通るので、z=0を代入して、

  −mk(βγ)+nk(βγ)=0

条件より、 k≠0 、βγ なので、 m=n となる。

 このとき、

 (k)z−(kγ−β)+(k)z−(kβ−γ)=0

から、 (k−1)(()z−(β+γ))=0

 k≠1 なので、 ()z−(β+γ)=0 となる。

 これが、直線AFの方程式である。

 直線BCの方程式は、 (z−β)/(γ−β)=()/() より、

  ()z−(γ−β)=βγ

そこで、直線AFと直線BCの交点を求めると、

(γ−β)()z−(β+γ)()z=−(β+γ)(βγ)

から、 −2(βγ)z=−(β+γ)(βγ)

 βγ なので、 z=(β+γ)/2 となる。

 この点は、辺BCの中点である。


(コメント) 複素数の場合は、単に計算だけなので、面白みに欠ける場合もありますね!
      この問題については、チェバの定理が最も平易で簡単ですね。
      (→ 参考:「重心座標を用いた解法」)

軌跡

問題 複素数 z が、|z−2|=|z+2−4i|を満たすとき、|z−1|+|z−4|の最
   小値を求めよ。

(解) 複素数 z が、|z−2|=|z+2−4i|を満たすとき、2と−2+4iを結ぶ線分の垂

  直2等分線Lを描く。Lに関して、1と対称な点は、−2+3i なので、求める最小値は、

  3  (終)


問題 複素数 z が、|z|=2 を満たすとき、w=(2z−1)/(z−8) は、どんな図形を描く
   か。

(解) w=(2z−1)/(z−8) より、 z=(8w−1)/(w−2) なので、

   |z|=|8w−1|/|w−2|=2

 よって、 |8w−1|=2|w−2| すなわち、 |w−1/8|=(1/4)|w−2|

 求めるwの軌跡は、2点 1/8、2 を 1 : 4 に内分、外分する点を直径の両端とする円と
なる。

 このとき、直径の両端は、2点 1/2、−1/2 となるので、

 wは、円 |w|=1/2 を描く。  (終)


直角三角形の条件

 相異なる3点 z1、z2、z3 について、△z123 が直角三角形であるための必要十分条件
は、
   (z3 −z1)/(z2−z1) が純虚数


(コメント) 条件より、 ∠z213=±π/2 より、z12⊥z13 である。


問題 ガウス平面上に、3点A(α)、B()、C(1/α)があり、△ABCは、C=90°の直
   角三角形である。このとき、αの存在範囲を定めよ。

(解) 直角三角形の条件より、 (α−1/α)/(−1/α) は純虚数である。すなわち、

 (α−1/α)/(−1/α)+(−1/)/(α−1/)=0 より、

 (α2−1)/(α−1)+(2−1)/(α−1)=0

よって、 α≠1 のもとで、 α22=2 を満たす。

 ここで、 α=x+i・y (x、y は実数) とおくと、 x2−y2=1

 また、α≠1 より、 x2+y2≠1 なので、 x≠±1 である。

 以上から、αの存在範囲を図示すると、下図を得る。

    


(コメント) α=2+i とおくと、=2−i 、1/α=(2−i)/7 で、下図のような
      直角三角形が得られる。

    


 山形大学工学部(2022)で、次のような問題が出題されている。(一部、改題。)

問題  2次方程式 x2−2x+k=0 (kは実数の定数) の解を α、β とする。ガウス平
    面において、原点をOとし、A(α)、B(β)とするとき、△OABが直角三角形となるよ
    うにkの値を定めよ。

(解) 解の公式より、 x=1±√(1−k) で、題意より、 1−k<0 すなわち、 k>1

 このとき、 α、β=1±i・√(k−1)

 直角三角形となるので、 1+i・√(k−1)=i×(1−i・√(k−1))

すなわち、 1+i・√(k−1)=i+√(k−1) から、 √(k−1)=1

 よって、 k=2  (終)


(別解) 解の公式より、 x=1±√(1−k) で、題意より、 1−k<0 すなわち、 k>1

このとき、直角三角形の条件から、 {1+i・√(k−1)}/{1−i・√(k−1)}は純虚数

すなわち、{1+i・√(k−1)}/{1−i・√(k−1)}+{1−i・√(k−1)}/{1+i・√(k−1)}=0

よって、 {1+i・√(k−1)}2+{1−i・√(k−1)}2=0 より、 2−2(k−1)=0

 したがって、 k=2  (終)


正三角形の条件

 単位円上の異なる3点 z1、z2、z3 について、△z123 が正三角形であるための必要
十分条件は、
        1+z2+z3=0

 実際に、△z123 が正三角形ならば、平行四辺形z12(−z1)z3 から、z1+z2+z3=0
は明らか。

 逆に、z1+z2+z3=0 とする。ここで、z1=1 としても一般性を失わない。

 z2=cosθ+i・sinθ 、z3=cosφ+i・sinφ とおくと、条件より、

 cosθ+cosφ=−1 、 sinθ+sinφ=0

 cos2φ+sin2φ=1 より、 (cosθ+1)2+sin2θ=1 すなわち、 cosθ=−1/2

 このとき、 cosφ=−1/2 も成り立つ。

 したがって、 △z123 は正三角形となる。


 単位円上の異なる3点 z1、z2、z3 について、z1+z2+z3=0 のとき、△z123 が正三
角形となることは次のようにも示される。

 実際に、△z123 の外心は、原点Oで、(z1+z2+z3)/3=0 から、原点Oは重心でも
ある。

 よって、△z123 において、外心と重心が一致するので、△z123 は正三角形となる。


 一般に、次の公式も知られている。

 ガウス平面上の異なる3点 α、β、γ について、△αβγ が正三角形であるための必
要十分条件は、
        α2+β2+γ2−αβ−βγ−γα=0

 これを示せ、という問題が、大阪教育大学(2021)で出題されている。

 実際に、△αβγ が正三角形であるための必要十分条件は、

  (γ−β)/(α−β)=(α−γ)/(β−γ)

 すなわち、 α2+β2+γ2−αβ−βγ−γα=0 である。


問題  ガウス平面において、原点Oとは異なる2点A(α)、B(β)について、△OABが正
    三角形であるための必要十分条件は、 α2−αβ+β2=0

(解) 上記の公式で、γ=0 とおけば、 α2−αβ+β2=0 が得られる。  (終)


(別解) 正三角形であるためには、 α/β=(1±i)/2 が必要十分条件である。

 α22=(−1±i)/2=α/β−1 すなわち、 α2−αβ+β2=0  (終)


問題  ガウス平面において、原点Oとは異なる2点A(α)、B(α2)について、△OABが正
    三角形となる複素数αをすべて求めよ。

(解) 上記の問題から、 α2−α3+α4=0 で、α≠0 から、 α2−α+1=0

 よって、 α=(1±i)/2  (終)


問題  ガウス平面において、原点Oとは異なる3点A(α)、B(α2)、C(α3) について、
    四角形OABCは正方形にはなりえないことを示せ。

(解) 四角形OABCが正方形になるものと仮定する。このとき、対角線の交点は互いに他

 を2等分するので、 (α3+α)/2=α2/2 すなわち、 α2−α+1=0

 よって、上記の問題より、△OABは正三角形となり、四角形OABCは正方形にはなりえ

ない。  (終)


 ところで、異なる3つの複素数α、β、γについて、△αβγが正三角形であるための必
要十分条件が、

  α2+β2+γ2−αβ−βγ−γα=0

であることは、次のようにも示すことができる。

 ω=(−1+i)/2 とすると、 ω2+ω+1=0 、ω3=1 が成り立つ。

(α+ωβ+ω2γ)(α+ω2β+ωγ) を計算すると、

(α+ωβ+ω2γ)(α+ω2β+ωγ)

=α2+ω2αβ+ωγα+ωαβ+β2+ω2βγ+ω2γα+ωβγ+γ2

=α2+β2+γ2+(ω2+ω)αβ+(ω2+ω)βγ+(ω2+ω)γα

=α2+β2+γ2−αβ−βγ−γα

となる。よって、 α2+β2+γ2−αβ−βγ−γα=0 ならば、

 α+ωβ+ω2γ=0 または α+ω2β+ωγ=0

が成り立つ。

 α+ωβ+ω2γ=0 のとき、 α+ωβ+(−1−ω)γ=0 より、

   α−γ=ω(γ−β) すなわち、 (α−γ)/(γ−β)=ω から、

 △αβγは正三角形となる。

 α+ω2β+ωγ=0 のとき、 α+(−1−ω)β+ωγ=0 より、

   α−β=ω(β−γ) すなわち、 (α−β)/(β−γ)=ω から、

 △αγβは正三角形となる。

 何れにしても、α2+β2+γ2−αβ−βγ−γα=0 のとき、

3点α、β、γを頂点とする三角形は、正三角形となる。


 正三角形の条件として、次の事実もよく用いられる。

 ω=(−1+i)/2 は、1の3乗根で、大きさを変えない120°の回転を表す複素数
である。

 ω3=1 から、 ω2+ω+1=0 すなわち、 ω2=−ω−1 が成り立つ。

 このとき、

 △αβγが正の向き(反時計回り)に正三角形であるための必要十分条件は、

  α+βω+γω2=0


(証明) △αβγが正の向き(反時計回り)に正三角形であるとすると、

 α−γ=ω(γ−β) すなわち、 α+βω+γ(−ω−1)=α+βω+γω2=0

が成り立つ。

 逆に、α+βω+γω2=0 が成り立つとすると、 α−γ=ω(γ−β) が成り立ち

△αβγは正の向き(反時計回り)に正三角形となる。  (証終)


問題  △ABCの頂点Aのある側に正三角形PBCを描き、外側に正三角形QCA、RABを
    描くとき、四辺形AQPRは平行四辺形であることを示せ。

       

(解) ガウス平面において、A(α)、B(β)、C(γ)、P(p)、Q(q)、R(r)とおくと、題意より、

  p+βω+γω2=0 、q+αω+γω2=0 、r+βω+αω2=0

なので、 p=−βω−γω2 、q=−αω−γω2 、r=−βω−αω2

 よって、 AQ=q−α=−α−αω−γω2=(α−γ)ω2

同様に、 RP=p−r=−βω−γω2+βω+αω2=(α−γ)ω2

 以上から、 AQRP が成り立つので、四辺形AQPRは平行四辺形である。  (終)


問題  ガウス平面において、正六角形ABCDEFを考える。A(α)、B(β)のとき、Cの座
    標を求めよ。

(解) 題意より、求める点C(z)は、

 z=β+(α−β)(cos(±120°)+i・sin(±120°) (複号同順)

 =β+(α−β)(−1±i)/2 (複号同順)

 ={(−1±i)/2}α+{(3i)/2}β (複号同順)  (終)


 佐賀大学医学部(2022)で、次のような問題が出題されている。(一部、改題。)

問題  ガウス平面において、A(1)、B(z)、C(z2)とする。次の問いに答えよ。

(1) △ABCが正三角形となるように、z を定めよ。

(2) △ABCが直角三角形となるように、z の存在範囲を定めよ。

(解) 題意より、 z≠0、±1 である。

(1) (z2−1)/(z−1)=(1±i)/2 より、 z+1=(1±i)/2

 よって、 z=(−1±i)/2

(2) A=90°のとき、 (z2−1)/(z−1)=z+1 が純虚数より、 Re(z)=−1

B=90°のとき、 (z2−z)/(1−z)=−z が純虚数より、 Re(z)=0

c=90°のとき、 (z−z2)/(1−z2)=z/(1+z) が純虚数より、

 z/(1+z)+/(1+)=0 なので、 2z+z+=0

 すなわち、 |z+1/2|=1/2

 以上から、求める z の存在範囲は、下図の青色の実線部分である。

      (終)


長方形の条件

 お茶の水女子大学(2016)で、次のような問題が出題された。一部改題しました。

問題  ガウス平面上の4点A(α)、B(β)、C(γ)、D(δ)は、原点Oを中心とする半径1
    の円周上にあるものとする。α+β+γ+δ=0 のとき、A、B、C、Dを頂点とする
    四角形は長方形であることを示せ。

(解) 線分AB、CDの中点をそれぞれM(m)、N(n)とおくと、

     m=(α+β)/2 、n=(γ+δ)/2

(1) α+β=0 のとき、 γ+δ=0 なので、 3点O、M、Nは一致する。

  よって、四角形の対角線が互いに他を2等分するので、四角形は平行四辺形である。

(2) α+β≠0 のとき、 (α+β)/2=−(γ+δ)/2 より、MとNは、原点Oに関して

  点対称で、3点O、M、Nは同一直線上にある。

 このとき、AM=BM より、 OM⊥AB である。同様にして、 ON⊥CD である。

 よって、 ABとCDは平行となる。

 また、OM=ON より、 AB=CD なので、四角形は平行四辺形である。

 以上から、(1)(2)の何れの場合も、α+β+γ+δ=0 のとき、四角形は平行四辺形
となる。

 この四角形は円に内接するので、対角は等しく、その和は180°である。すなわち、

 四角形の内角は全て直角である。

 このような平行四辺形は、長方形しかない。  (終)


問題  四辺形ABCDの各辺の外側に、正三角形ABP、BCQ、CDR、DASを作る。
    このとき、四辺形PQRSが正方形ならば、四辺形ABCDも正方形であることを示せ。

  

(解) ガウス平面において、A(α)、B(β)、C(γ)、D(δ)とおき、さらに、P(p)、Q(q)、

 R(r)、S(s)とおく。

 △ABPは正三角形なので、 p+αω+βω2=0 すなわち、 p=−αω−βω2

同様にして、 q=−βω−γω2 、r=−γω−δω2 、s=−δω−αω2

四辺形PQRSは正方形なので、 (p+r)/2=(q+s)/2 より、

 −αω−βω2−γω−δω2=−βω−γω2−δω−αω2

すなわち、 α(ω2−ω)−β(ω2−ω)+γ(ω2−ω)−δ(ω2−ω)=0

ω2−ω≠0 なので、 α−β+γ−δ=0 より、 (α+γ)/2=(β+δ)/2

したがって、四辺形ABCDは平行四辺形である。

 また、 s−p=i・(q−p) なので、

 −δω−αω2+αω+βω2=i・(−βω−γω2+αω+βω2

すなわち、 α(−ω2+ω−i・ω)+β(ω2+i・ω−i・ω2)+γ・i・ω2−δω=0

 ここで、α−β+γ−δ=0 より、 γ=−α+β+δ なので、

 α(−ω2+ω−i・ω−i・ω2)+β(ω2+i・ω)+δ(−ω+i・ω2)=0

すなわち、 −α(ω2+i・ω)−i・α(ω2+i・ω)+β(ω2+i・ω)+i・δ(ω2+iω)=0

ω2+i・ω≠0 なので、 −α−i・α+β+i・δ=0 より、 β−α=−i・(δ−α)

したがって、AB=AD 、AB⊥AD なので、平行四辺形ABCDは正方形である。  (終)


問題  相異なる4点A(α)、B(β)、C(γ)、D(δ)が正方形ABCDとなるための必要十
    分条件は、

  α−β=δ−γ かつ (β−α)2+(δ−α)2=0

(解) BACD で、 AB=±i・AD なので、

  α−β=δ−γ かつ β−α=±i・(δ−α)

すなわち、 α−β=δ−γ かつ (β−α)2=−(δ−α)2 から、明らか。  (終)


問題  2つの2次方程式 x2+ax+b=0 、x2+x+1=0 の解が、ガウス平面上で正
   方形の4頂点となるとき、定数 a、b の値を求めよ。

(解) x2+x+1=0 の解は、α=(−1+i)/2 、β=(−1−i)/2

 x2+ax+b=0 の解をγ、δとすると、解と係数の関係から、 γ+δ=−a、γδ=b

このとき、4頂点α、β、γ、δが正方形αβγδの4頂点となるための必要十分条件は、

 α−β=δ−γ かつ (β−α)2+(δ−α)2=0

である。

 α−β=i より、 δ−γ=i すなわち、 γ=δ−

また、 −3+(δ−α)2=0 より、

 δ=α±=(2−1+i)/2 、(−2−1+i)/2

このとき、

 γ=(2−1−i)/2 、(−2−1−i)/2

よって、 a=−(γ+δ)=1−2 、1+2

 b=γδ=((±2−1)2+3)/4=4− 、4+

 また、4頂点α、β、γ、δが正方形αδβγの4頂点となるための必要十分条件は、

 α−δ=γ−β かつ (γ−δ)2+(α−β)2=0

である。 α+β=γ+δ より、 a=−(γ+δ)=−(α+β)=1

 γ+δ=−1 、(γ−δ)2=−(α−β)2=3 より、 

 b=γδ=((γ+δ)2−(γ−δ)2)/4=−1/2

以上から、

(a,b)=(1−2,4−)、(1+2,4+)、(1,−1/2)  (終)


 ks さんからご投稿いただきました。(令和4年9月14日付け)

トレミーの不等式

 異なる4点が左回りにA(α)、B(β)、C(γ)、D(δ)とあるとき、

  AB・CD+AD・BC≧AC・BD

が成り立つ。


 特に、四角形ABCDが円に内接するとき、等号が成立する。それは、トレミーの定理と呼
ばれる。(→ 参考:「プトレマイオスの定理」)

 初等幾何的証明もありますが、ガウス平面を利用しても証明されます。、

 |x|+|y|≧|x+y|を使って、

|α−β||γーδ|+|αーδ||βーγ|

≧|(α−β)(γーδ)+(αーδ)(βーγ)|

=|αγ−αδーβγ+βδ+αβーαγーδβ+δγ|

=|αβーαδ+δγーβγ|

=|α(βーδ)+γ(δーβ)|

=|(αーγ)(βーδ)|=|αーγ||βーδ|

 よって、与式が示された。


 ks さんからのコメントです。(令和4年9月16日付け)

 等号が成り立つ

⇔ (α−β)(γーδ)=k(αーδ)(βーγ)

⇔ (α−β)(γーδ)/(αーδ)(βーγ)=k (実数)

⇔ 偏角を取ると、0ではなく、180°の場合になる

⇔ arg((α−β)(γーδ)/(αーδ)(βーγ))=π

⇔ arg(α−β/αーδ)+arg(γーδ/γーβ)=π

⇔ ∠DAB+∠DCB=π (対角の和がπ)

⇔ A、B、C、Dは同一円周上の点


平行と垂直

 相異なる4つの複素数 α、β、γ、δ に対して、2つの線分αβ、γδの位置関係を考
える。

(1) αβ、γδ が平行 ⇔ (α−β)/(γ−δ)は実数

(2) αβ、γδ が垂直 ⇔ (α−β)/(γ−δ)は純虚数

 上記のことは、 α−β と γ−δ の偏角を考えれば明らかだろう。


(追記) 令和4年9月24日付け

 ガウス平面上の異なる3点A、B、Cについて、△ABCが正三角形となる場合を問う問題
が、青山学院大学 理工(2018)、立教大学 理(2020)等で出題されている。

問題  ガウス平面上の異なる3点A(z)、B(z2)、C(z3)について、△ABCが正三角形と
    なるような z の値をすべて求めよ。

(解) 条件から、z≠z2 、z≠z3 なので、 z≠0、1、−1 である。

・Bを中心に、辺BCをπ/3回転してBAとなる場合

 (z−z2)/(z3−z2)=cos(π/3)+i・sin(π/3)

・Bを中心に、辺BAをπ/3回転してBCとなる場合

 (z3−z2)/(z−z2)=cos(π/3)+i・sin(π/3)

 以上の2つの場合は、一つの式にまとめられる。すなわち、

  (z3−z2)/(z−z2)=cos(±π/3)+i・sin(±π/3)=1/2±i・(/2)

 よって、 −z=1/2±i・(/2) より、 z=−1/2±i・(/2)  (終)


 ks さんからのコメントです。(令和4年9月24日付け)

ブラマグプタの定理(→ 参考:「ブラマグプタの定理」)

 円Oに内接する四角形ABCDで、AC⊥BDのとき、対角線の交点Pを通る線分Lが
CDと垂直のとき、LとABとの交点MはABの中点である


   


 円に内接してなくても似た定理が成り立つ。

 PA=PD、PB=PC のとき、四角形ABCDで、AC⊥BD のとき、対角線の交点Pを通
る線分LがCDと垂直のとき、LとABとの交点MはABの中点である

(証明) Pを原点とし、A(α)、B(β)とおくと、PA、PBを回転して、C(βi)、D(-αi)となる。

 このとき、M((α+β)/2) である。

 CD方向は、(β+α)i なので、PMはCDに垂直である。  (証終)


 この結果を使うと、四角形ABCDが円に内接する場合が次のように示される。

(証明) PA、PBの直線上に、PA’=PD、PB’=PC となるようにA’、B’をとれば、線分A’B’

 の中点M’について、PM’はCDと垂直になる。

 一方、△PAB∽△PDC なので、PA/PD=PB/PC 即ち、PA/PA’=PB/PB’である。

 よって、AB とA’B’ は平行となり、直線PM’は、ABの中点Mを通る。  (証終)


プトレマイオスの定理

    四角形ABCDが円に内接するとき、

      AC・BD=AB・CD+AD・BC

   が成り立つ。

(証明) 上図のように、対角線の交点をガウス平面の原点とし、各頂点に複素数を割り付
    ける。

 四角形ABCDは、円に内接する四角形なので、∠BAD+∠DCB=180° が成り立つ。

すなわち、
      
 よって、
        
 となる。

 いま、 Z=(δ−α)(β−γ)、W=(β−α)(δ−γ) とおくと、

            

よって、原点 O と、2点 Z、−W の位置関係は、例えば、下記のようになっている。

          

 したがって、 |Z−W|=|Z|+|−W|=|Z|+|W| が成り立つ。

ここで、簡単な計算により、

     Z−W=(δ−α)(β−γ)−(β−α)(δ−γ)=(α−γ)(δ−β)

なので、

    |(α−γ)(δ−β)|=|(δ−α)(β−γ)|+|(β−α)(δ−γ)|

が成り立つ。これより、 AC・BD=AB・CD+AD・BC が得られる。  (証終)


相似条件 ・・・ 裏返しなし(「同じ向きに相似」と言われる)

 △z123 ∽ △w123 ⇔ (z3−z1)/(z2−z1)=(w3−w1)/(w2−w1

 この条件は、2辺の比と夾角相等である。辺と角の関係が、1本の式で表現できることが
複素数の醍醐味である。

 裏返しあり(「反対向きに相似」と言われる)の場合は、 △z123 と △ が同
じ向きに相似になればよい。


 ある条件のもとで、2つの三角形の重心が一致するという問題は、図形問題では定番の
問題だろう。

問題  △ABCにおいて、A(α)、B(β)、C(γ)とする。下図のように、△ABCの各辺上
    に、△ABCと同じ向きに相似な△BCA’、△CAB’、△ABC’を作る。
    ここで、A’(α’)、B’(β’)、C’(γ’)とする。

    

 このとき、△ABCと△A’B’C’の重心は一致することを示せ。

(解) 題意より、 (α’−γ)/(β−γ)=(β’−α)/(γ−α)=(γ’−β)/(α−β)
   が成り立つ。そこで、等式=w とおくと、

 α’−γ=w(β−γ) 、β’−α=w(γ−α) 、γ’−β=w(α−β)

すなわち、 α’=γ+w(β−γ) 、β’=α+w(γ−α) 、γ’=β+w(α−β)

 3式を辺々加えて、 α’+β’+γ’=α+β+γ

すなわち、 (α’+β’+γ’)/3=(α+β+γ)/3 なので、

 △ABCと△A’B’C’の重心は一致する。  (終)


(追記) よおすけさんからの問題提供です。(令和4年10月8日付け)

 大阪大学前期(2007)の文理共通で出題された問題です。

問題  xy平面において、原点Oを通る半径 r (r>0)の円をCとし、その中心をAとする。Oを
    除くC上の点Pに対し、次の2つの条件(a)、(b)で定まる点Qを考える。

(条件)(a) OPOQの向きが同じ
    (b) |OP||OQ|=1

 以下の問いに答えよ。

(1) 点PがOを除くC上を動くとき、点QはOAに直交する直線上を動くことを示せ。

(2) (1)の直線をLとする。LがCと2点で交わるとき、r のとりうる値の範囲を求めよ。


 分野は数学B(ベクトル)となっていますが、複素数を用いて解いてみたいと思います。

(解) ガウス平面において、A(α)、P(z)、Q(w)とすると、(条件)(a)、(b)は次のようにな
   る。

 (条件) (a) w=kz (k>0)  (b) |z||w|=1

(1) 題意より、 |z−α|=r 、|α|=r である。条件(a)より、w=kz (k>0)と書けるので、

  |z||w|=1 より、 k=1/|z|2 すなわち、 z=|z|2

 このとき、 |z−α|=||z|2w−α|=r の両辺を平方して、

  |z|4|w|2−|z|2(wα)+|α|2=r2

 すなわち、 |z|4|w|2−|z|2(wα)=0 で、|z|≠0 から

  |z|2|w|2−(wα)=0 すなわち、 wα=1 が言える。

   α=a+i・b 、w=x+i・y とおくと、 ax+by=1/2

 よって、点Qは、OAに垂直な直線上を動く。

(2) 直線Lの方程式は、 ax+by=1/2

 題意を満たすためには、A(a,b)と、直線L:ax+by=1/2 の距離が r 未満であればよい。

 |a2+b2−1/2|/√(a2+b2)<r

 ここで、 a2+b2=r2 なので、 |r2−1/2|/r<r すなわち、 |r2−1/2|<r2

 よって、 −r2<r2−1/2<r2 より、 r2>1/4 すなわち、 r>1/2  (終)


(追記) 令和4年10月22日付け

 平面幾何の問題で、次の定理は有名である。ナポレオンが発見したと伝えられているが、
そのことを示す資料はないとか...。

ナポレオンの定理

 △ABCの外側に辺BC、CA、ABを1辺とする正三角形を作る。それらの正三角形
の重心を結んで出来る三角形は正三角形である


 複素数を用いれば、簡単に示される。

(証明) △ABCにおいて、A(α)、B(β)、C(γ)とし、正三角形の残りの頂点をそれぞれ
    P(p)、Q(q)、R(r) とする。

   

 G(g)、H(h)、I(i)とおくと、

 β−g=ω(γ−g) すなわち、 (1−ω)g=β−ωγ

 γ−h=ω(α−h) すなわち、 (1−ω)h=γ−ωα

 α−i=ω(β−i) すなわち、 (1−ω)i=α−ωβ

このとき、

 (1−ω)(g+hω+iω2)=β−ωγ+ωγ−ω2α+ω2α−β=0

なので、 g+hω+iω2=0 が成り立つ。

 よって、 △GHI は正三角形である。  (証終)


 正三角形GHI はナポレオンの三角形(ナポレオンの外三角形)と呼ばれる。上記では、
△ABCの外側に辺BC、CA、ABを1辺とする正三角形を作ったが、内側に作ってもナポレ
オンの三角形(ナポレオンの内三角形)を得ることができる。


 ナポレオンの三角形は、いろいろな性質を有している。

(1) △AIH∽△ARC で、相似比は、1/ である。

 実際に、

  △AIHと△ARCにおいて、 ∠IAH=∠RAC で、AI=AR/、AH=AC/
 だからである。


(2) 3つの線分AG、BH、CI は1点で交わる。(この交点は、ナポレオン点と言われる)

 実際に、 
       

 線分AGと辺BCの交点をK、線分BHと辺CAの交点をL、線分CI と辺ABの交点をMとお
く。このとき、

 △AIC=(1/2)AI・AC・sin∠CAI

     =(1/2)(1/)AB・AH・sin∠BAH=(1/2)AB・AH・sin∠BAH=△ABH

 同様にして、 △BGA=△BCI 、△CHB=△CAG が成り立つ。

 よって、

(AM/MB)(BK/KC)(CL/LA)

=(△AIC/△BCI)(△BGA/△CAG)(△CHB/△ABH)

=(△ABH/△BCI)(△BCI/△CAG)(△CAG/△ABH)

=1

となり、チェバの定理より、3つの線分AG、BH、CI は1点で交わる。


(3) △ABCの重心とナポレオンの三角形GHI の重心は一致する。

 実際に、

 △ABCの重心を表す複素数は、 (α+β+γ)/3

 △GHI の重心を表す複素数は、

 (g+h+i)/3=(β−ωγ+γ−ωα+α−ωβ)/(3(1−ω))=(α+β+γ)/3

なので、△ABCと△GHI のそれぞれの重心は一致する。


(4) ナポレオンの外三角形とナポレオンの内三角形の面積の差は、△ABCの面積に等し
   い。

 実際に、
      

 上図において、ナポレオンの外三角形の1辺の長さをXとおくと、△AIHにおいて、余弦定
理より、

 X2=(AB/2+(AC/2-2(AB/)(AC/)cos(A+60°)

  =(AB2+AC2-2・AB・AC・cos(A+60°))/3

 よって、△GHI=(/4)X2=(/12)(AB2+AC2-2・AB・AC・cos(A+60°))

 同様にして、ナポレオンの内三角形の1辺の長さをYとおくと、△AI’H’において、余弦定
理より、

 Y2=(AB/2+(AC/2-2(AB/)(AC/)cos(A−60°)

  =(AB2+AC2-2・AB・AC・cos(A−60°))/3

 よって、△G’H’I’=(/4)Y2=(/12)(AB2+AC2-2・AB・AC・cos(A−60°))

 このとき、

 △GHI−△G’H’I’

=(/6)AB・AC・(cos(A−60°)−cos(A+60°))

=(/3)AB・AC・sinAsin60°

=(1/2)AB・AC・sinA=△ABC

が成り立つ。


(5) 下図において、四角形AIG’Hは平行四辺形となる。

      

 実際に、A(α)、I((α−ωβ)/(1−ω))、H((γ−ωα)/(1−ω))で、G(g’)とすると、

 γ−g’=ω(β−g’) より、 g’=(γ−ωβ)/(1−ω)

よって、 AI=(α−ωβ)/(1−ω)−α=ω(α−β)/(1−ω)

 HG’=(γ−ωβ)/(1−ω)−(γ−ωα)/(1−ω)=ω(α−β)/(1−ω)

となるので、 AIHG’ すなわち、四角形AIG’Hは平行四辺形となる。


(コメント) A=120°のとき、4点A、I、G’、Hは1直線上にあり、四角形AIG’Hは潰れて
      しまう。


 首都大学東京(現 都立大学)前期理系(2017)で次のような問題が出題されている。
問題は、一部改題。

問題  ガウス平面上の原点Oと2点A(2−4i)、B(3+i)を考える。ただし、i を虚
    数単位とする。△OABの外側に、3辺AB、BO、OAをそれぞれ1辺とする正三角形
    △ALB、△BMO、△ONAを作る。次の問いに答えよ。

        

(1) 点L、M、Nを表す複素数をそれぞれ求めよ。
(2) 直線QLと直線AMの交点をPとするとき、点Pを表す複素数を求めよ。
(3) 3点B、P、Nが一直線上にあることを示せ。 

(解)(1) w=(1+i)/2 として、

 α−(3+i)=w((2−4i)−(3+i))=7−3i より、 α=10−2

 同様にして、 β=w(3+i)=2

 γ−(2−4i)=w(−2+4i)=−7+i より、 γ=−5−3

(2) 直線AM上の点は、 m(2−4i)+(1−m)(2i)=2m+(2−6m)

 直線OL上の点は、 n(10−2i)=10n−2ni (ただし、m、nは実数) と書ける。

 よって、 2m+(2−6m)i=10n−2ni から、

  2m=10n 、 2−6m=−2n なので、 m=5/14 、n=1/14

 よって、点Pを表す複素数は、 (5−i)/7

(3) NB=(3+i)−(−5−3i)=8+4i=4(2+i)

 NP=(5−i)/7−(−5−3i)=(20/7)(2+i) なので、 NP=(5/7)NB

 よって、 3点B、P、Nが一直線上にある。  (終)


 今まで、複素数の底力を問題を通して見てきたが、次の円周角の定理は、初等幾何で示
すのが最も簡単だろう。複素数を用いて示せないことはないが、若干煩わしさを感じる。

円周角の定理  円周上の劣弧を見込む円周角φは、中心角θの半分

 初等幾何では、次のように示される。

     

(証明) θ=2(×+○)=2φ より、 φ=(1/2)θ  (証終)


 ちょっと気が進まないが、複素数を用いて定理を証明してみよう。

(証明) O(0)、A(α)、B(β)、P(z)とおく。円Oの半径は1としても一般性を失わない。

    

 上図において、∠AOB=∠αOβ=θ すなわち、 θ=arg(β/α) である。

また、円周上の点P(z)について、 ∠APB=φ=arg(β−z)/(α−z) である。

 題意より、 |α|=|β|=|z|=1 なので、 α=β=z=1

このとき、

 2φ=2arg(β−z)/(α−z)

   =arg{(β−z)/(α−z)}2

   =arg{(β−z)/(α−z)}{(1/−1/)/(1/−1/)}

   =arg{(β−z)/(α−z)}{()/()}(/

   =arg|(β−z)/(α−z)|2(β/α)

   =arg(β/α)=θ  である。  (証終)


垂線の足 ・・・ 垂線の足を求める方法も基本的な技と考えられる。

 原点Oが中心で半径1の円周上に2点A(α)、B(β)がある。平面上の点P(p)より、直線
ABに垂線を下し、その足をH(h)とする。このとき、

   h=(α+β+p−αβ)/2

である。
     

#よく用いられる公式なので、覚えた方がいいだろう。

(証明) 3点A、B、Hは同一直線上にあるので、 (β−h)/(α−h)は実数である。

 すなわち、 (β−h)/(α−h)=()/(

分母を払って、 ()(β−h)=(α−h)(

 =1/α、=1/β を代入して整理すると、 h+αβ−α−β=0 ・・・ (1)

 また、 PH⊥AB より、 (p−h)/(β−α)は純虚数である。

 すなわち、 (p−h)/(β−α)+()/()=0

 =1/α、=1/β を代入して整理すると、 p−h−αβ+αβ=0 ・・・ (2)

よって、 (1)−(2)より、 −p+2h+αβ−α−β=0 なので、

  h=(α+β+p−αβ)/2

が成り立つ。  (証終)

 特に、PがOに一致するとき、当然ながら、 h=(α+β)/2 という中点の公式が成り立つ。


 上記の垂線の足の公式を用いて、次のシムソンの定理が示される。

シムソンの定理

 △ABCの外接円の周上の点Pから直線BC、CA、ABに下した垂線の足D、E、Fは
同一直線上にある

     

 垂線の足を通る直線はシムソン線と呼ばれる。

 この定理の複素数による証明は、「シムソンの定理」でも述べたが、再度検討し、証明を
再構成してみた。

 証明に入る前に、初等幾何による証明(略証)を覗いておこう。複素数による証明よりも簡
明かもしれない。

(証明) 上図において、

 4点B、D、F、Pが同一円周上にあるので、 ∠PBD+∠PFD=180°

  すなわち、 ∠PFD=180°−∠PBD

 同様に、4点P、F、A、Eが同一円周上にあるので、 ∠PFE=∠PAE

  すなわち、 ∠PFE=180°−∠PAC

 よって、 ∠PFD+∠PFE=360°−(∠PBD+∠PAC)

 ここで、4点A、P、B、Cは同一円周上にあるので、 ∠PBD+∠PAC=180°

以上から、∠PFD+∠PFE=180°となり、3点D、E、Fは同一直線上にある。  (証終)


(複素数によるシムソンの定理の証明)

 上図において、A(α)、B(β)、C(γ)、P(p)とおく。垂線の足D(d)、E(e)、F(f)とおくと、

垂線の足の公式から、=1/p に注意して、

 d=(β+γ+p−βγ)/2=(β+γ+p−βγ/p)/2

 e=(γ+α+p−γα)/2=(γ+α+p−γα/p)/2

 f=(α+β+p−αβ)/2=(α+β+p−αβ/p)/2

が成り立つ。このとき、

 d−e=(β+γ+p−βγ/p)/2−(γ+α+p−γα/p)/2=(β−α)(p−γ)/(2p)

 d−f=(β+γ+p−βγ/p)/2−(α+β+p−αβ/p)/2=(γ−α)(p−β)/(2p)

なので、

 (d−e)/(d−f)=(β−α)(p−γ)/{(γ−α)(p−β)}

そこで、

(β−α)(p−γ)/{(γ−α)(p−β)}−()()/{()()}

=(β−α)(p−γ)/{(γ−α)(p−β)}−(β−α)(p−γ)/{(γ−α)(p−β)}=0

なので、 (d−e)/(d−f) は実数となる。

 よって、3点D、E、Fは同一直線上にある。  (証終)


 京都大学 前期理系(2004)で、次のような問題が出題されている。

問題  αを実数ではない複素数とする。ガウス平面内の円Cが、1、−1、αを通るなら
    ぱ、Cは −1/ も通ることを示せ。

       


(コメント) 円Cが原点中心の円であれば、半径が1なので、 α=1 で、 −α=−1/

明らかに、−αは原点に関してαと対称な点で、もちろん、円C上の点となる。

すなわち、−1/ は円C上にある。

 このことを、一般の場合にも成り立つことを示せという問題が、京都大学の問題である。


(解) 題意より、円Cの方程式は、 |z−ci|= (cは実数) とおける。

 点αを通るので、 |α−ci|=

両辺を2乗して、 (α−ci)(+ci)=c2+1

すなわち、 α+(ci)α−(ci)=1 となる。

両辺を α で割って、 1+(ci)/−(ci)/α=1/(α

すなわち、 1/(α)+(ci)/α−(ci)/=1

 よって、 (−1/−ci)(−1/α+ci)=c2+1 から、 |−1/−ci|=

これは、点−1/ が円C上にあることを示す。  (終)


 意外なところに正三角形」で、KS さんより、有名なモーリ―の定理をご紹介いただいた。

 角の3等分は、定木・コンパスを用いては不可能なので、モーリーの定理の図を作図する
ことは不可能である。「私の備忘録」においても、「モーリーの定理」として、問題提起はして
あったが、初等幾何で示すのは半端なく難しく手が出せないでいた。しかし、結果自体は非
常に美しい定理である。

モーリ―の定理(Morley’s trisector theorem) ・・・ 1899年米国のモーリーにより証明

 三角形の3つの内角の3等分線の、辺に近い2つずつの交点は、正三角形の頂点を
なす

     

 三角関数を使う手も考えられるが、やはり半端ない計算になる...予感。(→ 参考

 複素数を用いると、どのような計算になるのか、怖いもの見たさに挑戦してみた。

(証明) △ABCの外接円の中心をO(0)とする。一般性を失うことなく、半径は、1としてよい。

 また、C(1)とし、A(α)、B(β)とおく。

   

 ここで、 arg(α)=6θ 、arg(β)=6φ とおく。 0<6θ<6φ<2π である。

 このとき、 A=(2π−6φ)/2=π−3φ 、B=6θ/2=3θ で、

 C=π−(π−3φ)−3θ=3(φ−θ) となる。

 ここで、 e^(2θi)=a 、e^(2φi)=b とおくと、 α=a3 、β=b3 と書ける。

 いま、P(p)、Q(q)、R(r)とおく。

まず、 AR=r−α=ke^((π/3−φ)i)(β−α) (kは正の数)

両辺の共役複素数を求めて、 =ke^(−(π/3−φ)i)(

これらを辺々割って、 (r−α)/()=e^((2π/3−2φ)i)(β−α)/(

 ここで、α=1 、β=1 、e^((2π/3−2φ)i)=ω/b (ただし、ωは1の3乗根)

なので、 (r−α)/(−1/α)=−ωαβ/b より、 b(r−α)=−ωαβ(−1/α)

ω3=1 なので、 ω2b(r−α)=β−αβ=b3−a33

 よって、 ω2(r−α)=b2−a32 となる。 ・・・ (*)

同様に、 BR=r−β=he^(−θi)(α−β) (hは正の数)

両辺の共役複素数を求めて、 =he^(θi)(

これらを辺々割って、 (r−β)/()=e^((−2θ)i)(α−β)/(

 ここで、α=1 、β=1 、e^((−2θ)i)=1/a なので、

 (r−β)/(−1/β)=−αβ/a より、 a(r−β)=−αβ(−1/β)

すなわち、 a(r−β)=α−αβ=a3−a33 より、 r−β=a2−a23 ・・・ (**)

(*)と(**)から、 を消去して、 bω2(r−α)−a(r−β)=β−α

 すなわち、 (bω2−a)r=β−α+bαω2−aβ より、

 r=(β−α+bαω2−aβ)/(bω2−a)

ここで、 β−α=b3−a3=(b−a)(b−aω)(b−aω2

 bω2−a=(b−aω)/ω なので、

 r=ω(b−a)(b−aω2)+(bαω2−aβ)/(bω2−a)

ここで、

 bαω2−aβ

=a3bω2−ab3

=ab(a2ω2−b2)=−ab(b+aω)(b−aω)=−abω(b+aω)(bω2−a)

なので、

 r=ω(b−a)(b−aω2)−abω(b+aω)=−a2bω2−ab2ω+a2+abω2+b2ω

同様に、

 BP=p−β=k’e^(θi)(1−β) (k’は正の数)

両辺の共役複素数を求めて、 =k’e^(−θi)(1−

これらを辺々割って、 (p−β)/()=e^(2θi)(1−β)/(1−

 ここで、β=1 、e^(2θi)=a なので、

 (p−β)/(−1/β)=−aβ より、

 p−β=−aβ(−1/β)=−aβ+a ・・・ (***)

同様に、 CP=p−1=h’e^((θ−φ)i)(β−1) (h’は正の数)

両辺の共役複素数を求めて、 −1=h’e^((φ−θ)i)(−1)

これらを辺々割って、 (p−1)/(−1)=e^((2θ−2φ)i)(β−1)/(−1)

 ここで、β=1 、e^((2θ−2φ)i)=a/b なので、

 (p−1)/(−1)=−(a/b)β より、 b(p−1)=−aβ(−1)=−ab3+ab3

すなわち、 p−1=−ab2+ab2 ・・・ (****))

(***))と(****))から、 を消去して、 p−β−b(p−1)=a−aβ

 すなわち、 (1−b)p=a−b+(1−a)β より、

 p=(a−b+(1−a)β)/(1−b)

ここで、

 a−b+(1−a)β

=a−b+(1−a)b3

=a(1−b3)−b(1−b2)=(1−b)(ab2+ab+a−b2−b) なので、

 p=ab2+ab+a−b2−b

同様に、

CQ=q−1=k”e^((φ−θ)i)(α−1) (k”は正の数)

両辺の共役複素数を求めて、 −1=k”e^(θ−φ)i)(−1)

これらを辺々割って、 (q−1)/(−1)=e^((2φ−2θ)i)(α−1)/(−1)

 ここで、α=1 、e^((2φ−2θ)i)=b/a なので、

 (q−1)/(−1)=−bα/a より、 a(q−1)=−bα(−1)=−a3+a3

 よって、 q−1=−a2+a2b となる。 ・・・ (*****)

同様に、 AQ=q−α=h”e^(−(π/3−φ)i)(1−α) (h”は正の数)

両辺の共役複素数を求めて、 =h”e^((π/3−φ)i)(1−

これらを辺々割って、 (q−α)/()=e^(−(2π/3−2φ)i)(1−α)/(1−

 ここで、α=1 、e^(−(2π/3−2φ)i)=b/ω (ただし、ωは1の3乗根)なので、

 (q−α)/(−1/α)=−bα/ω より、 ω(q−α)=−bα(−1/α)

すなわち、 ω(q−α)=b−bα=b−a3 より、 

 q−α=bω2−a3bω2 ・・・ (******)

(*****)と(******)から、 を消去して、

 aω2(q−1)−(q−α)=a3bω2−bω2

 すなわち、 (aω2−1)q=a3bω2−bω2+aω2−a3 より、

 q=(a3bω2−bω2+aω2−a3)/(aω2−1)

ここで、

 a3bω2−bω2+aω2−a3

=bω2(a3−1)+aω2−1−(a3−1)

=(bω2−1)(a3−1)+aω2−1

=(bω2−1)(a−1)(a−ω)(a−ω2)+aω2−1

=(b−1/ω2)(a−1)(aω2−1)(a−ω2)+aω2−1

=((b−ω)(a−1)(a−ω2)+1)(aω2−1)

=(a2−abω2−ab+bω2−a2ω+a+aω)(aω2−1)

=(a2b+abω+bω2−a2ω−aω2)(aω2−1)

なので、 q=a2b+abω+bω2−a2ω−aω2 である。

 以上から、

 p=ab2+ab+a−b2−b

 q=a2b+abω+bω2−a2ω−aω2

 r=−a2bω2−ab2ω+a2+abω2+b2ω

について、

p+qω+rω2

=(ab2+ab+a−b2−b)+(a2bω+abω2+b−a2ω2−a)
 −a2bω−ab2+a2ω2+abω+b2

=ab(ω2+ω+1)

=0

 よって、p+qω+rω2=0 なので、△PQRは正三角形である。


(コメント) 計算の方針は素朴ですが、実際に計算してみると、かなりハードですね!


 三角関数を用いた証明がどうなるのか、興味があり、挑戦してみた。

 PQ=QR=RP であることを示そうと思う。

     

 △ABCの外接円の半径を、1/2 とし、A=3θ、B=3φ、C=3τ とおくと、

  θ+φ+τ=π/3

が成り立つ。まず、正弦定理より、 AB=sin3τ 、BC=sin3θ 、CA=sin3φ である。

 △PBCにおいて、正弦定理より、

 PB/sinτ=BC/sin(π−(φ+τ))=sin3θ/sin(φ+τ)

なので、 PB=sinτ・sin3θ/sin(φ+τ)

 ここで、 sin3θ=3sinθ−4sin3θ=4sinθ(3/4−sin2θ) で、さらに、

3/4−sin2θ=sin2(π/3)−sin2θ=(sin(π/3)+sinθ)(sin(π/3)−sinθ)

 =2sin(π/3+θ)/2・cos(π/3−θ)/2・2cos(π/3+θ)/2・sin(π/3−θ)/2

 =2sin(π/3+θ)/2・cos(π/3+θ)/2・2sin(π/3−θ)/2・cos(π/3−θ)/2

 =sin(π/3+θ)・sin(π/3−θ)

 =sin(π/3+θ)・sin(φ+τ)

なので、 PB=4sinτ・sinθsin(π/3+θ)

 同様に、△RABにおいて、正弦定理より、

 RB/sinθ=AB/sin(π−(θ+φ))=sin3τ/sin(θ+φ)

なので、 RB=sinθ・sin3τ/sin(θ+φ)

 ここで、 sin3τ=3sinτ−4sin3τ=4sinτ(3/4−sin2τ) で、さらに、

3/4−sin2τ=sin2(π/3)−sin2τ=(sin(π/3)+sinτ)(sin(π/3)−sinτ)

 =2sin(π/3+τ)/2・cos(π/3−τ)/2・2cos(π/3+τ)/2・sin(π/3−τ)/2

 =2sin(π/3+τ)/2・cos(π/3+τ)/2・2sin(π/3−τ)/2・cos(π/3−τ)/2

 =sin(π/3+τ)・sin(π/3−τ)

 =sin(π/3+τ)・sin(θ+φ)

なので、 RB=4sinθ・sinτsin(π/3+τ)

 以上から、△BPRにおいて、余弦定理より、

RP2=16sin2τ・sin2θsin2(π/3+θ)+16sin2θ・sin2τsin2(π/3+τ)

   −32sin2τ・sin2θsin(π/3+θ)sin(π/3+τ)・cosφ

 ここで、 (π/3+θ)+(π/3+τ)+φ=π なので、 π/3+θ、π/3+τ、φ の
3つの角を持つ三角形を考え、その外接円の半径をLとおくと、余弦定理から、

 4L2sin2φ

=4L2sin2(π/3+θ)+4L2sin2(π/3+τ)−8L2sin(π/3+θ)sin(π/3+τ)cosφ

 すなわち、

 sin2φ

=sin2(π/3+θ)+sin2(π/3+τ)−2sin(π/3+θ)sin(π/3+τ)cosφ

より、

 sin(π/3+θ)sin(π/3+τ)cosφ

=(sin2(π/3+θ)+sin2(π/3+τ)−sin2φ)/2

なので、

RP2=16sin2τ・sin2θsin2(π/3+θ)+16sin2θ・sin2τsin2(π/3+τ)

   −16sin2τ・sin2θ(sin2(π/3+θ)+sin2(π/3+τ)−sin2φ)

  =16sin2θsin2φsin2τ

 よって、 RP=4sinθsinφsinτ となる。

 同様にして、 PQ=4sinθsinφsinτ 、 QR=4sinθsinφsinτ となるので、

 PQ=QR=RP すなわち、 △PQRは正三角形である。


(コメント) 三角関数による証明も残り2/3を端折ってしまいましたが、かなりハードですね。
      でも、与えられた三角形の内角の大きさが分かれば、上記の結果を用いて、内部
      に出来る正三角形の1辺の長さが求まる(ただし、元の△ABCの外接円の半径は
      1/2であることに注意!)ということで、この計算も捨てがたいです。


例 外接円の半径が1/2の正三角形の1辺の長さは、/2 である。

 よって、この正三角形の内部に出来る正三角形の1辺の長さは、4sin3(π/9)=0.16

 そこで、元の正三角形の1辺の長さを 3 とすると、相似拡大で、内部に出来る正三角形
の1辺の長さは、 4sin3(π/9)×2=0.5544 となる。かなり小さい三角形ですね!


 次のパスカルの定理も平面幾何ではよく知られた定理だろう。

 円周上の6点A、B、C、D、E、Fについて、

左図のように直線を引き、その交点をP、Q、R

とすると、3点P、Q、Rは一直線上にある。



 方べきの定理やメネラウスの定理を活用した証明は、「パスカルの定理」でなされた。

 ここでは、複素数による証明に挑戦しよう。そのためにいくつか準備しておこう。

 正六角形ABCDEFにおいて、ABとDE、BCとEF、CDとFAが対辺といわれる所以は、下
図を見れば了解されるだろう。

    

 この「対辺」という言葉を用いれば、パスカルの定理は次のように表現できる。

パスカルの定理

 円に内接する六角形ABCDEFの3組の対辺が交わるとし、その交点をP、Q、Rと
おく。

 このとき、3点P、Q、Rは一直線上にある。



 パスカルの定理の証明において、この円の半径は「1」としても一般性を失うことはない。


単位円の割線

 単位円上の2点α、βを通る直線の方程式は、 z+αβ=α+β で与えられる。

実際に、(z−α)/(β−α)が実数 より、

  (z−α)/(β−α)=()/()=−αβ(−1/α)/(β−α)

 すなわち、 z−α=−αβ(−1/α)=−αβ+β より、

 z+αβ=α+β が成り立つ。


(パスカルの定理の証明)

   

 単位円において、A(a)、B(b)、C(c)、D(d)、E(e)、F(f) とし、対辺の交点を
P(α)、Q(β)、R(γ) とする。

 このとき、直線ABは、 z+ab=a+b で、直線DEは、 z+de=d+e

この2式より、zを消去して、 (ab−de)=a+b−d−e

 よって、 (ab−de)=a+b−d−e より、 =(a+b−d−e)/(ab−de)

同様にして、 =(b+c−e−f)/(bc−ef) 、=(c+d−f−a)/(cd−fa) が成り立つ。

 このとき、



=(a+b−d−e)/(ab−de)−(c+d−f−a)/(cd−fa)

=(a−d)(ab+cd+ef−bc−de−fa)/{(ab−de)(cd−fa)}



=(b+c−e−f)/(bc−ef)−(c+d−f−a)/(cd−fa)

=(c−f)(ab+cd+ef−bc−de−fa)/{(bc−ef)(cd−fa)}

なので、

 ()/()={(a−d)(bc−ef)}/{(c−f)(ab−de)}

となる。ここで、

 (α−γ)/(β−γ)

={(1/a−1/d)(1/(bc)−1/(ef))}/{(1/c−1/f)(1/(ab)−1/(de))}

=(d−a)(ef−bc)/{(f−c)/(de−ab)}

={(a−d)(bc−ef)}/{(c−f)(ab−de)}

より、 (α−γ)/(β−γ)=()/() が成り立つ。

 よって、 (α−γ)/(β−γ)は実数 となり、3点P、Q、Rは一直線上にある。  (終)


 パスカルの定理において、AとB、CとD、EとF がそれぞれ一致した場合を考えると、直線
AB、CD、EFは何れも円の接線となる。

 このとき、次の定理が成り立つ。

定理  円に内接する三角形の各頂点における円の接線が対辺と交わる点は、一直
    線上にある。

     

 証明はパスカルの定理から自明だろうが、ここでは、メネラウスの定理を用いて証明しよう。

(証明) 高さが共通で、面積比は底辺の比より、 △APB/△APC=PB/PC

 また、△APB∽△CPA で、相似比は、AB : CA から、 PB/PC=AB2/CA2 となる。

同様にして、 QC/QA=BC2/AB2 、RA/RB=CA2/BC2 となる。

このとき、 (PB/PC)(QC/QA)(RA/RB)=(AB2/CA2)(BC2/AB2)(CA2/BC2)=1

すなわち、△ABCにおいて、 (BP/PC)(CQ/QA)(AR/RB)=1 が成り立つので、

メネラウスの定理の逆より、 3点P、Q、Rは一直線上にある。  (証終)


一般の垂線の足

 これまでに、原点Oが中心で半径1の円周上に2点A(α)、B(β)があるとき、点P(p)よ
り、直線ABに垂線を下した足H(h)は、

   h=(α+β+p−αβ)/2

であることを示してある。(→ 垂線の足

     

 では、一般の場合の垂線の足は、どう表現されるのだろうか。

 そのために、まず、原点Oと点A(α)を結ぶ線分に、点P(p)より下した垂線の足H(h)を
求めることを考える。

    

 題意より、HPOA から、(p−h)/α は純虚数である。 h=kα (kは実数) として、

すなわち、 (p−kα)/α+(−k)/=0 が成り立つ。

両辺を α 倍して、 p−kαα−kα=0 から、 2kα=pα

すなわち、 h=(pα)/(2)=p/2+α/(2) と書ける。


 この公式を用いて、直線OAに関して点Pと対称な点Qを求めることも容易だろう。

    

 h=(pα)/(2)=p/2+α/(2) で、 h=(p+q)/2 すなわち、

  q=2h−p=p+α/−p=α/

と書ける。


 次のように考えれば、もっと直接的に、公式:「 q=α/ 」を導くことが出来る。

 複素数αに対して、複素数α/|α|は、複素数αと同じ向きの大きさ1の複素数である。

 α/|α|=cosθ+i・sinθ とおけば、 /|α|=cos(−θ)+i・sin(−θ)

そこで、 p・/|α| と q・/|α| は、実軸に関して対称となるので、

 q・/|α|=・α/|α| すなわち、 q・・α となる。

したがって、 q=α/ と書ける。



 上記の公式を用いて、一般の場合の垂線の足を求めることが出来る。

    

 ガウス平面内で、−αの平行移動を行って、

 h−α=((p−α)()+()(β−α)/(2())

    =(p−α)/2+()(β−α)/(2())

より、

 h=(p+α)/2+()(β−α)/(2())

 =(p−p+α−αβ−α−β+α)(2())

 =(p−p+αβ−α−β)(2())

 ={(β−α)+αβ}/{2()}+p/2

と書ける。


問題  座標平面上の点(−1,5)から、原点Oと点A(4,6)に下ろした垂線の足の座標
    を求めよ。

(解) (複素数を用いる場合)

 p=−1+5i 、α=4+6i なので、垂線の足H(h)は、

 h=p/2+α/(2

 =(−1+5i)/2+(−1−5i)(4+6i)/(8−12i)

 =−1/2+(5/2)i+(−1−5i)(2+3i)/(4−6i)

 =−1/2+(5/2)i+(13−13i)/(4−6i)

 =−1/2+(5/2)i+(65+13i)/26

 =(52+78i)/26=2+3i

 よって、求める垂線の足の座標は、 (2,3)

(座標幾何を用いる場合)

 直線OAの傾きは、3/2 なので、点Pを通る垂線の方程式は、 y=(−2/3)+13/3

 よって、 (3/2)x=(−2/3)+13/3 より、 (13/6)x=13/3 なので、 x=2

 このとき、 y=3 なので、求める垂線の足の座標は、 (2,3)  (終)


 ガウス平面内の2点A(α)、B(β)があるとき、点P(p)より、直線ABに垂線を下した足
H(h)は、直接的に、次のようにしても求められる。

    

 PHAB より、 (h−p)/(β−α)+()/()=0

 AHAB は平行より、 (h−α)/(β−α)−()/()=0

2式を辺々加えて、 (2h−p−α)/(β−α)+()/()=0

 よって、 

h={(p+α)()−(β−α)()}/{2()}

 ={p()+(β−α)+αβ}/{2()}

 ={(β−α)+αβ}/{2()}+p/2

 これは先の結果と一致する。


 上記の公式は、煩雑過ぎて覚えにくいと思われるので、もっと簡素化させよう。

 ガウス平面上の異なる2点A(α)、B(β)を通る直線Lがある。原点Oから直線Lに
下した垂線の足をH(h)とするとき、hは、


 
h=(αβ)/(2())


  

 公式で、p=0 とすれば、直ちに上記の式は得られるが、直接求めてみよう。

(証明) OHAB より、h/(β−α)は純虚数なので、 h/(β−α)+/()=0

 すなわち、 h()+(β−α)=0 ・・・ (*)

 AHAB は平行より、 (h−α)/(β−α)−()/()=0

 すなわち、 (h−α)()−()(β−α)=0 より、

 h()−α()−(β−α)+(β−α)=0 ・・・ (**)

(*)+(**) より、 2h()−α()+(β−α)=0

 すなわち、2h()=αβ より、h=(αβ)/(2())  (証終)


(コメント) こちらの方がスッキリしていますね!


一般の2点を通る直線

 これまで、ガウス平面において、2点 α、β を通る直線上の点 z は、

  z=mα+nβ (m+n=1)

と表されることを学んだが、パラメータ m、n が少し煩わしい。パラメータを含まない形式も
見ておこう。

 平行条件から、 (z−α)/(β−α)−()/()=0 と書ける。

 分母を払って整理すると、 z()−(α−β)+αβ=0

 上記の式は、行列式を用いて表現した方が美しいだろう。

   


(追記) 令和5年9月7日付け

 2点α、βを通る直線の方程式 z()−(α−β)+αβ=0 において、

α、βが単位円周上にあるとき、α=1 、β=1 なので、上式の両辺にαβを掛けて、

  z+αβ−α−β=0

と簡潔に記述することが出来る。このことを用いて、次の問題を考えてみよう。

問題  ガウス平面上の単位円の周を18等分する。λ=e^iθ (θ=π/9) とすると、

 A(λ5)、B(λ7)、C(λ10)、D(λ14)、E(λ15)、F(λ17) である。ただし、λ18=1

  

 このとき、3つの線分AD、BE、CF は1点Pで交わることを示せ。

(参考) お茶の時間 クイズ&パズル 「角の大きさ(2)」のラングレーの図形

(解) 直線ADは、 z+λ−λ5−λ14=0

 直線BEは、 z+λ4−λ7−λ15=0

 直線CFは、 z+λ9−λ10−λ17=0 である。このとき、

λ18=1 、λ9=−1 、λ6−λ3+1=0 に注意して、

 

=−λ5+λ3+λ8−λ7+λ2−1+λ7−λ6

=λ8−λ6−λ5+λ3+λ2−1=λ2(λ6−λ3+1)−(λ6−λ3+1)=0

 よって、3つの線分AD、BE、CF は1点Pで交わる。  (終)



法線に垂直な直線

   

 点A(α)、P(p)とすると、 OP=p で、 (z−α)/(pi) は実数なので、

  (z−α)/(pi)=()/(−i)

が成り立つ。すなわち、

  z/p+/=α/p+/

両辺に、pを掛けて、 z+p=αp となる。

 ここで、αp は実数なので、αp=k とおくと、

法線に垂直な直線の方程式は、 z+p=k (kは実数) と書けることが分かる。

 特に、k≠0 のとき、 z+p=k (kは実数) は、原点を通らない直線の方程式を
表す。

 そこで、α=p/k とおけば、原点を通らない直線の方程式は、一般に、

  z+α=1

と書ける。


 原点を通らない直線の方程式は、z+α=1 と書けることを上記で示したが、次のよ
うにしても求められる。

 原点を通らない直線の方程式は、xy座標系において、

  ax+by+c=0 (a、b、c は実数で、a、b のどちらかは0でなく、c≠0)

と書ける。 z=x+i・y とおくと、=x−i・y で、 x=(z+)/2 、y=(z−)/(2i)

 ax+by+c=0 に代入して、 a(z+)/2+b(z−)/(2i)+c=0

すなわち、 (ai+b)z+(ai−b)=−2ic

そこで、 α=(−a−bi)/(2c) とおくと、 z+α=1 と書ける。


問題 点 2+i を通り、OP=−1+2i に垂直な直線の方程式を求めよ。

(解) 求める直線の方程式は、

  z/(−1+2i )+/(−1−2i)=(2+i)/(−1+2i )+(2−i)/(−1−2i)

両辺に (−1+2i )(−1−2i)=5 を掛けて、

 (−1−2i)z+(−1+2i )=(−1−2i)(2+i)+(−1+2i )(2−i)=0

  このとき、(−1−2i)z は純虚数となるので、 (−1−2i)z=ki (kは実数) とおける。

 すなわち、 z=ki/(−1−2i)=k(−2−i)/5

 ここで、 z=x+yi とおけば、 x=−2k/5 、y=−k/5 から、kを消去して、

 y=(1/2)x となる。  (終)


(別解) 求める直線の方程式は、 (−1−2i)z+(−1+2i )=k と置ける。

 点 2+i を通るので、 k=(−1−2i)(2+i )+(−1+2i )(2−i )=0

 よって、求める直線の方程式は、 (−1−2i)z+(−1+2i )=0  (終)


# 上記の計算で、z=x+i・y とおくと、 y=(1/2)x が得られる。


線分の垂直2等分線

 A(α)とする。原点Oと点Aを結ぶ線分の垂直2等分線は、α/2を通り、OA=αに垂直
なので、z/p+/=α/p+/ において、pにα 、αにα/2 を代入して、

  z/α+/=1/2+1/2=1 すなわち、 z+α=α

を得る。

 また、この式は、 |z|=|z−α|からも得られる。

 実際に、両辺を平方して、 z=(z−α)()=zz−α+α

これより、z+α=α となるので、両辺をα で割ると、z/α+/=1 となる。


問題 点A(1+i) のとき、線分OAの垂直2等分線を求めよ。

(解) 求める直線は、 z/(1+i)+/(1−i)=1

 両辺に (1+i)(1−i)=2 を掛けて、 (1−i)z+(1+i)=2  (終)


 直線 (1−i)z+(1+i)=2 と実軸との交点は、zが実数なので、 z=

 よって、 2z=2 より、 z=1 である。

 直線 (1−i)z+(1+i)=2 と虚軸との交点は、zが純虚数なので、 z+=0

 よって、−2i・z=2 より、 z=−1/i=i である。


 上記の計算を一般化すると、

 直線 z/α+/=1 と実軸との交点は、

 (1/α+1/)z=1 より、 z=α/(α+

 直線 z/α+/=1 と虚軸との交点は、

 (1/α−1/)z=1 より、 z=α/(−α)


 上記では、原点Oと点Aを結ぶ線分の垂直2等分線を求めたが、一般に、

 線分αβの垂直2等分線を求めると、

  ()z+(β−α)=β−α

である。

 実際に、求める直線は、β−α に垂直な直線なので、

  ()z+(β−α)=k (kは実数)

とおける。この直線が、(β+α)/2 を通るので、 k=β−α と定まる。

 よって、 ()z+(β−α)=β−α となる。


(別解) −αだけ平行移動すると、原点と点β−αを結ぶ線分の垂直2等分線は、

   ()z+(β−α)=(β−α)(

 αだけ平行移動すると、

 ()(z−α)+(β−α)()=(β−α)(

よって、

 ()z+(β−α)

=(β−α)()+α()+(β−α)=β−α

より、 ()z+(β−α)=β−α が得られる。


ヘッセの標準形

 ガウス平面内の直線に、原点Oから垂線を下ろし、その足をHとおく。

      

 OH=r とし、H(α)とおくとき、直線の方程式は、2点O、A(2α)を結ぶ線分OAの垂直
2等分線なので、

   z/(2α)+/(2)=1

で与えられる。すなわち、 z+α=2α=2r2 と書ける。

両辺をで割ると、 z+(α/=2r2/ すなわち、 z+(α2/r2=2α

 そこで、α方向の大きさ1の複素数 t=α/r を考えると、

  z+t2=(2r)t ・・・ r は原点と直線との距離

と書けることが分かる。

 この形の直線の方程式は、ヘッセの標準形と言われる。


 極形式で、 α=r(cosθ+i・sinθ) と表されるとき、z=x+i・y とおいて、

 r(cosθ−i・sinθ)(x+i・y )+r(cosθ+i・sinθ)(x−i・y )=2r2

両辺を r で割り、展開して、

 xcosθ+i・ycosθ−i・xsinθ+ysinθ+xcosθ−i・ycosθ+i・xsinθ+ysinθ=2r

すなわち、 xcosθ+ysinθ=r となる。

 この形は、直交座標系における、よく見慣れたヘッセの標準形である。


問題  2直線 z+α=k ・・・ (*) 、αz+=k ・・・ (**) の位置関係を調べよ。

(解) (*)と実軸との交点は、 z=k/(α+) である。

 (**)と実軸との交点も、 z=k/(α+) である。

よって、(*)と(**)は、実軸上の点 k/(α+) で交わる。

 (*)に垂直な法線は で、(**)に垂直な法線は α なので、2直線(*)と(**)は、実軸に関し
て線対称である。


(コメント) (*)の z に を代入したものが(**)なので、当然の帰結ですね!


 上記では、α+≠0 と仮定して交点を求めたわけであるが、α+=0 すなわち、
αが純虚数のときはどうなのだろう。

 このとき、2直線は実軸に関して対称で、かつ、実軸に平行となる。

 直線 z+α=k は、−αz+α=k となるが、虚軸との交点を求めるには、

=−z を代入して、 −2αz=k より、 z=−k/(2α)

 直線 αz+=k は、αz−α=k となるが、虚軸との交点を求めるには、

=−z を代入して、 2αz=k より、 z=k/(2α)

となる。


 ヘッセの標準形を利用して、直線に関する対称移動を考えよう。

 直線の方程式は、 z+t2=(2r)t と書ける。この直線に関して、点 z と対称な点を w
とする。

   

 このとき、 (w−z)/t は実数なので、 (w−z)/t=()/ が成り立つ。

すなわち、 w−z=(t/)()=t2) ・・・ (*)

 また、(z+w)/2 は、直線上の点なので、

 (z+w)/2+t2)/2=(2r)t  即ち、 z+w+t2)=(4r)t ・・・ (**)

(*)(**)から を消去して、 w=−t2+(2r)t


(コメント) 直線の方程式 z+t2=(2r)t と対称点の座標 w=−t2+(2r)t を比較
      すると、美しい関係ですね!


 ある直線に関する対称移動を2回繰り返すと、元に戻る。これは自明かもしれない。

 実際に、 w=−t2+(2r)t から、

 −t2+(2r)t

 =−t2(−2z+(2r))+(2r)t=t22z−(2r)t2+(2r)t=z (t=1に注意)

より、元に戻ることが示された。


 直線L:z+t2=(2r)t に関して対称移動し、さらに、直線M:z+u2=(2r’)u に

 関して対称移動させる。ただし、t、u は、法線方向の複素数で、大きさは1とする。

 z → w → p とするとき、z と p はどのような関係にあるだろうか?

 上記の結果を用いれば、 w=−t2+(2r)t 、p=−u2+(2r’)u と書けるので、

 p=−u2(−2z+(2r))+(2r’)u=u22z−(2r)u2+(2r’)u

ここで、 u22=1 すなわち、 u2=t2 から、 u=±t なので、2直線L、Mは平行とな

り、z → p は平行移動を表す。

 u22≠1 すなわち、 u2≠t2 から、 u≠±t で、arg(u22)=2(arg(u)−arg(t))で、

arg(u)−arg(t)は、2直線L、Mのなす角に等しいから、z → p は、回転角 arg(u22

の回転移動を表す。回転移動の中心は、2直線L、Mの交点である。


2直線の交角

 直線 z+α=k (ただし、α≠0 で、kは実数) は、複素数 α に垂直である。

 したがって、

 2直線 z+α=k 、z+β=h (ただし、α≠0 、β≠0 で、k、hは実数)

のなす角は、 arg(β/α) により求められる。

 このとき、2直線 z+α=k 、z+β=h について、

 平行であるための必要十分条件は、β/α が実数

 垂直であるための必要十分条件は、β/α が純虚数

である。


2直線の交点

 2直線 z+α=k 、z+β=h (ただし、α≠0 、β≠0 で、k、hは実数)の
交点を求める。

 2式から を消去して、 (β−α)z=βk−αh から、

 β−α≠0 のとき、 z=(βk−αh)/(β−α

となる。

 β−α=0 のときは、 β/α=/ から、 β/αは実数 と
なり、2直線は平行で、βk−αh≠0 のときは、2直線は交わらない。

 2直線は平行で、βk−αh=0 のときは、2直線は重なり、交点は直線上の無数の点と
なる。


三角形の垂心

 △ABCの3つの頂点からの垂線は、1点で交わる。

#この交点は垂心と言われる。

(証明) 一般性を失うことなく、A(α)、B(β)、C(0)としてよい。

 Aより辺BCに下ろした垂線の方程式は、 (z−α)/β+()/=0

  すなわち、 z+β=αβ ・・・ (*)

 Bより辺CAに下ろした垂線の方程式は、 (z−β)/α+()/=0

  すなわち、 z+α=αβ ・・・ (**)

 Cより辺ABに下ろした垂線の方程式は、 z/(β−α)+/()=0

  すなわち、 ()z+(β−α)=0 ・・・ (***)

(*)×α−(**)×β より、 (αβ)z=(α−β)(αβ)

 αβ≠0 なので、 z=(α−β)(αβ)/(αβ)

 このとき、 =−()(αβ)/(αβ) なので、(***)に代入して、

 ()z+(β−α)

=()(α−β)(αβ)/(αβ)
  −(β−α)()(αβ)/(αβ)

=0

が成り立つので、(*)と(**)の交点は、(***)の上にある。

 したがって、△ABCの3つの頂点からの垂線は、1点で交わる。  (証終)


 上記では、「△ABCの3つの頂点からの垂線は、1点で交わる」ことを示す過程で、具体
的な垂心の位置は、

  z=(α−β)(αβ)/(αβ)

であることが求められているが、若干、式が汚い。

 △ABCの外心を原点に選び、その外接円を単位円とすれば、具体的な垂心の位置を綺麗
に表すことができる。そうでもしないと、垂心の位置を覚えようという気は起きない。

 A(α)、B(β)、C(γ)とし、垂心をH(h)とするとき、直線AHの方程式は、

 (z−α)/(β−γ)+()/()=0

すなわち、 (z−α)/(β−γ)−βγ(α−1)/{α(β−γ)}=0 より、

 α(z−α)−βγ(α−1)=0 から、 αz−αβγ−α2+βγ=0

同様に、直線BHの方程式は、 βz−αβγ−β2+γα=0

これらの2式からを消去して、

 (α−β)z=α2−β2+γ(α−β)=(α−β)(α+β+γ)

α≠β なので、 z=α+β+γ


(コメント) 断然、「z=α+β+γ」の方が美しい!

 △ABCの重心Gは、複素数 (α+β+γ)/3 で表されるので、OH=3OG が成り立つ。

よって、

 3点O、G、Hは一直線上にあり、Gは線分OHを1:2に内分する

ことも分かる。この3点O、G、Hを通る直線は、オイラー線と言われる。


問題  同一円周上に4点A、B、C、Dがある。4点から3点を選んで三角形を作る。このと
    き出来る4つの三角形の垂心は、同一円周上にあることを示せ。

    

(解) 4点A(α)、B(β)、C(γ)、D(δ)は、単位円周上の点としても一般性は失われない。

 △ABCの垂心H1は、 α+β+γ で表される。

同様に、△ABDの垂心H2は、 α+β+δ で、△ACDの垂心H3は、 α+γ+δ で、

△BCDの垂心H4は、 β+γ+δ で表される。

 このとき、点O’(z)について、 z=α+β+γ+δ とおくと、

  O’H1=|δ|、O’H2=|γ|、O’H3=|β|、O’H4=|α|

で、4点A、B、C、Dは同一円周上にあるので、 O’H1=O’H2=O’H3=O’H4 が成り

立つ。

 よって、4つの垂心H1、H2、H3、H4は、点O’を中心とする同一円周上にある。


(コメント) 当初の円が平行移動した感じなんですね!美しい結果です。


(追記) 令和6年1月1日付け

問題  ガウス平面上の原点以外の定点Aと、単位円O上の動点B、Cがあり、線分BCは直
   径とする。このとき、△ABCの垂心の軌跡を求めよ。

  

(解) A(α)、B(t)、C(−t)とおき、垂心をP(z)とおく。

 Bから直線ACに引いた垂線の足をD、Cから直線ABに引いた垂線の足をEとおく。

直線BDの方程式は、(z−t)/(α+t)が純虚数であることから、

 (z−t)/(α+t)+()/()=0 すなわち、

 ()(z−t)+(α+t)()=0 より、()z+(α+t)=tα+2

 両辺に t を掛けて、 (1+t)z+(t2+tα)=t2+α+2t

同様にして、直線CEの方程式は、

 (1−t)z+(t2−tα)=t2+α−2t

2式を辺々引いて、両辺を2t で割って、 z+α=2

 これが、求める垂心の軌跡の方程式である。  (終)

(補足) z=x+i・y 、α=a+i・b とおくと、ax+by=1 と書ける。

 このことから、垂心の軌跡の方程式は、線分OAに垂直な直線であることが分かる。


(追記) 令和6年1月3日付け

問題  ガウス平面において、単位円O上に3点A(α)、B(β)、C(γ)がある。Aは動点
   で、B、Cは定点である。このとき、△ABCの垂心H(z)の軌跡を求めよ。

  

(解) △ABCの垂心H(z)は、 z=α+β+γ で表される。このとき、

 z−(β+γ)=α で、|α|=1 より、 |z−(β+γ)|=1

 よって、求める軌跡は、β+γを中心とし、半径1の円である。  (終)



2直線の交点を通る直線

 交わる2直線 z+α=k 、z+β=h (ただし、k、hは実数) の交点を通る直
線は、同時に0ではない実数 m、n を用いて、

  m(z+α−k)+n(z+β−h)=0

と書ける。この公式は、座標幾何では定番のものだろう。

問題  2直線 (1−i)z+(1+i)+2=0 、(2+i)z+(2−i)+1=0 の交点を通る
    直線で、1 を通るものを求めよ。

(解) 2直線を連立して、

{(2−i)(1−i)−(2+i)(1+i)}z+{(2−i)(1+i)−(2−i)(1+i)}+2(2−i)−(1+i)=0

すなわち、 −6i・z=−3+3i より、 z=−(1+i)/2 が交点

 −(1+i)/2 と 1 を通る直線は、 (z−1)/(3+i)=(−1)/(3−i)

よって、 (3−i)(z−1)=(3+i)(−1) より、 (3−i)z−(3+i)+2i=0  (終)


 上記の問題を、公式を用いて解いてみよう。

(別解) 求める直線は、同時に0ではない実数 m、n を用いて、

 m((1−i)z+(1+i)+2)+n((2+i)z+(2−i)+1)=0

と書ける。1を通るから、 4m+5n=0 すなわち、 n=−4m/5

 これを上式に代入して、

 (1−i)z+(1+i)+2+(−4/5)((2+i)z+(2−i)+1)=0

すなわち、 (−3−9i)z+(−3+9i)+6=0 から、(1+3i)z+(1−3i)−2=0

両辺に i を掛けて、 (i−3)z+(i+3)−2i=0 から、

  (3−i)z−(3+i)+2i=0  (終)


3直線が1点で交わる

 3直線 z+α=k 、z+β=h 、z+γ=g (ただし、k、h、gは実数) に
対して、恒等式

 s(z+α−k)+t(z+β−h)+u(z+γ−g)=0

が成り立つような複素数 s、t、u (stu≠0) が存在すれば、

 3直線は1点で交わる または 3直線は平行

である。

 実際に、2直線 z+α=k 、z+β=h の交点を z0 とおくと、

 s(0+α0−k)+t(0+β0−h)+u(0+γ0−g)=0

すなわち、 u(0+γ0−g)=0 より、 u≠0 なので、 0+γ0=g

したがって、3直線は、1点(z0)で交わる。

 また、2直線 z+α=k 、z+β=h が平行なとき、β=bα (bは実数) と書
けるので、

 s(z+α−k)+t(bz+bα−h)+u(z+γ−g)=0

すなわち、 (s+tb)z+(s+tb)α−sk−th+u(z+γ−g)=0

 これは、2直線 z+α=k と z+γ=g が平行であることを示す。

したがって、3直線は平行である。


三角形の外心

問題  △ABCにおいて、3辺の垂直2等分線は1点で交わることを示せ。

#この点は△ABCの外心と言われる。

(解) ガウス平面において、A(α)、B(β)、C(γ)とする。

 線分BCの垂直2等分線は、

 {z−(β+γ)/2}/(β−γ)+{−()/2}/()=0

すなわち、 ()z+(β−γ)−β+γ=0

同様にして、線分CAの垂直2等分線は、 ()z+(γ−α)−γ+α=0

 線分ABの垂直2等分線は、 ()z+(α−β)−α+β=0

 3式を辺々加えると、

1・(()z+(β−γ)−β+γ
 +1・(()z+(γ−α)−γ+α
  +1・(()z+(α−β)−α+β)=0

が恒等的に成り立つので、3直線は1点で交わる。  (終)


 上記では、具体的な外心の座標を計算していなかった。以下で、計算してみた。

 ガウス平面において、A(α)、B(β)、C(γ)とするとき、△ABCの外心は

{α(β−γ)+β(γ−α)+γ(α−β)}/{(β−γ)+(γ−α)+(α−β)}

で与えられる。

 実際に、

線分BCの垂直2等分線は、()z+(β−γ)−β+γ=0 ・・・ (1)

線分CAの垂直2等分線は、 ()z+(γ−α)−γ+α=0 ・・・ (2)

(1)×(γ−α)−(2)×(β−γ) により、 を消去すれば、

{(γ−α)()−(β−γ)()}z+(γ−α)(−β+γ)−(β−γ)(−γ+α)=0

(β−γ)+(γ−α)+(α−β)}z={α(β−γ)+β(γ−α)+γ(α−β)}

 ここで、α、β、γは△ABCの頂点なので、 α≠β≠γ より、

 (β−γ)+(γ−α)+(α−β)≠0 である。

よって、外心は、

{α(β−γ)+β(γ−α)+γ(α−β)}/{(β−γ)+(γ−α)+(α−β)}

となる。


問題  △ABCにおいて、△ABCの外側に、辺BC、CA、ABをそれぞれ1辺とする正三角
    形PBC、QCA、RABを作るとき、3直線AP、BQ、CRは1点で交わることを示せ。

     

(解) ガウス平面において、A(α)、B(β)、C(γ)、P(p)、Q(q)、R(r) とおく。

 ω=(−1+i)/2 とすると、 β−p=ω(γ−β) より、 p=(1+ω)β−ωγ

 1+ω=−ω2 より、 p=−ω2β−ωγ である。

 同様にして、 q=−ω2γ−ωα 、r=−ω2α−ωβ である。

 直線APは、 (z−α)/(α+ω2β+ωγ)−()/(+ω+ω2)=0

  すなわち、

 (+ω+ω2)z−(α+ω2β+ωγ)+ω2β−α)+ω(γ−α)=0

同様にして、直線BQは、

 (+ω+ω2)z−(β+ω2γ+ωα)+ω2γ−β)+ω(α−β)=0

直線RCは、

 (+ω+ω2)z−(γ+ω2α+ωβ)+ω2α−γ)+ω(β−γ)=0

となる。3式を辺々加えると、

 z の係数は、

 (+ω+ω2)+(+ω+ω2)+(+ω+ω2

=(1+ω+ω2)()=0

  の係数は、

 −(α+ω2β+ωγ)−(β+ω2γ+ωα)−(γ+ω2α+ωβ)

=−(1+ω+ω2)(α+β+γ)=0

 定数項は、

 ω2β−α)+ω(γ−α)+ω2γ−β)+ω(α−β
+ω2α−γ)+ω(β−γ)=0

なので、3式の和は0となる。

 よって、3直線AP、BQ、CRは1点で交わる。


(コメント) 解法が定型的になって、円滑に結論が得られますね!これは感動です...。


(追記) 令和5年12月27日付け

問題  ガウス平面上の原点以外の定点Aと、単位円O上の動点B、Cがあり、線分BCは直
   径とする。このとき、△ABCの外心の軌跡を求めよ。

  

(解) A(α)、B(t)、C(−t)とおき、外心をP(z)とおく。題意より、

 |z−t|=|z+t|=|z−α|

が成り立つ。|z−t|2=|z+t|2 より、 z+t=0

|z+t|2=|z−α|2 より、 z+tz+α=α−1

2式を連立して、 z+α=α−1

 これが、求める外心の軌跡の方程式である。  (終)

(補足) z=x+i・y 、α=a+i・b とおくと、

 ax+by=(a2+b2−1)/2 と書ける。

 このことから、外心の軌跡の方程式は、線分OAに垂直な直線であることが分かる。



(追記) 令和4年11月24日付け

 当HPがいつもお世話になっているHN「よおすけ」さんから「4点の共円」問題をいただき
ました。

 今回は、同一円周上の4点の証明です。

問題 ガウス平面において、互いに異なる4点α、β、γ、δが同一円周上にあるとき、

 {(α−γ)/(β−γ)}/{(α−δ)/(β−δ)}は、0でない実数

であることを証明せよ。

(解) arg(α−γ)/(β−γ)=∠βγα 、arg(α−δ)/(β−δ)=∠βδα である。

 2つの場合が考えられる。

(1) 点α、βに対して、点γ、δが同じ側にあるとき、

     

 この場合は、円周角の定理より、 ∠βγα=∠βδα なので、

  arg{(α−γ)/(β−γ)}/{(α−δ)/(β−δ)}=0

 よって、 {(α−γ)/(β−γ)}/{(α−δ)/(β−δ)}は、正の実数

(2) 点α、βに対して、点γ、δが両側にあるとき、

     

 この場合は、角の向きも考慮して、 ∠βγα+∠αδβ=±π である。

すなわち、 ∠βγα−∠βδα=±π より、

 arg{(α−γ)/(β−γ)}/{(α−δ)/(β−δ)}=±π

 よって、 {(α−γ)/(β−γ)}/{(α−δ)/(β−δ)}は、負の実数

以上から、{(α−γ)/(β−γ)}/{(α−δ)/(β−δ)}は、0でない実数  (終)


(コメント) ガウス平面において、互いに異なる4点α、β、γ、δが

 同一円周上 または 同一直線上にあるとき、

 {(α−γ)/(β−γ)}/{(α−δ)/(β−δ)}は、0でない実数

である。逆も成り立つ。

 特に、同一直線上にあるとき、

  ∠βγα=∠βδα=0 または、 ∠βγα、∠βδαの一方が0で、他方がπ

である。


円に内接する四角形

 凸四角形ABCDが円に内接するための必要十分条件は、

   {(α−γ)/(β−γ)}/{(α−δ)/(β−δ)}は、正の実数

である。



 証明は、上記のよおすけさんの問題の解答(1)から明らかだろう。


複比

 複素数の問題では、{(α−γ)/(β−γ)}/{(α−δ)/(β−δ)}が活躍する場面が多
い。

 そこで、{(α−γ)/(β−γ)}/{(α−δ)/(β−δ)} は、複比または非調和比
と言われ、
       (α,β,γ,δ)

と表記される。

 複比の定義式は、一見複雑そうに見えるが、2つの円周角を意識すれば、当然の式にも
見えてくる。

 直線上に、4点A、P、B、Qがこの順に並ぶとき、

 AP/BPは、ABを基準として、Pがいくらに内分しているかを表し、

 AQ/BQは、ABを基準として、Qがいくらに外分しているかを表している。

 この2つの比の比 (AP/BP)/(AQ/BQ) が複比と言われる由縁である。

特に、(AP/BP)/(AQ/BQ)=1 のとき、すなわち、内分比と外分比が等しいとき、

4点A、P、B、Qは、調和点列と言われる。


複比について、次の性質が知られている。

性質1. (α,β,γ,δ)=(β,α,δ,γ)=(γ,δ,α,β)

 実際に、

(α,β,γ,δ)={(α−γ)/(β−γ)}/{(α−δ)/(β−δ)} より、

(β,α,δ,γ)={(β−δ)/(α−δ)}/{(β−γ)/(α−γ)}

          ={(α−γ)/(β−γ)}/{(α−δ)/(β−δ)}=(α,β,γ,δ)

(γ,δ,α,β)={(γ−α)/(δ−α)}/{(γ−β)/(δ−β)}

          ={(α−γ)/(β−γ)}/{(α−δ)/(β−δ)}=(α,β,γ,δ)

より、成り立つ。図解すると、

     、 

は、同値となる。

性質2. (α,β,δ,γ)=1/(α,β,γ,δ)

      (β,α,γ,δ)=1/(α,β,γ,δ)

 実際に、

(α,β,δ,γ)={(α−δ)/(β−δ)}/{(α−γ)/(β−γ)}

          =1÷{(α−γ)/(β−γ)}/{(α−δ)/(β−δ)}=1/(α,β,γ,δ)

(β,α,γ,δ)={(β−γ)/(α−γ)}/{(β−δ)/(α−δ)}

          =1÷{(α−γ)/(β−γ)}/{(α−δ)/(β−δ)}=1/(α,β,γ,δ)

より、成り立つ。図解すると、

     、 

は、逆数となる。

性質3. (α,γ,β,δ)=1−(α,β,γ,δ)

      (δ,β,γ,α)=1−(α,β,γ,δ)

 実際に、

(α,γ,β,δ)={(α−β)/(γ−β)}/{(α−δ)/(γ−δ)}

1−(α,β,γ,δ)

=1−{(α−γ)/(β−γ)}/{(α−δ)/(β−δ)}

=[{(α−δ)/(β−δ)}−{(α−γ)/(β−γ)}]/{(α−δ)/(β−δ)}

={(β−γ)(α−δ)−(β−δ)(α−γ)}/{(α−δ)(β−γ)}

=(α−β)(δ−γ)/{(α−δ)(β−γ)}

={(α−β)/(γ−β)}/{(α−δ)/(γ−δ)}

より、 (α,γ,β,δ)=1−(α,β,γ,δ) が成り立つ。

 同様にして、

(δ,β,γ,α)={(δ−γ)/(β−γ)}/{(δ−α)/(β−α)}

1−(α,β,γ,δ)

=(α−β)(δ−γ)/{(α−δ)(β−γ)}

={(δ−γ)/(β−γ)}/{(δ−α)/(β−α)}

より、 (δ,β,γ,α)=1−(α,β,γ,δ) が成り立つ。

 図解すると、

     、 

との和は1となる。

 読者のために、練習問題を残しておこう。

問題  (1,2,3,4)=x として、次の複比を x を用いて表せ。

(1) (1,2,4,3)

(2) (1,3,2,4)

(3) (1,3,4,2)

(4) (1,4,2,3)

(5) (1,4,3,2)

(解)(1) (1,2,4,3)=1/(1,2,3,4)=1/x

(2) (1,3,2,4)=1−(1,2,4,3)=1−x

(3) (1,3,4,2)=1/(1,3,2,4)=1/(1−x)

(4) (1,4,2,3)=1−(1,2,4,3)=1−1/(1,2,3,4)=1−1/x=(x−1)/x

(5) (1,4,3,2)

   =1−(1,3,4,2)=1−1/(1,3,2,4)=1−1/{1−(1,2,3,4)}

   =1−1/(1−x)=x/(x−1)  (終)

 以上を一般化して、複比の取り得る値は、次の6個あることが分かる。すなわち、

  x 、1−x 、1/x 、x/(x−1) 、(x−1)/x 、1/(1−x)

 特に、x=−1 のとき、複比は、 −1 、2 、1/2

 x=(1+i)/2 のとき、複比は、 x 、x5 (ただし、x6=1) となる。



 この複比と以前述べたトレミーの不等式は密接な関係がある。

 トレミーの不等式

 異なる4点が左回りにA(α)、B(β)、C(γ)、D(δ)とあるとき、

  AB・CD+AD・BC≧AC・BD

が成り立つ。


 上記の証明には、恒等式

  (α−β)(γーδ)+(αーδ)(βーγ)=(αーγ)(βーδ)

が利用された。( → 参考:トレミーの不等式の証明

 等式の両辺を (α−γ)(β−δ) で割ると、

 (α−β)(γーδ)/{(α−γ)(β−δ)}+(αーδ)(βーγ)/{(α−γ)(β−δ)}=1

すなわち、

{(β−α)/(γ−α)}/{(β−δ)/(γ−δ)}+{(β−γ)/(α−γ)}/{(β−δ)/(αーδ)}=1

 この等式を複比で表現すれば、

  (β,γ,α,δ)+(β,α,γ,δ)=1

となる。これは複比の性質3.そのものである。

 四角形ABCDが円に内接するとき、

円に内接する四角形

 凸四角形ABCDが円に内接するための必要十分条件は、

   {(α−γ)/(β−γ)}/{(α−δ)/(β−δ)}は、正の実数

である。


より、複比 (α,β,γ,δ)>0 である。すなわち、

 直線ABに関して、C、Dが同じ側にあれば、 (α,β,γ,δ)>0 で、

  (β,α,γ,δ)=1/(α,β,γ,δ)>0

 同様に、直線BCに関して、A、Dが同じ側にあるので、 (β,γ,α,δ)>0

よって、 

|(β−α)(γ−δ)/{(β−δ)(γ−α)}+|(β−γ)(αーδ)/{(β−δ)(α−γ)}=1

すなわち、

 |β−α||γ−δ|+|β−γ||αーδ|=|β−δ||γ−α|

より、

  AB・CD+AD・BC=AC・BD  (トレミーの定理)

が成り立つ。


九点円

 △ABCにおいて、

 3辺の中点、、各頂点から下ろした垂線の足、垂心と各頂点を結ぶ線分の中点

の合計9点は、同一円周上にある。この円のことを、九点円という。

(→ 参考:「九点円について」)

 九点円の中心は、外心と垂心を結ぶ線分の中点で、九点円の半径は、外接円の半径
の半分


である。


O:外心

G:重心

H:垂心

N:九点円の中心

 4点O,G,N,Hは同一直線上にある。この直線のことを、オイラー線という。線分の比は、

         OG:GN:NH=2:1:3
となる。

 九点円が上記のような性質を何故もつのか?その理由は簡単である。

∠PKD=90°より、点Kは線分PDを直径とする円周上にある。他も同様。このことから、
九点円のもつ性質は、明らかとなる。

 実際に、O(0)、A( α )、B( β )、C( γ )とする。但し、複素数 α 、β 、γ の絶対値
は 1 とする。すなわち、△ABCの外接円は単位円であるものとする。

 このとき、 D(( β + γ )/2 ) 、 P({ α +( α + β + γ )}/2 ) なので、線分DP

の中点を N(n) とすると、 n=( α + β + γ )/2 となる。

 よって、線分DPの中点 N は、外心Oと垂心Hを結ぶ線分の中点である。

 さらに、 PN=(1/2)|α|=1/2 が成り立つので、九点円の半径は、外接円の半径

の半分である。 同様にして、(E、Q、L)や(F、R、M)についても同じことがいえる。

 また、G(( α + β + γ )/3 )、H( α + β + γ ) なので、

    OG : GN : NH = 2 : 1 : 3

が成り立つ。


(コメント) 平面幾何の有名事実も、複素数を使うと、易しく思えてくるから不思議です。


フランケ点

 上記では、三角形の外心O、重心G、垂心H、九点円の中心Nはすべてオイラー線の上に
あることを見た。更に、次の定理が成り立つ。

定理  △ABCの外心Oから辺BC、CA、ABに下ろした垂線の足をD、E、Fとする。
    直線OD、OE、OF上に、それぞれD’、E’、F’を

     OD’=λOD 、OE’=λOE 、OF’=λOF  (λは実数)

 となるようにとる。このとき、3直線AD’、BE’、CF’はオイラー線上で1点で交わる。


    

#3直線AD’、BE’、CF’の交点は、フランケ点と言われる。

 λ=0 のとき、 フランケ点は、外心と一致する。
 λ=1 のとき、 フランケ点は、重心と一致する。
 λ=2 のとき、 フランケ点は、九点円の中心と一致する。
 λ=±∞ のとき、 フランケ点は、垂心と一致する。

(証明) ガウス平面において、O(0)、A( α )、B( β )、C( γ )とする。

 但し、複素数 α 、β 、γ の絶対値は 1 とする。すなわち、△ABCの外接円は単位円
であるものとする。

 D((β+γ)/2) 、E((γ+α)/2) 、F((α+β)/2)

である。このとき、

 D’(μ(β+γ)) 、E’(μ(γ+α)) 、F(μ(α+β))  (ただし、μ=λ/2 とおく)

直線AD’の方程式は、

 (z−α)/(μ(β+γ)−α)=()/(μ()−

すなわち、

(μα(β+γ)−βγ)z−αβγ(μ(β+γ)−α)−μα2(β+γ)+μβγ(β+γ)=0

 ここで、基本対称式 s=α+β+γ 、t=αβ+βγ+γα 、u=αβγ とおくと、

 β+γ=s−α 、 βγ=t−α(s−α)=t−sα+α2 なので、

直線AD’の方程式は、次のように書き変えられる。

(μt−(μ+1)u/α)z−u(μs−(μ+1)α)−μ(tα−u)+μu(s−α)/α=0

すなわち、

(μtα−(μ+1)u)z−u(μsα−(μ+1)α2−μ(tα2−uα)+μu(s−α)=0

αについて、降べきの順に整理して、

α2(u(μ+1)−μt)+α(μtz−uμs)−(μ+1)uz+μus=0

f(α)=α2(u(μ+1)−μt)+α(μtz−uμs)−(μ+1)uz+μus とおくと、

直線AD’の方程式は、f(α)=0 と表せる。

 α2、α、1 の各係数は、α、β、γの対称式となっているので、

直線BE’の方程式は、f(β)=0 と表せる。

同様にして、直線CF’の方程式は、f(γ)=0 と表せる。

 f(α)=0 は2次方程式なので、3つの解α、β、γを持つためには、

 u(μ+1)−μt=0 、μtz−uμs=0 、−(μ+1)uz+μus=0

でなければならない。

 このとき、 −(μ+1)uz+μus=0 から、 z=μs/(μ+1) が交点となる。

 z=μs/(μ+1) から、 =μt/(u(μ+1)) なので、

u(μ+1)−μt=μt−μt=0

μtz−uμs=μ2ts/(μ+1)−uμ2st/(u(μ+1))=μ2ts/(μ+1)−μ2st/(μ+1)=0

よって、 z=μs/(μ+1) は、第1式、第2式をともに満たす。  (証終)


三角形の内心

 ガウス平面において、A( α )、B( β )、C( γ )とし、△ABCの外心Oは、原点とする。

 但し、複素数 α 、β 、γ の絶対値は 1 とする。すなわち、△ABCの外接円は単位円
であるものとする。

 九点円の話で、重心Gは、OG=(α+β+γ)/3 であり、垂心Hは、OH=α+β+γ
と簡単に表せた。同様に、内心も簡単に表すことを考える。

 単位円周上の2点A(α2)、B(β2) (ただし、0≦argα<argβ<π) とすると、

劣弧ABの中点M(αβ)、優弧ABの中点M’(−αβ) と表せる。

  

 以上の準備のもとに、単位円周上の3点A(α2)、B(β2)、C(γ2) をとる。
(ただし、0≦argα、argβ<π、π≦argγ<2πとする。)

  

 このとき、弧BC、CA、ABの中点をそれぞれD、E、Fとおくと、

 D(−βγ) 、E(−γα) 、F(−αβ)

となる。∠Aの2等分線ADの方程式は、

 (z+βγ)/(α2+βγ)=()/(2)=α2(βγ+1)/(α2+βγ)

すなわち、 z+βγ=α2(βγ+1) ・・・ (1)

同様にして、∠Bの2等分線Eの方程式は、 z+γα=β2(γα+1) ・・・ (2)

(1)×β−(2)×α より、 (β−α)z+γ(β2−α2)=αβ(α−β)

α≠β より、 z+γ(α+β)=−αβ なので、 z=−αβ−βγ−γα

 これが、内心の座標を表す。


三角形の傍心

 内心と同様に、傍心も求められる。

 D、E、Fを通る直径の他端をそれぞれD’、E’、F’とすると、偏角の関係から、

 D’(βγ) 、E’(γα) 、F’(αβ)

である。

   

 このとき、直線BE’は、∠Bの外角の2等分線である。

実際に、 四角形ABE’Eは円に内接するので、 ∠GBE’=∠AEE’

 Eは弧ACの中点なので、 ∠AEE’=∠CEE’

 円周角の定理より、 ∠CEE’=∠CBE’

よって、∠GBE’=∠CBE’ となるので、直線BE’は、∠Bの外角の2等分線である。

 同様にして、直線CF’は、∠Cの外角の2等分線である。

 したがって、∠Aの2等分線AD、∠Bの外角の2等分線BE’、∠Cの外角の2等分線CF’
の交点が傍心Jとなる。

 内心の計算で用いた、 D(−βγ) 、E(−γα) 、F(−αβ) と

∠A内の傍心の計算で用いる、 D(−βγ) 、E’(γα) 、F’(αβ)

を比較して、∠A内の傍心の計算は、 z=−αβ−βγ−γα で、αの代わりに、−α

を代入すればよいことが分かる。

 以上から、∠A内の傍心Jの座標は、 z=αβ−βγ+γα

同様にして、∠B内の傍心J’の座標は、 z=αβ+βγ−γα

∠C内の傍心J”の座標は、 z=−αβ+βγ+γα

となる。


(コメント) 結果が奇麗ですね!三角形の5心までの距離については、「内接円と傍接円
      を参照。


 △ABCの内接円、傍接円、九点円について、次の定理が成り立つことが知られている。

フォイエルバッハの定理

 △ABCの内接円は九点円に内接し、傍接円は九点円に外接する。

      

 証明は難しい...。(参考 → 初等幾何的証明

 複素数による証明は、今後の研究課題としよう。


擬似重心

 さらに、「複素数の底力」の探究を続けよう。次の定理が知られている。

定理  △ABCの外接円Oに各頂点で接線を引き、△DEFを作る。
    このとき、3直線AD、BE、CFは1点Gで交わる。

      

(証明) ガウス平面において、O(0)、A( α )、B( β )、C( γ )とする。

 但し、複素数 α 、β 、γ の絶対値は 1 とする。すなわち、△ABCの外接円は単位円
であるものとする。

接線BDの方程式は、 (z−β)/β+()/=0

 すなわち、 (z−β)/β+β−1=0 より、 z+β2−2β=0 ・・・ (1)

同様に、接線CDの方程式は、 z+γ2−2γ=0 ・・・ (2)

 (1)×γ2−(2)×β2 より、 (γ2−β2)z−2βγ(γ−β)=0

 γ≠β なので、 z=2βγ/(β+γ) が交点Dの座標である。

このとき、直線ADの方程式は、

 (z−α)/(2βγ/(β+γ)−α)=()/(2/()−

すなわち、 (2α−β−γ)z−α(2βγ−αβ−γα)−2α2+2βγ=0

 ここで、s=α+β+γ、t=αβ+βγ+γα、u=αβγ とおくと、

直線ADの方程式は、 (3α−s)z−α(3u/α−t)−2α2+2u/α=0

すなわち、 α(3α−s)z−α(3u−αt)−2α3+2u=0

 この式をαの降べきの順に整理して、

 2α3−(3z+t)α2+(sz+3u)α−2u=0

f(α)=2α3−(3z+t)α2+(sz+3u)α−2u とおくと、

直線ADの方程式は、f(α)=0 と表せる。

 α3、α2、α、1 の各係数は、α、β、γの対称式となっているので、

直線BEの方程式は、f(β)=0 と表せる。

同様にして、直線CFの方程式は、f(γ)=0 と表せる。

 解と係数の関係より、

  3z+t=2s 、 sz+3u=2t

が成り立つ。これより、を消去して、 

 z=(6us−2t2)/(9u−st)

 これが、3直線AD、BE、CFの交点となる。  (証終)


重心座標

 ガウス平面上において、同一直線上にない異なる3点A(α)、B(β)、C(γ)をとると、
△ABCが定まる。このとき、ガウス平面上の任意の点P(z)に対して、

  z=sα+tβ+uγ 、s+t+u=1

となる実数(s,t,u)が、△ABCのもとで、ただ一組定まる。

   

 この(s,t,u)を、△ABCを基礎三角形とする点Pの重心座標と言う。

 実際に、直線APと直線BCの交点をD(δ)とおくとき、Dは線分BCを u’ : t’ に分ける点

なので、 δ=(t’β+u’γ)/(t’+u’) と書ける。

 同様に、Pは、線分ADを t’+u’ : s’ に分ける点なので、

 z=(s’α+(t’+u’)δ)/(s’+t’+u’)=(s’α+t’β+u’γ)/(s’+t’+u’)

と書ける。そこで、改めて、

 s=s’/(s’+t’+u’)、t=t’/(s’+t’+u’)、u=u’/(s’+t’+u’)

と置きなおせば、

  z=sα+tβ+uγ 、s+t+u=1

となる実数(s,t,u)が、△ABCのもとで、ただ一組定まる。


 この重心座標の考えを用いて、次の問題を再考してみよう。


問題  △ABCの底辺BCに平行な直線が辺AB、ACと交わる点をそれぞれD、Eとおく。
    線分BEとCDの交点をFとおくと、直線AFは、底辺BCの中点を通ることを示せ。

    

(解) ガウス平面において、A(α)、B(β)、C(γ)とおくと、F(z)は、

  z=sα+tβ+uγ (s+t+u=1)

と書ける。

 z−tβ=sα+uγ より、 (z−tβ)/(−t+1)=(sα+uγ)/(u+s) なので、

Eは線分ACを u : s に分ける点である。

 同様にして、

 z−uγ=sα+tβ より、 (z−uγ)/(−u+1)=(sα+tβ)/(t+s) なので、

Dは線分ABを t : s に分ける点である。

 線分DEと線分BCは平行なので、 t=u が成り立つ。すなわち、

  z=sα+tβ+tγ (s+2t=1)

から、 z−sα=tβ+tγ より、 (z−sα)/(−s+1)=(β+γ)/2

 したがって、直線AFは、底辺BCの中点を通る。  (終)


(コメント) チェバの定理を利用する場合に比べて、まだまだ煩雑ですが、以前の計算より
      は、だいぶ簡略化できましたね!重心座標の概念の素晴らしさに感動しました。


問題  △ABCの辺BC、CA、ABを k : 1 に分ける点をそれぞれP、Q、Rとする。
    直線APとCR、BQとAP、CRとBQの交点をそれぞれD、E、Fとする。

     このとき、△ABCと△DEFの重心は一致することを示せ。

     

(解) ガウス平面において、A(α)、B(β)、C(γ)とおく。

Dの重心座標を(s,t,u)とおくと、D(z)は、z=sα+tβ+uγ (s+t+u=1) と書ける。

 z−sα=tβ+uγ から、 (z−sα)/(−s+1)=(tβ+uγ)/(u+t)

 よって、PはBCを、u : t に分ける点である。

題意より、 u : t=k : 1 なので、 u=tk である。

同様に、 z−uγ=sα+tβ から、(z−uγ)/(−u+1)=(sα+tβ)/(t+s)

 よって、Rは、ABを、t : s に分ける点である。

題意より、 t : s=k : 1 なので、 t=sk である。よって、 u=sk2

以上から、 z=(α+kβ+k2γ)/(1+k+k2) と書けることが分かる。

 同様にして、E(z’)は、 z’=(β+kγ+k2α)/(1+k+k2

 F(z”)は、 z”=(γ+kα+k2β)/(1+k+k2

よって、△DEFの重心G(g)は、

 g=(z+z’+z”)/3={(α+β+γ)/3}・(1+k+k2)/(1+k+k2)=(α+β+γ)/3

したがって、△ABCと△DEFの重心は一致する。  (終)


面積

 三角形の面積の公式から、

例2. 座標平面上で、A( a , b )、B( c , d )、O( 0 ,0 ) のとき、

 △OAB=(1/2)| ad−bc |   (|  |は絶対値を表す)


であるので、この公式を複素数で表現することは易しい。

 Oをガウス平面の原点とし、A(α)、B(β)とする。α=a+i・b 、β=c+i・d とおくと、

 α=(a+i・b)(c−i・d)=ac+bd+i・(bc−ad)

から、 ad−bc=−Im(α) なので、 △OAB=(1/2)|Im(α)| となる。

Im(α)=(αβ)/(2i) なので、 △OAB=(1/4)|αβ| とも書ける。

 実際の計算には、前者の方が有効だろう。


問題  O、A(2+i)、B(−1+2i) に対して、△OABの面積を求めよ。

(解) Im(2+i)(−1−2i)=−1−4=−5 から、△OAB=5/2  (終)


 面積の公式をガウス平面上でも求めておこう。

 3点O、A、Bが反時計回りの順にあるとして、OAとOBのなす角をθとおく。

 このとき、△OAB=(1/2)|α||β|sinθ である。

 ここで、θ=arg(β/α) なので、sinθ=|Im(β/α)|/(|β/α|)

ここで、 1/α=/|α|2 なので、 sinθ=Im(β)/(|α||β|)

よって、 S=(1/2)|Im(β)|=(1/2)|Im(α)| となる。


 当HPがいつもお世話になっているHN「よおすけ」さんから、令和4年12月30日付けで、
問題をいただきました。一部改題しました。

名古屋大学前期文系(2019)

 非負の整数nに対して、P_n を xy 平面上の点とする。P_0 の座標を (1,0) とし、P_n の座
標 (x_n,y_n) とP_(n+1) の座標 (x_(n+1),y_(n+1)) は、

  x_(n+1)=x_n−k(y_n+y_(n+1))

  y_(n+1)=y_n+k(x_n+x_(n+1))

をみたすとする。ただし、kを正の実数とする。

(1) k=tan(α/2) とする。ただし、0<α<π とする。

  このとき、P_1、P_2 の座標(x_1,y_1)、(x_2,y_2) をαを用いて表せ。

(2) P_n の座標 (x_n,y_n) を、(1)のαとnを用いて表せ。

(3) Oを xy 平面の原点とするとき、△P_nOP_(n+1) の面積をkを用いて表せ。


(解) z_n=x_n+i・y_n とおくと、条件式は、 (1−ik)z_(n+1)=(1+ik)z_n となる。

 すなわち、 z_(n+1)=(1+ik)2/(1+k2)・z_n=(1−k2+2ik)/(1+k2)・z_n

 ここで、 (1−k2)/(1+k2)=(1−tan2(α/2) )/(1+tan2(α/2) )=cosα

  2k/(1+k2)=2tan(α/2) )/(1+tan2(α/2) )=sinα

なので、 z_(n+1)=(cosα+i・sinα)z_n となる。

(1) 題意より、z_0=1 なので、 z_1=cosα+i・sinα

 よって、 x_1=cosα 、y_1=sinα

また、 z_2=(cosα+i・sinα)2=cos2α+i・sin2α より、

 x_2=cos2α 、y_2=sin2α

となる。

(2) z_(n+1)=(cosα+i・sinα)z_n より、

 z_n=(cosα+i・sinα)z_0=cos(nα)+i・sin(nα)

なので、  (x_n,y_n) =(cos(nα),sin(nα))

(3) P_n(cos(nα),sin(nα))、P_(n+1)(cos((n+1)α),sin((n+1)α)) なので、

△P_nOP_(n+1)

=(1/2)|sin(nα)cos((n+1)α)−cos(nα)sin((n+1)α)|

=(1/2)|sin(nα−(n+1)α)|

=(1/2)|sinα|

 条件より、0<α<π なので、 sinα>0 より、 △P_nOP_(n+1)=(1/2)sinα

よって、 △P_nOP_(n+1)=k/(1+k2) である。  (終)


 よおすけさんから問題をいただきました。(令和5年1月16日付け)

 上記の名古屋大学前期文系の問題と同様の趣旨の計算が含まれています。

問題  pを有理数とし、次の関係をもつ x_n、y_n を座標にもつ平面上の点P_n (n=1,2,......)を
    考える。

  x_(n+1)=x_n+p(y_(n+1)+y_n) 、y_(n+1)=y_n-p(x_(n+1)+x_n)

 いま、x_1、y_1 がともに有理数で、かつ、P_1は原点ではないとする。このとき、

 すべての x_n、y_n は有理数であり、点P_n は原点を中心とする定円上にあることを示せ。

(出典:京都大学前期文系(1981))

(解) 条件式を x_(n+1)=x_n-i・p(i・y_(n+1)+i・y_n) 、i・y_(n+1)=i・y_n-i・p(x_(n+1)+x_n) と変

 形して、z_n=x_n+i・y_n とおくと、 z_(n+1)=z_n-i・p(z_(n+1)+z_n) と書ける。すなわち、

 (1+i・p)z_(n+1)=(1-i・p)z_n から、 z_(n+1)=((1-p2)/(1+p2)-2pi/(1+p2))z_n

 よって、 p=tanθ とおくと、 z_(n+1)=(cos2θ-i・sin2θ)z_n なので、

 |z_(n+1)|=|z_n| が成り立つ。

 条件から、x_1、y_1 がともに有理数で、|z_1|= r とおくと、全ての自然数nに対して、

|z_n|= r となる。(厳密には、数学的帰納法で示されるが、自明だろう。)

 さて、z_(n+1)=((1-p2)/(1+p2)-2pi/(1+p2))z_n から、

 x_(n+1)=(1-p2)/(1+p2)・x_n+2p/(1+p2)・y_n

 y_(n+1)=-2p/(1+p2)・x_n+(1-p2)/(1+p2)・y_n

である。全ての自然数nに対して、x_n、y_n は有理数であることを数学的帰納法により示す。

n=1 のときは、条件から明らかに成り立つ。

n=k (k≧1) のとき、命題が成り立つと仮定する。すなわち、x_k、y_k は有理数である。

 pは有理数より、(1-p2)/(1+p2)、2p/(1+p2)も有理数で、x_(k+1)、y_(k+1) は有理数

であることが分かる。

 よって、n=k+1 のときも命題は成り立つ。

以上から、すべての自然数nに対して、x_n、y_n は有理数で、点P_n は原点を中心とする定

円上にあることが示された。  (終)


 上記のよおすけさんの問題では、次の定理が効果的に用いられている。


ド・モアブルの定理

 加法定理から、

 (cosθ+i・sinθ)(cosφ+i・sinφ)=cos(θ+φ)+i・sin(θ+φ)

が成り立つことは明らかだろう。これより、一般に、

 (cosθ+i・sinθ)=cos(nθ)+i・sin(nθ) (ド・モアブルの定理)

が成り立つ。ここで、n は任意の整数である。


問題  △ABCにおいて、 (cosA+i・sinA)(cosB+i・sinB)(cosC+i・sinC)の値を
    求めよ。

(解) 与式=cos(A+B+C)+i・sin(A+B+C)=cosπ+i・sinπ=−1  (終)


問題  △ABCにおいて、(cosA+i・sinA)/{(cosB+i・sinB)(cosC+i・sinC)}が実数
    であるとき、△ABCの形状を求めよ。

(解) 与式=cos(A−B−C)+i・sin(A−B−C) が実数なので、 sin(A−B−C)=0

 0<A、B、C<π より、 −2π<A−B−C<π なので、 A−B−C=−π、0

 A−B−C=−π のとき、 A+B+C=π から、 A=0 となり、これは起こり得ない。

 A−B−C=0 のとき、 A+B+C=π から、 A=π/2 となり、△ABCは、A=90°

の直角三角形となる。

 以上から、△ABCは、A=90°の直角三角形である。  (終)


問題  次の計算をせよ。

(1) (sin(π/3)+i・cos(π/3))6

(2) (−cos(π/4)+i・sin(π/4))8

(解)(1) 与式=(cos(π/6)+i・sin(π/6))6=cosπ+i・sinπ=−1

(2) 与式=(cos(3π/4)+i・sin(3/4))8=cos6π+i・sin6π=1  (終)


問題  z=1−cosθ+i・sinθ について、z2n を求めよ。

(解) z=1−cosθ+i・sinθ=2sin2(θ/2)+i・2sin(θ/2)cos(θ/2)

   =2sin(θ/2)(sin(θ/2)+i・cos(θ/2))

   =2sin(θ/2)(cos((π−θ)/2)+i・sin((π−θ)/2))

 よって、 z2n=4sin2n(θ/2)(cos(n(π−θ))+i・sin(n(π−θ))

 したがって、 z2n=(−4)sin2n(θ/2)(cos(nθ)−i・sin(nθ))  (終)


問題  {(+i)/(1−i)}6 を計算せよ。

(解) +i=2(cos(π/6)+i・sin(π/6))、1−i=(cos(−π/4)+i・sin(−π/4))

なので、 (+i)/(1−i)=(cos(5π/12)+i・sin(5π/12))

よって、 {(+i)/(1−i)}6=8(cos(5π/2)+i・sin(5π/2))=8i  (終)


問題  α=+i のとき、 α4/(2i+α)2 の値を求めよ。

(解) α=+i=2(cos(π/6)+i・sin(π/6)) より、

 α4=16(cos(2π/3)+i・sin(2π/3))

また、 2i+α=+3i=2(cos(π/3)+i・sin(π/3)) より、

 (2i+α)2=12(cos(2π/3)+i・sin(2π/3))

したがって、 α4/(2i+α)2=4/3  (終)


問題  複素数 z=r(cosθ+i・sinθ) (r>0) は実数でなく、z+1/z が実数であるとき、
    z+1/z の値を求めよ。ただし、nは自然数とする。

(解) z=r(cosθ+i・sinθ) は実数でないことから、 sinθ≠0

 z+1/z=r(cosθ+i・sinθ)+(1/r)(cosθ−i・sinθ)

 =(r+1/r)cosθ+i・(r−1/r)sinθ が実数であることから、 (r−1/r)sinθ=0

sinθ≠0 より、 r−1/r=0 よって、 r=1 となる。すなわち、 z=cosθ+i・sinθ

このとき、

 z+1/z=cos(nθ)+i・sin(nθ)+cos(nθ)−i・sin(nθ)=2cos(nθ)  (終)


(追記) 令和5年12月6日付け

 上記の問題に関連して、次の問題が慶應義塾大学で出題されている。

問題  複素数 z=r(cosθ+i・sinθ) (r>0) に対して、z+1/z が実数であるとき、
    z+1/z も実数であることを示せ。ただし、nは整数で、z≠0とする。

(解) z+1/z=r(cosθ+i・sinθ)+(1/r)(cosθ−i・sinθ)

 =(r+1/r)cosθ+i・(r−1/r)sinθ が実数であることから、 (r−1/r)sinθ=0

よって、 r−1/r=0 または、 sinθ=0

 r−1/r=0 のとき、r=1 となる。すなわち、 z=cosθ+i・sinθ となる。

このとき、

 z+1/z=cos(nθ)+i・sin(nθ)+cos(nθ)−i・sin(nθ)=2cos(nθ) は実数

 sinθ=0 のとき、 z=r・cosθ 、1/z=(1/r)・cosθ より、

 z+1/z=(r+(1/r))cosθ は実数

以上から、何れにしても、z+1/z も実数である。  (終)


問題  以下で 、θ≠2nπ (nは整数) とする。次の等式が成り立つことを示せ。

(1) Σk=1 cos(k−1)θ={cos((n−1)θ/2)sin(nθ/2)}/sin(θ/2)

(2) Σk=1n-1 sin(kθ)={sin((n−1)θ/2)sin(nθ/2)}/sin(θ/2)

(解) z=cosθ+i・sinθ に対して、z=cos(nθ)+i・sin(nθ)

 (1)の左辺を a 、(2)の左辺を b とおくと、

 a+i・b=1+z+z2+・・・+zn-1=(1−z)/(1−z)

ここで、

1−z

=1−cos(nθ)−i・sin(nθ)

=2sin2(nθ/2)−2i・sin(nθ/2)cos(nθ/2)

=2sin(nθ/2)(sin(nθ/2)−i・cos(nθ/2))

=2sin(nθ/2)(cos(nθ/2)+i・sin(nθ/2))・(−i)

特に、1−z=2sin(θ/2)(cos(nθ/2)+i・sin(nθ/2))・(−i)

よって、

a+i・b

=sin(nθ/2)(cos(nθ/2)+i・sin(nθ/2))/sin(θ/2)(cos(nθ/2)+i・sin(nθ/2))

=(sin(nθ/2)/sin(θ/2))(cos((n−1)θ/2)+i・sin((n−1)θ/2))

より、

a=Σk=1 cos(k−1)θ={cos((n−1)θ/2)sin(nθ/2)}/sin(θ/2)

b=Σk=1n-1 sin(kθ)={sin((n−1)θ/2)sin(nθ/2)}/sin(θ/2)  (終)


(コメント) 等式の左辺に 2sin(θ/2)を掛けて、積和の公式から求められるが、なぜ
      「2sin(θ/2)」を掛けるのか、その理由が分かる解き方でした。


(追記) 当HPがいつもお世話になっているHN「よおすけ」さんから問題をいただきました。
     (令和5年7月16日付け)

問題  次の式を証明せよ。

 sin220°+sin240°+sin260°+sin280°=9/4

(解) z=cosθ+i・sinθ の両辺を n 乗すると、zn=cos(nθ)+i・sin(nθ)

 同様に、n の代わりに −n とすると、z-n=cos(−nθ)+i・sin(−nθ) より、

1/zn=cos(nθ)−i・sin(nθ)

 この2式を辺々足すと、cos(nθ)=(1/2)(zn+1/zn)

辺々引くと、 sin(nθ)=(−i/2)(zn−1/zn) が得られる。

 ここで使うのは、 sin(nθ)=(−i/2)(zn−1/zn) の方。

 θ=20°とし、n=1、2、3、4 をおのおの代入すると、

 sin2(1×20°)=(−i/2)2(z−1/z)2=−(1/4)(z2+1/z2−2)

 sin2(2×20°)=(−i/2)2(z2−1/z2)2=−(1/4)(z4+1/z4−2)

 sin2(3×20°)=(−i/2)2(z3−1/z3)2=−(1/4)(z6+1/z6−2)

 sin2(4×20°)=(−i/2)2(z4−1/z4)2=−(1/4)(z8+1/z8−2)

より、

sin220°+sin240°+sin260°+sin280°

=−(1/4)(z2+1/z2+z4+1/z4+z6+1/z6+z8+1/z8−8)

ここで、z=cosθ+i・sinθ で、θ=20°としたので、z9=cos180°+i・sin180°=−1

より、 z9+1=(z+1)(z8−z7+z6−z5+z4−z3+z2−z+1)=0

 z=cos20°+i・sin20°≠−1 より、 z8−z7+z6−z5+z4−z3+z2−z+1=0

よって、 z8−z7+z6−z5+z4−z3+z2−z=−1

ここで、z7=−1/z2、z5=−1/z4、z3=−1/z6、z=−1/z8 より

 z8−z7+z6−z5+z4−z3+z2−z

=z8+1/z2+z6+1/z4+z4+1/z6+z2+1/z8

=z2+1/z2+z4+1/z4+z6+1/z6+z8+1/z8=−1

よって、 sin220°+sin240°+sin260°+sin280°=−(1/4)(−1−8)=9/4

(出典)九州大学前期理系(2016年)問題5

(補足) 「加法定理」に既に載っていますが、小問(1)に、z=cosθ+i・sinθ の記述があっ
    たのでそれを使いました。


問題  複素数 z が z5+z=1 、|z|=1 を満たすとき、z の値を求めよ。

(解) |z|=1 より、 z=cosθ+i・sinθ とおける。このとき、

 (cos5θ+i・sin5θ)+(cosθ+i・sinθ)=cos5θ+cosθ+i・(sin5θ+sinθ)=1

より、 cos5θ+cosθ=1 、sin5θ+sinθ=0

 cos25θ+sin25θ=1 なので、 (1−cosθ)2+sin2θ=1

すなわち、 cosθ=1/2 より、 sinθ=±/2 より、 z=(1±i)/2

 何れも、 z5+z=1 、|z|=1 という条件を満たす。  (終)


問題  複素数 z が |z−1|=1 を満たし、z3 が実数のとき、z の値を求めよ。

(解) |z−1|=1 より、 z=1+cosθ+i・sinθ とおける。このとき、

 z=2cos2(θ/2)+i・2sin(θ/2)cos(θ/2)

 =2cos(θ/2)(cos(θ/2)+i・sin(θ/2))

よって、 z3 =8cos3(θ/2)(cos(3θ/2)+i・sin(3θ/2)) が実数なので、

 cos3(θ/2)sin(3θ/2)=0 より、 cos(θ/2)=0 または sin(3θ/2)=0

cos(θ/2)=0 のとき、 θ/2=nπ±π/2 すなわち、 θ=2nπ±π

 このとき、 z=0

sin(3θ/2)=0 のとき、 3θ/2=nπ すなわち、 θ=2nπ/3

 このとき、 z=1+cos(2nπ/3)+i・sin(2nπ/3)

 n=0、1、2 を代入して、 z=2、(1±)/2

よって、 z=0、2、(1±)/2  (終)


 上記の解の中で、「1+cosθ+i・sinθ=2cos(θ/2)(cos(θ/2)+i・sin(θ/2))」と
いう式変形は、この分野では頻出な気がする。すなわち、次の顕著な公式が存在する。

公式
    

(証明) 1+cosθ+i・sinθ=2cos(θ/2)(cos(θ/2)+i・sin(θ/2))

同様に、 1+cosθ+i・sinθ=2cos(θ/2)(cos(−θ/2)+i・sin(−θ/2))

よって、

左辺=(cos(nθ/2)+i・sin(nθ/2))/(cos(−nθ/2)+i・sin(−nθ/2))

  =cos(nθ)+i・sin(nθ)=右辺 より、成り立つ。  (証終)


問題  複素数 z が z+1/z=1 のとき、z+1/z (nは自然数)の取り得る値を求めよ。

(解) z=r(cosθ+i・sinθ) (r>0) とおくと、(r+1/r)cosθ+i・(r−1/r)sinθ=1 から、

 (r+1/r)cosθ=1 、(r−1/r)sinθ=0

 r−1/r=0 すなわち、 r=1 のとき、 cosθ=1/2 より、θ=±π/3

  このとき、z=cos(±π/3)+i・sin(±π/3)=cos(π/3)+i・sin(±π/3) なので、

  z+1/z=2cos(nπ/3)

 sinθ=0 すなわち、 θ=nπ のとき、 r+1/r=1 から、 r2−r+1=0

  この式を満たす実数 r は存在しない。

 以上から、z+1/z (nは自然数)の取り得る値は、2cos(nπ/3)=±1、±2  (終)


問題  複素数 z が z4−2z3cosθ+2z2−2zcosθ+1=0 を満たすとき、次の値を求
    めよ。

(1) z+1/z  (2) z+1/z (nは整数)

(解)(1) z≠0 から、与式の両辺を z で割って、

 z2−2zcosθ+2−2cosθ(1/z)+1/z2=0

すなわち、 (z+1/z)2−2cosθ(z+1/z)=0

 よって、 z+1/z=0 、2cosθ

(2) z+1/z=0 のとき、 z2=−1 から、 z=±i

 z=i のとき、 1/z=−i

 z=−i のとき、 1/z=i

なので、 z+1/z=(i)+(−i)

 n=4k (k:整数)のとき、 z+1/z=1+1=2

 n=4k+1 (k:整数)のとき、 z+1/z=i−i=0

 n=4k+2 (k:整数)のとき、 z+1/z=−1−1=−2

 n=4k+3 (k:整数)のとき、 z+1/z=−i+i=0

また、z+1/z=2cosθ のとき、 z2−2zcosθ+1=0 より、 z=cosθ±i・sinθ

 z=cosθ+i・sinθ のとき、 1/z=cosθ−i・sinθ

 z=cosθ−i・sinθ のとき、 1/z=cosθ+i・sinθ

なので、

 z+1/z=(cosθ+i・sinθ)+(cosθ−i・sinθ)

  =(cos(nθ)+i・sin(nθ))+(cos(nθ)−i・sin(nθ))=2cos(nθ)  (終)


問題  a=((+1)/2+i・(−1)/2) が実数となる最小の自然数 n を求めよ。

(解) a=((()/4+i・()/4)

    =((cos(π/12)+i・sin(π/12))

    =((cos(nπ/12)+i・sin(nπ/12))

 a が実数となるためには、 sin(nπ/12)=0 で、この式を満たす最小の自然数 n は、

n=12 である。  (終)


問題  α=cos(2π/7)+i・sin(2π/7) のとき、

A=1/(1−α)+1/(1−α2)+1/(1−α3)+1/(1−α4)+1/(1−α5)+1/(1−α6

の値を求めよ。(出典:自治医科大学(2017)を改題)

(解) α7=cos(2π)+i・sin(2π)=1 なので、

A=1/(1−α)+1/(1−α2)+1/(1−α3)+α3/(α3−1)+α2/(α2−1)+α/(α−1)

 =1/(1−α)+α/(α−1)+1/(1−α2)+α2/(α2−1)+1/(1−α3)+α3/(α3−1)

 =1/(1−α)−α/(1−α)+1/(1−α2)−α2/(1−α2)+1/(1−α3)−α3/(1−α3

 =1+1+1=3  (終)


問題  z=cos(2π/7)+i・sin(2π/7) に対して、

 a=z+1/z 、b=z2+1/z2 、c=z3+1/z3

とおく。このとき、次の問いに答えよ。(出典:立教大学理学部(2021)を改題)

(1) a+b+c の値を求めよ。

(2) a2+b2+c2 の値を求めよ。

(3) ab+bc+ca の値を求めよ。

(解)(1) z7=cos(2π)+i・sin(2π)=1 なので、

 (z−1)(z6+z5+z4+z3+z2+z+1)=0

 z≠1 なので、 z6+z5+z4+z3+z2+z+1=0

このとき、

a+b+c=z+1/z+z2+1/z2+z3+1/z3=z+z6+z2+z5+z3+z4=−1

(2) a2+b2+c2

=z2+2+1/z2+z4+2+1/z4+z6+2+1/z6

=z2+2+z5+z4+2+z3+z6+2+z

=z6+z5+z4+z3+z2+z+6=5

(3) ab+bc+ca={(a+b+c )2−(a2+b2+c2)}/2=(1−5)/2=−2  (終)


(コメント) ド・モアブルの定理の応用問題を意図しましたが、本質は単なる式の値の計算
      でした。


問題  cos(4π/5) 、cos(2π/5) の値をそれぞれ求めよ。

(解) z=cos(4π/5)+i・sin(4π/5) とおくと、 z5=1 より、

 (z−1)(z4+z3+z2+z+1)=0

 z≠1 なので、 z4+z3+z2+z+1=0

 z≠0 なので、両辺を z2 で割って、 z2+z+1+1/z+1/z2=0 より、

 (z+1/z)2+(z+1/z)−1=0

よって、 z+1/z=(−1±)/2 となる。

 ここで、z+1/z=2cos(4π/5)<0 から、 2cos(4π/5)=(−1−)/2

したがって、 cos(4π/5)=(−1−)/4=−(1+)/4

 このとき、 cos2(2π/5)=(1+cos(4π/5))/2=(3−)/8=(6−2)/16

cos(2π/5)>0 なので、 cos(2π/5)=(−1)/4  (終)


(別解) θ=2π/5 とおくと、 5θ=2π

よって、 cos3θ=cos(2π−2θ)=cos2θ から、

 4cos3θ−3cosθ=2cos2θ−1 より、4cos3θ−2cos2θ−3cosθ+1=0

よって、 (cosθ−1)(4cos2θ+2cosθ−1)=0

 cosθ≠1 より、 4cos2θ+2cosθ−1=0

 cosθ>0 なので、 cosθ=(−1)/4

 このとき、 cos2θ=2cos2θ−1=−(1+)/4  (終)


問題  α=cos(2π/n)+i・sin(2π/n) に対して、

 (1−α)(1−α2)(1−α3)・・・(1−αn-1

の値を求めよ。

(解) α=1 より、αは、z=1 の解である。同様に、α2、α3、・・・、αn-1 はすべて異

なり、z=1 の解である。

 よって、z−1=(z−1)(z−α)(z−α2)(z−α3)・・・(z−αn-1) と因数分解できる。

z≠1 より、

 zn-1+zn-2+・・・+z2+z+1=(z−α)(z−α2)(z−α3)・・・(z−αn-1

 ここで、z=1 とおくと、 (1−α)(1−α2)(1−α3)・・・(1−αn-1)=n  (終)


問題  α=cos(2π/5)+i・sin(2π/5) のとき、

 z4+z3+z2+z+1=(z−α)(z−α2)(z−α3)(z−α4

であることを示せ。

(解) α5=1 より、αは、z5=1 の解である。同様に、α2、α3、α4 はすべて異なり、

5=1 の解である。

 よって、z5−1=(z−1)(z−α)(z−α2)(z−α3)(z−α4) と因数分解できる。

z≠1 より、

 z4+z3+z2+z+1=(z−α)(z−α2)(z−α3)(z−α4)  (終)


問題  z6+z5+z4+z3+z2+z+1 を2次式の積に分解せよ。

(解) α=cos(2π/7)+i・sin(2π/7) とおくと、 α7=1 で、前問と同様にして、

 z7−1=(z−1)(z−α)(z−α2)(z−α3)・・・(z−α6

z≠1 より、

6+z5+z4+z3+z2+z+1

=(z−α)(z−α2)(z−α3)・・・(z−α6

=(z−α)(z−α6)・(z−α2)(z−α5)・(z−α3)(z−α4

=(z−α)(z−1/α)・(z−α2)(z−1/α2)・(z−α3)(z−1/α3

=(z2−(α+1/α)z+1)(z2−(α2+1/α2)z+1)(z2−(α3+1/α3)z+1)

=(z2−2(cos(2π/7))z+1)(z2−2(cos(4π/7))z+1)(z2−2(cos(6π/7))z+1)

(終)


問題  sinα+sinβ+sinγ=0 、cosα+cosβ+cosγ=0 のとき、

 sin3α+sin3β+sin3γ=3sin(α+β+γ)

 cos3α+cos3β+cos3γ=3cos(α+β+γ)

が成り立つことを示せ。

(解) u=cosα+i・sinα 、v=cosβ+i・sinβ 、w=cosγ+i・sinγ とおく。

 このとき、 u+v+w=0 であるので、

 u3+v3+w3−3uvw=(u+v+w)(u2+v2+w2−uv−vw−wu)=0

よって、 u3+v3+w3=3uvw=3(cos(α+β+γ)+i・sin(α+β+γ))

ここで、

 u3+v3+w3=cos3α+cos3β+cos3γ+i・(sin3α+sin3β+sin3γ) なので、

 sin3α+sin3β+sin3γ=3sin(α+β+γ)

 cos3α+cos3β+cos3γ=3cos(α+β+γ)

が成り立つ。  (終)


 よおすけさんからのコメントです。(令和5年7月29日付け)

 僕が7年前、平成28年4月18日付けで投稿した、

 お茶の時間-クイズ&パズル-三角形の形状2

に関連問題があります。


問題  x=cosα+i・sinα、y=cosβ+i・sinβ、z=cosγ+i・sinγ について、

 xyz=x+y+z が成り立つとき、cos(β−γ)+cos(γ−α)+cos(α−β) の値を

求めよ。

(解) 条件より、 cos(α+β+γ)=cosα+cosβ+cosγ

 sin(α+β+γ)=sinα+sinβ+sinγ

が成り立つ。このとき、

 cos(β−γ)+cos(γ−α)+cos(α−β)

=cosβcosγ+sinβsinγ+cosγcosα+sinγsinα+cosαcosβ+sinαsinβ

=cosβcosγ+cosγcosα+cosαcosβ+sinβsinγ+sinγsinα+sinαsinβ

=(1/2)((cosα+cosβ+cosγ)2−(cos2α+cos2β+cos2γ))

 +(1/2)((sinα+sinβ+sinγ)2−(sin2α+sin2β+sin2γ))

=(1/2)(cos2(α+β+γ)−(cos2α+cos2β+cos2γ))

 +(1/2)(sin2(α+β+γ)−(sin2α+sin2β+sin2γ))

=(1/2)(cos2(α+β+γ)+sin2(α+β+γ))

 −(1/2)(cos2α+sin2α)−(1/2)(cos2β+sin2β)−(1/2)(cos2γ+sin2γ)

=1/2−3/2=−1  (終)


(コメント) この問題については、当初次のように計算していた。

cosα+cosβ=cos(α+β+γ)−cosγ=−2sin(γ+(α+β)/2)sin((α+β)/2)

sinα+sinβ=sin(α+β+γ)−sinγ=2cos(γ+(α+β)/2)sin((α+β)/2)

であるので、

cos(β−γ)+cos(γ−α)+cos(α−β)

=cosβcosγ+sinβsinγ+cosγcosα+sinγsinα+cos(α−β)

=cosγ(cosα+cosβ)+sinγ(sinα+sinβ)+cos(α−β)

=−2sin(γ+(α+β)/2)sin((α+β)/2)cosγ

 +2cos(γ+(α+β)/2)sin((α+β)/2)sinγ+cos(α−β)

=−2sin((α+β)/2)(sin(γ+(α+β)/2)cosγ−cos(γ+(α+β)/2)sinγ)

 +cos(α−β)

=−2sin((α+β)/2)・sin((α+β)/2)+cos(α−β)

=−2sin2((α+β)/2)+cos(α−β)

=−1+cos(α+β)+cos(α−β)

=−1+2cosαcosβ

=・・・・・・・・・・・・・・???


 何かおかしいという違和感だけが残る計算であった。本解のように解いてみると、その解
法の素晴らしさが十分堪能できる。果たして、「cosαcosβ=0」は示されるのだろうか?


 よおすけさんからのコメントです。(令和5年8月5日付け)

  以前、お茶の時間クイズ&パズルに同様の問題を投稿していたことを思い出して探して
いたら、お茶の時間クイズ&パズル「幾何学」にありました。10年くらい前に投稿した気が
する...と、うろ覚えでしたが、平成24年3月21日付けで、約11年前のことでした。


(コメント) x、y、z が単位円周上の点で、xyz=x+y+z が成り立つとき、かなり特殊な状
      況になるんですね!「幾何学」における図形的な解答も参考になりました。上記の
      別解が得られて嬉しいです!よおすけさんに感謝します。

 それはそうと、学士会館で行われた羽生善治九段の講演会を聞きながら、上記の別解を
ずっと考えていました。次のような別解を思いつきました。

(別解) xyz=x+y+z から、 1/(yz)+1/(zx)+1/(xy)=1 なので、

cos(β+γ)−i・sin(β+γ)+cos(γ+α)−i・sin(γ+α)

 +cos(α+β)−i・sin(α+β)=1

すなわち、 cos(β+γ)+cos(γ+α)+cos(α+β)=1

  sin(β+γ)+sin(γ+α)+sin(α+β)=0

このとき、 (cos(β+γ)+cos(γ+α))2=(1−cos(α+β))2

  (sin(β+γ)+sin(γ+α))2=(−sin(α+β))2

展開して辺々加えると、

 2+2(cos(β+γ)cos(γ+α)+sin(β+γ)+sin(γ+α))=2−2cos(α+β)

すなわち、 2+2cos(β−α)=2−2cos(α+β) から、

 cos(α−β)=−cos(α+β)

が成り立つ。同様にして、

 cos(β−γ)=−cos(β+γ)、cos(γ−α)=−cos(γ+α)

よって、

cos(β−γ)+cos(γ−α)+cos(α−β)

=−(cos(β+γ)+cos(γ+α)+cos(α+β))=−1

が言える。  (終)


(コメント) この別解の副産物として、上記で話題になった

 「『cosαcosβ=0』は示されるのだろうか?」

が肯定的に解決されることに気づかされた。

 すなわち、cos(α−β)=−cos(α+β) から、cos(α+β)+cos(α−β)=0

和積の公式により、 2cosαcosβ=0 すなわち、 cosαcosβ=0 が成り立つ。


#めでたし、めでたし!無事解決してスッキリしました。


 ド・モアブルの定理の応用として、複素数のn乗根の計算が有名である。

問題  x4=16 を解け。

(解) x2=±4 から、x=±2、±2i  (終)

 この4つの解をガウス平面上に図示すると、綺麗な正方形が得られる。

  

 この問題をド・モアブルの定理を用いて解いてみよう。

(別解) x=r(cosθ+i・sinθ) (r>0) とおくと、

  r4(cos4θ+i・sin4θ)=16(cos0+i・sin0)

よって、 r4=16 から、 r=2

 4θ=2nπ から、 θ=nπ/2

 n=0、1、2、3 として、θ=0、π/2、π、3π/2 なので、求める解は、

 x=2(cos0+i・sin0)=2

 x=2(cosπ/2+i・sinπ/2)=2i

 x=2(cosπ+i・sinπ)=−2

 x=2(cos3π/2+i・sin3π/2)=−2i  (終)


 読者のために、練習問題を残しておこう。

練習問題  x4=−1 を解け。

(解) x=r(cosθ+i・sinθ) (r>0) とおくと、

  r4(cos4θ+i・sin4θ)=cos(π)+i・sin(π)

よって、 r4=1 から、 r=1

 4θ=π+2nπ から、 θ=π/4+nπ/2

 n=0、1、2、3 として、θ=π/4、3π/4、5π/4、7π/4 なので、求める解は、

 x=cosπ/4+i・sinπ/4=(1+i)/

 x=cos3π/4+i・sin3π/4=(−1+i)/

 x=cos5π/4+i・sin5π/4=(−1−i)/

 x=cos7π/4+i・sin7π/4=(1−i)/  (終)


問題  連立方程式 x3−3xy2=1/ 、y3−3x2y=−1/ を解け。ただし、解は
    実数とする。

(解) z=x+i・y とおくと、 z3=x3−3xy2−i・(y3−3x2y)=(1+i)/

このとき、 z3=cos(π/4)+i・sin(π/4)

 ここで、z=cosθ+i・sinθ とおくと、

 cos3θ+i・sin3θ=cos(π/4)+i・sin(π/4)

よって、3θ=π/4+2nπ から、 θ=π/12+2nπ/3

 n=0、1、2 として、θ=π/12、3π/4、17π/12 なので、求める解は、

 z=cosπ/12+i・sinπ/12=()/4+i・()/4

  このとき、x=()/4 、y=()/4

 z=cos3π/4+i・sin3π/4=(−1+i)/

  このとき、x=−1/ 、y=1/

 z=cos17π/12+i・sin17π/12=−()/4−i・()/4

  このとき、x=−()/4 、y=−()/4  (終)


問題  z4+2z3+4z2+8z+16=0 を解け。

(解) 両辺を16で割って、z/2=t とおくと、 t4+t3+t2+t+1=0

 両辺に t−1 を掛けて、 t5−1=0 となる。

 t=cosθ+i・sinθ とおくと、 cos5θ+i・sin5θ=1=cos0+i・sin0

 よって、 5θ=2nπ (n=0、1、2、3、4) より、 θ=2nπ/5

 t≠1 なので、 n≠0 より、求める解は、

 z=2(cos2π/5+i・sin2π/5) 、2(cos4π/5+i・sin4π/5) 、

  2(cos6π/5+i・sin6π/5) 、2(cos8π/5+i・sin8π/5)  (終)


(追記) 当HPがいつもお世話になっているHN「よおすけ」さんから、問題をいただきました。
    (令和5年8月12日付け)

問題  0<α<3π/5、0<β<2π/5 のとき、次の式を満たすα、βの値を求めよ。

  (cosα+i・sinβ)2=1/(2)+i・(+2sinαcosβ)

ただし、i= とする。

(出典) 東北大学 文理共通(1969)

(解) cos2α−sin2β+i・2cosαsinβ=1/(2)+i・(+2sinαcosβ) より、

 cos2α−sin2β=1/(2) 、2cosαsinβ=+2sinαcosβ

第2式より、 sinαcosβ−cosαsinβ=−1/

すなわち、 sin(α−β)=−1/ で、−2π/5<α−β<3π/5 より、

 α−β=−π/4

 α=β−π/4 を第1式に代入して、 cos2(β−π/4)−sin2β=1/(2)

 ((1/)cosβ+(1/)sinβ)2−sin2β=1/(2)

すなわち、 (cosβ+sinβ)2−2sin2β=1/ より、 

 cos2β−sin2β+2sinβcosβ=1/

すなわち、 cos2β+sin2β=1/ から、 sin(2β+π/4)=1/2

 π/4<2β+π/4<21π/20 なので、 2β+π/4=5π/6

よって、 β=7π/24 で、 α=β−π/4=π/24  (終)


(コメント) 誤って、ド・モアブルの定理を想起させるようなトラップ問題でしたね!


問題  x の2次方程式 x2+2x+k=0 (kは実数) が虚数解αをもち、その3乗が実数
    になるという。このとき、kの値を求めよ。

(解) αが解なら、その共役複素数も解である。

 α=r(cosθ+i・sinθ) (r>0、0≦θ<π) とおくと、 =r(cosθ−i・sinθ)

 解と係数の関係から、 α+=−2 すなわち、 2rcosθ=−2 より、 rcosθ=−1

また、 α=k すなわち、 r2=k

 題意より、 α3=r3(cos3θ+i・sin3θ) が実数であることから、 sin3θ=0

 また、αは虚数解なので、 sinθ≠0 より、 θ≠nπ

 よって、 θ=π/3 、2π/3 なので、 cosθ=±1/2 すなわち、 r=2

 したがって、 k=4  (終)


問題  数列 {z} は、初項が9で、公比が (i)/4 の等比数列である。この数列
    の項が実数であるものを初めから順に、a1,a2,a3,・・・ とする。

    このとき、無限級数 Σn=1 の和を求めよ。

(解) 公比を r とおくと、

r=(i)/4=(1/)(/2+i・(1/2))=(1/)(cos(π/6)+i・sin(π/6))

よって、 r=(1/(cos(nπ/6)+i・sin(nπ/6)) なので、これが実数となるのは

 nπ/6=mπ (mは自然数) より、 n=6m なので、 n=6、12、・・・

したがって、 a1=9,a2=9r6=−9/8,a3=9/64,・・・ から、無限級数 Σn=1 は、

初項が9で、公比が−1/8の無限等比級数となる。よって、これは収束して、和は、

 Σn=1 =9/(1−(−1/8))=8  (終)


問題  (+i)=(1+i) が成り立つ自然数 m、n の値を求めよ。

(解) 2m(cos(mπ/6)+i・sin(mπ/6))=(cos(nπ/4)+i・sin(nπ/4))

より、 2m から、 n=2m で、

cos(mπ/6)=cos(nπ/4)=cos(mπ/2) 、sin(mπ/6)=sin(nπ/4)=sin(mπ/2)

から、 mπ/2=mπ/6+2kπ (kは自然数) より、 m=6k

 このとき、 n=12k  (終)


問題  ((+i)/2)+i=0 が成り立つ自然数 n の値を求めよ。

(解) cos(nπ/6)+i・sin(nπ/6)+cos(nπ/2)+i・sin(nπ/2)=0 より、

 cos(nπ/6)+cos(nπ/2)=0 すなわち、 2cos(nπ/3)cos(nπ/6)=0

 sin(nπ/6)+sin(nπ/2)=0 すなわち、 2sin(nπ/3)cos(nπ/6)=0

ここで、cos(nπ/3)とsin(nπ/3)が同時に0になることはないので、 cos(nπ/6)=0

よって、 nπ/6=kπ−π/2 (kは自然数) すなわち、 n=6k−3  (終)


問題  m、n は自然数とする。1の相異なるn個のn乗根それぞれのm乗の総和を求めよ。

(解) ω=cos(2π/n)+i・sin(2π/n) とおくと、ω=1 で、1の相異なるn個のn乗根

は、1、ω、ω2、・・・、ωn-1 である。このとき、ω+ω2+・・・+ωn-1+ω=0 である。

 Sm=ωm+(ω2m+・・・+(ωn-1m+(ωm とおく。このとき、

 Sm=ωm+(ωm2+・・・+(ωmn-1+(ωmn

である。ここで、ωm=cos(2mπ/n)+i・sin(2mπ/n) なので、

 m=m’n (m’は自然数) のとき、 ωm=1 なので、 Sm=n

 m≠m’n (m’は自然数) のとき、ω=1 から、(ωm=1 で、ωm≠1 から、

  1+ωm+(ωm2+・・・+(ωmn-1=0

 すなわち、 ωm+(ωm2+・・・+(ωmn-1+(ωmn=0

よって、Sm=ωm+(ωm2+・・・+(ωmn-1+(ωmn=0  (終)


 よおすけさんから問題をいただきました。(令和5年8月26日付け)

問題  i を虚数単位とし、a=cos(π/3)+i・sin(π/3) とおく。また、nは、
   すべての自然数にわたって動くとする。このとき、次の問いに答えよ。

(1) an は、何個の異なる値をとり得るか。

(2) {(1-a^n)(1-a^(2n))(1-a^(3n))(1-a^(4n))(1-a^(5n))}/{(1-a)(1-a^2)(1-a^3)(1-a^4)(1-a^5)}
  の値を求めよ。

(出典) 東京大学理系2次(1970)
※補足:1970年当時、東京大学は1次試験・2次試験が実施されていた。

(解)(1) an=cos(nπ/3)+i・sin(nπ/3) より、 π/3≦nπ/3≦2π より、1≦n≦6

 nは自然数なので、 n=1、2、3、4、5、6 の6個

(2) n≡1 (mod 6) のとき、1≦m≦5 に対して、

 a^(mn)=a^(6mk+m)=a^(6mk)a^(m)=a^(m) なので、 与式=1

n≡2 (mod 6) のとき、1≦m≦5 に対して、

 a^(mn)=a^(6mk+2m)=a^(6mk)a^(2m)=a^(2m) なので、m=3 のとき、a^(2m)=1

 よって、 与式=0

n≡3 (mod 6) のとき、1≦m≦5 に対して、

 a^(mn)=a^(6mk+3m)=a^(6mk)a^(3m)=a^(3m) なので、m=2 のとき、a^(3m)=1

 よって、 与式=0

n≡4 (mod 6) のとき、1≦m≦5 に対して、

 a^(mn)=a^(6mk+4m)=a^(6mk)a^(4m)=a^(4m) なので、m=3 のとき、a^(4m)=1

 よって、 与式=0

n≡5 (mod 6) のとき、1≦m≦5 に対して、

 a^(mn)=a^(6mk+5m)=a^(6mk)a^(5m)=a^(5m) なので、

 a^(n)=a^(5) 、a^(2n)=a^(10)=a^(4) 、a^(3n)=a^(15)=a^(3)

 a^(4n)=a^(20)=a^(2) 、a^(5n)=a^(25)=a^(1) なので、 与式=1

n≡6 (mod 6) のとき、1≦m≦5 に対して、

 a^(mn)=a^(6mk+6m)=a^(6mk)a^(6m)=a^(6m)=1 なので、 与式=0  (終)



変換

 複素数 z に複素数 w が1対1に対応するとき、複数数上の変換と言われる。この変換は
文字Fによって、

  F : z → w  または、 w=F(z)

などと書く。これまでにも、例1例2例3例4などで、変換に関する問題を扱ってきた。

 ここで、変換について、まとめていきたいと思う。


問題  ω=(−1+i)/2 とおく。ガウス平面において、3点A(1)、B(ω)、C(ω2)を
    考える。正三角形ABCにおいて、各頂点A、B、Cを中心とするπ/3の回転移動を
    RA、RB、RCとする。これらの移動の合成 RCBA は、点Bに関する対称移動で
    あることを示せ。

    

(解) 題意より、 w−1=(z−1)α (ただし、α=(1+i)/2=−ω2 ) なので、

 w=RA(z)=−ω2(z−1)+1=−ω2z+ω2+1=−ω2z−ω

同様にして、 p=RB(w)=(w−ω)α+ω=(−ω2z−2ω)(−ω2)+ω=ωz+2+ω

 すなわち、 p=ωz+1−ω2

 q=RC(p)=(p−ω2)α+ω2=(ωz+1−ω2−ω2)(−ω2)+ω2=−z+2ω

 以上から、 (q+z)/2=ω が成り立つ。

 よって、合成変換 RCBA は、点Bに関する対称移動を表す。  (終)


問題  △ABCの外側に正三角形BPC、CAQ、ABRを作る。さらに、下図のように、正三
    角形QRSを作る。このとき、点Aは線分PSの中点であることを示せ。

   

# 上記の問題では、移動の合成 RCBA が、点Bに関する対称移動であることが示さ
 れた。この事実を用いると、本問は次のように解かれる。

(解) 上図において、B(1)、A(ω)、R(ω2)としても一般性は失われない。各頂点B、A、

 Rを中心とするπ/3の回転移動を LB、L、LR とすると、

  LB : P → C 、L : C → Q 、LR : Q → S

から、移動の合成 LRAB は、点Aに関する対称移動となる。すなわち、点Aは線分PSの

中点であることが分かる。  (終)


問題  原点中心の角θの回転移動をR、複素数αによって表される平行移動をHとする。
    このとき、2つの移動の合成 HR の不動点を求め、その不動点を作図せよ。

(解) 題意より、 R(z)=z・e^(iθ) 、H(z)=z+α なので、 HR(z)=z・e^(iθ)+α

 不動点を z1 とすると、 z1・e^(iθ)+α=z1 から、 z1=α/(1−e^(iθ))

 ここで、 α=r(cosφ+i・sinφ) とおく。

 z1=α/(1−e^(iθ))

  =r(cosφ+i・sinφ)/(1−cosθ−i・sinθ)

  =r(cosφ+i・sinφ)/(2sin2(θ/2)−i・2sin(θ/2)cos(θ/2))

  =(r/(2sin(θ/2))(cosφ+i・sinφ)/(sin(θ/2)−i・cos(θ/2))

  =(r/(2sin(θ/2))(cosφ+i・sinφ)/(cos(θ/2−π/2)+i・sin(θ/2−π/2))

  =(r/(2sin(θ/2))(cos(φ+π/2−θ/2)+i・sin(φ+π/2−θ/2)

 以上から、不動点 z1 は、下図のように作図される。

   


問題  点A(α)中心の角θの回転移動をR、点B(β)中心の角φの回転移動をR’とする。
    このとき、移動の合成R’Rの不動点をC(c)、移動の合成RR’の不動点をC’(c’)と
    すると、CとC’は直線ABに関して対称であることを示せ。

      

(解) R(z)=(z−α)e^(iθ)+α 、R’(z)=(z−β)e^(iφ)+β なので、

 c=R’R(c)=R’((c−α)e^(iθ)+α)=((c−α)e^(iθ)+α−β)e^(iφ)+β

すなわち、 c=ce^(i(θ+φ))−αe^(i(θ+φ))+(α−β)e^(iφ)+β より、

 c(1−e^(i(θ+φ)))=−αe^(i(θ+φ))+(α−β)e^(iφ)+β

同様にして、

 c’=RR’(c’)=R((c’−β)e^(iφ)+β)=((c’−β)e^(iφ)+β−α)e^(iθ)+α

すなわち、 c’=c’e^(i(θ+φ))−βe^(i(θ+φ))+(β−α)e^(iθ)+α より、

 c’(1−e^(i(θ+φ)))=−βe^(i(θ+φ))+(β−α)e^(iθ)+α

そこで、

 (c+c’)(1−e^(i(θ+φ)))

=−(α+β)e^(i(θ+φ))+(α−β)(e^(iφ)−e^(iθ))+α+β

=α(1+e^(iφ)−e^(iθ)−e^(i(θ+φ)))+β(1−e^(iφ)+e^(iθ)−e^(i(θ+φ)))

 (1+e^(iφ)−e^(iθ)−e^(i(θ+φ)))+(1−e^(iφ)+e^(iθ)−e^(i(θ+φ)))

=2(1−e^(i(θ+φ)))

なので、 (c+c’)/2 は、αとβを結ぶ直線上にあることが分かる。

 さらに、

 (c−c’)(1−e^(i(θ+φ)))

=(β−α)e^(i(θ+φ))−(β−α)(e^(iφ)+e^(iθ))+β−α

=(β−α)(e^(i(θ+φ))−(e^(iφ)+e^(iθ))+1)

=(β−α)(1−e^(iφ))(1−e^(iθ))

から、

 (c−c’)/(β−α)=(1−e^(iφ))(1−e^(iθ))/(1−e^(i(θ+φ)))

ここで、

(1−e^(iφ))(1−e^(iθ))/(1−e^(i(θ+φ)))
 +(1−e^(−iφ))(1−e^(−iθ))/(1−e^(−i(θ+φ)))

=(1−e^(iφ))(1−e^(iθ))/(1−e^(i(θ+φ)))
 +(e^(iφ)−1)(e^(iθ)−1)/(e^(i(θ+φ))−1)

=(1−e^(iφ))(1−e^(iθ))/(1−e^(i(θ+φ)))
 −(1−e^(iφ))(1−e^(iθ))/(1−e^(i(θ+φ)))

=0 であることから、(1−e^(iφ))(1−e^(iθ))/(1−e^(i(θ+φ)))は純虚数となる。

すなわち、(c−c’)/(β−α)が純虚数から、 cc’⊥αβ が分かる。

 以上から、CとC’は直線ABに関して対称である。  (終)


問題  点A(α)を中心とする角θの回転移動をR、点B(β)を中心とする角φの回転移動
    をR’とする。ガウス平面上の任意の点C(z)に対して、z1=R’R(z) 、z2=RR’(z)
    となる点D(z1)、E(z2)を定める。

     このとき、zに無関係に、DE=z2−z1 は定数になることを示せ。

      

(解) 題意より、 R(z)=(z−α)e^(iθ)+α 、R’(z)=(z−β)e^(iφ)+β なので、

1=R’R(z)

 =((z−α)e^(iθ)+α−β)e^(iφ)+β

 =ze^(i(θ+φ))−αe^(i(θ+φ))+(α−β)e^(iφ)+β

2=RR’(z)

 =((z−β)e^(iφ)+β−α)e^(iθ)+α

 =ze^(i(θ+φ))−βe^(i(θ+φ))−(α−β)e^(iθ)+α

よって、

2−z1=(α−β)e^(i(θ+φ))−(α−β)(e^(iθ)+e^(iφ))+α−β

    =(α−β)(1−e^(iθ))(1−e^(iφ))

 ここで、右辺は、z に無関係な定数であるので、

 zに無関係に、DE=z2−z1 は定数になる。  (終)


問題  下図のように、△ABCの外側に正三角形ABP、ACQを作る。辺BCの中点Mから
    垂線RMを引き、RM=BC/(2)とする。

     このとき、 PR=QR 、∠QRP=2π/3 であることを示せ。

    

(解) ガウス平面において、A(α)、B(−b)、C(b) (bは正の実数) とすると、

M(0)で、R(bi) となる。

 このとき、C中心の角π/3の回転移動をFとおくと、

 F(z)−b=(z−b)(ω+1)=−ω2(z−b)=−ω2z+bω2

すなわち、 F(z)=−ω2z+b(ω2+1)=−ω2z−bω

同様に、B中心の角π/3の回転移動をGとおくと、

 G(z)+b=(z+b)(ω+1)=−ω2(z+b)=−ω2z−bω2

すなわち、 G(z)=−ω2z−b(ω2+1)=−ω2z+bω

 このとき、 GF(z)=−ω2(−ω2z−bω)+bω=ωz−bω2

 回転移動の合成GFの不動点を z0 とおくと、 z0=ωz0bω2 より、

 z0bω2/(ω−1)=bi となるので、点Rは合成変換GFの不動点となる。

 点R中心で角2π/3の回転移動をHとすると、H(z)−bi=(z−bi)ω より、

 H(z)=(z−bi)ω+bi=ωz+bi(1−ω)=ωz+b(ω+1)=ωz−bω2

なので、 H=GF である。 F:Q → A で、G:A → P から、 H:Q → P である。

 したがって、 PR=QR 、∠QRP=2π/3 である。  (終)


問題  点A(α)を中心とする角θの回転移動をRとし、点B(−α)を中心とする角π−θ
    の回転移動をR’とする。

    ガウス平面上の任意の点z1に対して、z2=R(z1)、z3=R’(z2)とする。

    このとき、線分z13は、A、Bを直径の両端とする円周によって、2等分されることを
   示せ。

  

(解) 題意より、 z2=(z1−α)e^(iθ)+α 、

 z3=(z2+α)e^(i(π−θ))−α

  =−((z1−α)e^(iθ)+2α)e^(−iθ)−α=−z1−2αe^(−iθ)

 よって、回転移動の合成R’Rの不動点z0は、 z0=−αe^(−iθ) で、この点は、

A、Bを直径の両端とする円周上にあり、線分z13の中点で、回転移動の合成R’Rは点対

称移動となる。

 以上から、線分z13は、A、Bを直径の両端とする円周によって、2等分される。  (終)


相似変換

 平行移動や回転移動は、図形の形や大きさをを変えない変換で、合同変換と言われる。

 平行移動や回転移動を表す変換式は、 F(z)=z・e^(iθ)+α と記述される。

 特に、e^(iθ)=1 すなわち、θ=0 のときは、平行移動を表し、それ以外は、回転移
動を表す。

 直線 z+t2=(2r)t (t は法線方向の複素数、r は原点と直線との距離) に関する対
称移動 F(z)=−t2+(2r)t は、図形の形は裏返しになるが大きさをを変えない変換で
ある。

 一般に、 G(z)=・e^(iθ)+α と記述される変換は、裏返し合同変換と言われる。

 裏返し合同変換は、線対称移動、回転移動、平行移動の合成で表される。

例 実軸に関する対称移動 F(z)= 、原点中心の角θの回転移動 G(z)=z・e^(iθ)、
 平行移動 H(z)=z+α に対して、その合成 HGF を考えると、

  HGF(z)=・e^(iθ)+α

と書くことが出来、これは裏返し合同変換そのものである。

   

 合同変換に対して、形を変えずに大きさのみを変える変換は、相似変換と言われる。

 相似の中心をC(c)とし、相似比をkとおくと、相似変換は、

  F(z)=k(z−c)+c (kは実数、cは複素数)

と記述される。


例 ガウス平面において、半径の異なる2つの円O、O’がある。円Oを円O’に変換する相
  似変換が2通りあることは、下図を見れば明らかだろう。C、C’が相似の中心である。



 実際に、O(0)、O(α) (α≠0 の実数)とすると、

 円Oの方程式は、 z=r・e^(iθ)

 円O’の方程式は、 w=r’・e^(iφ)+α

とおける。求める相似変換を F(z)=k(z−c)+c (kは実数、cは複素数) とおく。

 このとき、 w=F(z)=k(r・e^(iθ)−c)+c=(1−k)c−kr・e^(iθ)

 これが、 w=r’・e^(iφ)+α と一致するためには、

  (1−k)c=α 、 |kr|=|r’|

 第2式より、 k=±r’/r で、これを第1式に代入して、

  c=α/(1−k)=rα/(r−r’) または、 c=α/(1−k)=rα/(r+r’)

ここで、 F(z)=kz+(1−k)c=(±r’/r)z+α から、

 相似の中心がCのとき、OPO’P’は同じ向きで、

 相似の中心がC’のとき、OPO’P’は反対向きとなる。


 今、2つの相似変換F、Gを考える。すなわち、

 F(z)=k(z−c)+c (kは実数、cは複素数)

 G(z)=k’(z−c’)+c’ (k’は実数、c’は複素数)

 このとき、F、Gの合成変換GFを考えると、

 GF(z)=k’(k(z−c)+c−c’)+c’=k’kz+ck’(1−k)+c’(1−k’)

(イ) GFが相似変換になる場合、相似の中心をdとおくと、

 GF(z)=h(z−d)+d (hは実数、dは複素数)

の形に書けるはずである。すなわち、GF(d)=d となるdが存在するはずである。

 k’kd+ck’(1−k)+c’(1−k’)=d より、 (1−k’k)d=ck’(1−k)+c’(1−k’)

このとき、k’k≠1 であれば、d=(ck’(1−k)+c’(1−k’))/(1−k’k) と定まるので、

  GF(z)−d

 =k’kz+ck’(1−k)+c’(1−k’)−(k’kd+ck’(1−k)+c’(1−k’))

 =k’k(z−d)

より、 GF(z)=k’k(z−d)+d (k、k’は実数、dは複素数) となる。

 すなわち、k’k≠1 のとき、2つの相似変換F、Gの合成GFは相似変換となり、

 相似の中心は、d=(ck’(1−k)+c’(1−k’))/(1−k’k) で、相似比k’k である。

このとき、 d=(ck’(1−k)+c’(1−k’))/((1−k’)+k’(1−k)) と変形してみると、

d は、c と c’ を結ぶ線分を 1−k’ : k’(1−k) の比に分ける点であることが分かる。

    

(ロ) k’k=1 のとき、

 GF(z)=k’(k(z−c)+c−c’)+c’=z+(c’−c)(1−1/k)

となり、これは平行移動を表す変換である。

    

 変換式から明らかなように、この平行移動は、c と c’ を結ぶ線分に平行である。


相似変換の逆変換

 相似変換 w=F(z)=k(z−c)+c (kは実数、cは複素数) の逆変換を考える。

 このとき、 z=(1/k)(w−c)+c より、 F-1(w)=(1/k)(w−c)+c であるので、

Fの逆変換F-1は、 F-1(z)=(1/k)(z−c)+c (kは実数、cは複素数) と表される。

 よって、F-1の相似の中心とFの相似の中心は一致し、相似比は逆数となる。


相似回転 ・・・ 相似変換と回転の合成変換

 相似変換 F(z)=k(z−c)+c (kは実数、cは複素数)

 回転の中心 c で角θの回転移動 G(z)=e^(iθ)・(z−c)+c

に対して、 GF(z)=e^(iθ)・(k(z−c)+c−c)+c=ke^(iθ)・(z−c)+c

 ここで、ke^(iθ)=α とおくと、αは複素数で、相似回転Hは、

  H(z)=α(z−c)+c (α≠0)

と表される。


問題 ガウス平面において、A(1)、B(z)を結ぶ線分を斜辺とする直角三角形ABCを作る。
   このとき、C(w)として、w を z を用いて表せ。

(解) 題意より、

 w=(1/)e^(i(±π/4))(z−1)+1=(1±i)(z i)/2 (複号同順)  (終)


相似回転の合成

 2つの相似回転F、Gを考える。すなわち、

 F(z)=α(z−c)+c (α、cは複素数で、α≠0)

 G(z)=β(z−c’)+c’ (β、c’は複素数で、β≠0)

 このとき、F、Gの合成変換GFを考えると、

 GF(z)=β(α(z−c)+c−c’)+c’=αβz+cβ(1−α)+c’(1−β)

(イ) GFが相似回転になる場合、回転の中心をdとおくと、

 GF(z)=αβ(z−d)+d (α、β、dは複素数)

の形に書けるはずである。すなわち、GF(d)=d となるdが存在するはずである。

 αβd+cβ(1−α)+c’(1−β)=d より、 (1−αβ)d=cβ(1−α)+c’(1−β)

このとき、αβ≠1 であれば、d=(cβ(1−α)+c’(1−β))/(1−αβ) と定まるので、

  GF(z)−d

 =αβz+cβ(1−α)+c’(1−β)−(αβd+cβ(1−α)+c’(1−β))

 =αβ(z−d)

より、 GF(z)=αβ(z−d)+d (α、β、dは複素数で、αβ≠0) となる。

 すなわち、αβ≠1 のとき、2つの相似回転F、Gの合成GFは相似回転となり、

 回転の中心は、d=(cβ(1−α)+c’(1−β))/(1−αβ) である。

(ロ) αβ=1 のとき、

 GF(z)=z+c(β−1)+c’(1−β)=z+(c’−c)(1−1/α)

となり、これは平行移動を表す変換である。


 相似回転の概念で物事を見ると、簡単に道理が分かるような気がする。そのような問題を
いくつか練習しよう。

問題  ガウス平面の原点OとOとは異なる2点A、Bで、△OABを考える。辺OA、OBを斜
    辺として三角形の外側に直角2等辺三角形OPA、OQBを作る。
     辺ABの中点をMとするとき、MP=MQ、MP⊥MQ が成り立つことを示せ。

        

 相似回転の概念を用いない直接的な解法は、こちらを参照。

(解) A(α)、B(β)、M(m) とおくと、題意より、m=(α+β)/2 である。

 Bを回転の中心とするQ→Oの相似回転をF、Aを回転の中心とするO→Pの相似回転をG

とすると、

 F(z)=e^(πi/4)・(z−β)+β

 G(z)=(1/)e^(πi/4)・(z−α)+α

である。F、Gの合成GFは、

 GF(z)

=(1/)e^(πi/4)・(e^(πi/4)・(z−β)+β−α)+α

=i・(z−β)+(1+i)(β−α)/2+α

=i・z−i・β+(α+β)/2+i・(β−α)/2

=i・z+(α+β)/2−i・(α+β)/2

=i・(z−(α+β)/2)+(α+β)/2

=i・(z−m)+m

 これは、Mを回転の中心とする角90°の回転移動を表し、Mは不動点である。

 よって、 MP=MQ 、MP⊥MQ が成り立つ。  (終)


(コメント) Mをガウス平面の原点とおいた方が、GF(z)=i・z となって、式は美しくなる。


問題  円Oの周上の点Pを中心として、円Oを相似回転して円O’を作る。この相似回転
   で、円Oの周上の点Aは、円O’上の点Bにうつるものとする。
     このとき、直線ABは定点を通り、しかもその定点は2円O、O’の交点の一つであ
   ることを示せ。

   

(解) ガウス平面において、P(0)とする。円Oの半径を r とすると、O(r・e^(iθ0))と表さ

れ 円Oの方程式は、 z=r・e^(iθ)+r・e^(iθ0)=r・(e^(iθ)+e^(iθ0)) と書ける。

 点Pを中心とする相似回転Rを、 R(z)=αz とおくと、円O’の方程式は、

 z=αr(e^(iθ)+e^(iθ0)) と書ける。

 このとき、直線ABの方程式は、

 (z−r(e^(iθ)+e^(iθ0)))/{(α−1)r(e^(iθ)+e^(iθ0))}

=(−r(e^(−iθ)+e^(−iθ0)))/{(−1)r(e^(−iθ)+e^(−iθ0))}

=(e^(iθ)e^(iθ0−r(e^(iθ)+e^(iθ0)))/{(−1)r(e^(iθ)+e^(iθ0))}

より、

 (−1)(z−r(e^(iθ)+e^(iθ0)))=(α−1)(e^(iθ)e^(iθ0−r(e^(iθ)+e^(iθ0)))

すなわち、

 e^(iθ)・((α−1)e^(iθ0)+r(−α))=(−1)z−r(−α)e^(iθ0

が成り立つ。この式が任意のθについて成り立つためには、

 (α−1)e^(iθ0)+r(−α)=0 かつ (−1)z−r(−α)e^(iθ0)=0

第2式より、 z=r(−α)e^(iθ0)/(−1) で、これは、第1式を満たす。

すなわち、直線ABは、定点 r(−α)e^(iθ0)/(−1) を通る。

 この点を、円Oの式に代入してみると、

  r(−α)e^(iθ0)/(−1)=r・(e^(iθ)+e^(iθ0))

 を解いて、 e^(iθ)=e^(iθ0)・{(1−α)/(−1)}

 |(1−α)/(−1)|=1 なので、上式を満たすθは確かに存在し、定点は円Oの周

上にあることが分かる。同様にして、円O’の周上にもあることが分かる。

 以上から、直線ABは定点を通り、しかもその定点は2円O、O’の交点の一つであること

が示された。  (終)


等形変換 ・・・ 相似な図形に変換

 一般式は、 F(z)=az+b (a、bは複素数で、a≠0) で表される。

例 ガウス平面上の3点A(α)、B(β)、C(γ)を結び、△ABCを作る。この三角形に等形
  変換を施して得られる三角形を△A’B’C’とする。A’(α’)、B’(β’)、C’(γ’)とすると、

 α’=aα+b 、β’=aβ+b 、γ’=aγ+b

である。このとき、明らかに、(α’−β’)/(γ’−β’)=(α−β)/(γ−β) が成り立つ。

 よって、 △A’B’C’∽△ABC である。


 等形変換は、2つの値 a、b により定まるので、適当な2つの条件で決定することができる。


問題  ガウス平面において、3点A(i)、B(−2)、C(3)がある。AB、ACを斜辺とする直角
    2等辺三角形ABD、ACEを△ABCの外側に作る。

  このとき、△ACEを△BADに移す等形変換Fを求めよ。

(解) 求める等形変換Fを、F(z)=az+b とおくと、題意より、 F(A)=B 、 F(C)=A

 なので、 a・i+b=−2 、 3a+b=i より、 a=(1+i)/2 、b=−(3+i)/2

 したがって、 F(z)={(1+i)z−(3+i)}/2 となる。  (終)


 上記の計算では、F(A)=B 、F(C)=A から、等形変換が求められたが、その変換に
よって、残りの頂点は対応する頂点に移される。すなわち、F(E)=D が成り立つ。

 実際に、E(α)、D(β)とおくと、 i−3=e^(πi/4)(α−3) から、

 α=3+(i−3)(1−i)/2=2+2i

また、 β+2=(1/)e^(πi/4)(i+2) より、 β=(−3+3i)/2

 このとき、 F(E)={(1+i)(2+2i)−(3+i)}/2=(−3+3i)/2=D が成り立つ。

 この性質を用いると、図形に関するいくつかの性質を示すことが出来る。


問題  △ABC∽△A’B’C’とする。このとき、線分AA’、BB’、CC’の中点をそれぞれ
    A”、B”、C”とおくと、△ABC∽△A”B”C”が成り立つ。

(解) ガウス平面において、A(α)、B(β)、C(γ)、A’(α’)、B’(β’)、C’(γ’) とおく。

 等形変換 F:△ABC → △A’B’C’ を、F(z)=az+b で定義すると、

 α’=aα+b 、β’=aβ+b 、γ’=aγ+b

である。このとき、A”(α”)、B”(β”)、C”(γ”)に対して、

 α”=(α+α’)/2=(a+1)α/2+b/2 、β”=(β+β’)/2=(a+1)β/2+b/2

 γ”=(γ+γ’)/2=(a+1)γ/2+b/2

 そこで、等形変換 G:△ABC → △A”B”C” を、G(z)={(a+1)/2}z+b/2 で定義

すれば、 △ABC∽△A”B”C” となる。  (終)


(コメント) 上記の問題では、中点の場合を考えたが、m : n の比に分ける点でも明らかに
      成り立つ。


問題  O、O’を中心とし、半径がそれぞれ r、r’ の円を考える。このとき、等形変換Fで
    円Oを円O’に移すものを求めよ。また、等形変換Fの不動点を求めよ。

(解) 等形変換Fでは、中心O(α)は中心O’(α’)に移される。

また、円Oの周上の点 α+re^(iθ)は、円O’の周上の点 α’+r’e^(iθ’)に移される。

そこで、F(z)=az+b とおくと、α’=aα+b で、α’+r’e^(iθ’)=a(α+re^(iθ))+b

2式を連立して、aα+b+r’e^(iθ’)=a(α+re^(iθ))+b から、r’e^(iθ’)=are^(iθ)

よって、 a=(r’/r)e^(i(θ’−θ)) となる。

 このとき、 b=α’−aα=α’−(r’/r)e^(i(θ’−θ))α となるので、

求める等形変換Fは、 F(z)=(r’/r)e^(i(θ’−θ))z+α’−(r’/r)e^(i(θ’−θ))α

すなわち、 F(z)=(r’/r)e^(i(θ’−θ))・(z−α)+α’

で表される。

 特に、(r’/r)e^(i(θ’−θ))=1 のとき、r’=r が成り立つので、e^(i(θ’−θ))=1

より、θ’=θ なので、 F(z)=z+α’−α となる。これは、平行移動を表し、不動点は

ない。

 不動点 z が存在するとすると、 (r’/r)e^(i(θ’−θ))・(z−α)+α’=z

すなわち、 (r’/r)e^(i(θ’−θ))=(z−α’)/(z−α)

 両辺の絶対値をとって、 |z−α’|/|z−α|=r’/r と書ける。

 r≠r’ のときは、アポロニウスの円の定義式から、z は、線分α’αを r’ : r の比に内分、

外分する点を直径の両端とする円周上にある。

 r=r’ のときは、線分α’αの垂直2等分線上にある。


 等形変換全体は群をなし、可換でないことは明らかだろう。

問題  位数3の等形変換を求めよ。

(解) 求める等形変換Fを、F(z)=az+b とおく。このとき、

 F3(z)=a(a(az+b)+b)+b=a3z+a2b+ab+b=z (恒等式) より、

 a3=1 、a2b+ab+b=(a2+a+1)b=0 が成り立つ。

 a3=1 より、 a=1 、ω 、ω2

 a=1 のとき、 b=0 で、F(z)=z となり、不適

 a=ω、ω2 のとき、 a2+a+1=0 なので、 bは任意の複素数

 よって、求める等形変換は、 F(z)=az+b (ただし、a=ω、ω2)  (終)


(コメント) 不動点について調べておこう。不動点を z0 とすると、 F(z0)=z0

 すなわち、 az0+b=z0 より、 b=z0−az0 なので、

 F(z)=az+z0−az0=a(z−z0)+z0

 このことから、位数3の等形変換Fは、不動点 z0 のまわりの回転であることが分かる。
(回転角は、2π/3 または 4π/3)


問題  2つの等形変換F、Gを、F(z)=z+i 、G(z)=2z+3i とする。このとき、HF=G
    となる等形変換Hを求めよ。

(解) F-1(z)=z−i なので、 H(z)=GF-1(z)=2(z−i)+3i=2z+i  (終)


裏返し等形変換

 合同変換で、「裏返し合同変換」を考えたように、等形変換でも「裏返し等形変換」が考え
られる。

 実軸に関する線対称移動(z → )で、図形は裏返されるので、裏返し等形変換Fの定
義式は、
     F(z)=a+b (a、bは複素数で、a≠0)

で表される。

 当然ながら、2つの裏返し等形変換の合成は、等形変換となる。

 実際に、 F(z)=p+q 、G(z)=r+s に対して、

 GF(z)=r(z+b)+s=(r)z+(rb+s) は、確かに、等形変換となる。

 このことから、裏返し等形変換の全体は、群にならないことが分かる。


問題  裏返し等形変換Fを、F(z)=i・+1 で定義する。また、法線方向の複素数が

  (1+i)/ で、原点と直線との距離を1とする直線の方程式は

  z+i・(1+i) (← ヘッセの標準形

で、この直線に関する線対称移動をGとすると、 G(z)=−i・(1+i) である。

 このとき、F=HG となる等形変換Hを求めよ。

(解) 題意より、G-1=G なので、 H=FG-1=FG である。

 よって、 H(z)=FG(z)=i・(i・z+(1−i))+1=−z+1+i  (終)


問題  ある点 c を中心とする相似比 k の相似変換Fは、原点を中心とする相似変換G
    と平行移動Hの合成で表されることを示せ。

(解) 題意より、 F(z)=k(z−c)+c と書けるので、 F(z)=kz+(1−k)c

 そこで、 原点を中心とする相似変換Gを、 G(z)=kz

       平行移動Hを、 H(z)=z+(1−k)c

と定義すれば、 F(z)=HG(z) が成り立つ。  (終)


問題  2つの相似変換F、Gを、 F(z)=k(z−c)+c 、G(z)=h(z−d)+d で定義す
    る。ただし、kh≠1とする。
    w=GF(z) 、w’=FG(z) とおくとき、 w’=H(w) となる変換Hを求めよ。

(解) w=h(k(z−c)+c−d)+d=hk(z−c)+h(c−d)+d

 w’=k(h(z−d)+d−c)+c=hk(z−d)−k(c−d)+c なので、

 w’−w=hk(c−d)−(h+k)(c−d)+c−d=(h−1)(k−1)(c−d)

よって、 H(w)=w+(h−1)(k−1)(c−d) と表される。

 この変換Hは平行移動である。  (終)


問題  △ABCの辺AB、ACをそれぞれ1辺とする正方形ABPQ、ACRSを△ABCの外側
    に作る。さらに、辺BCを斜辺とする直角二等辺三角形BCTを△ABCの内側に作る。
     このとき、 TQ=TS 、TQ⊥TS であることを示せ。

  

(解) ガウス平面において、a を正の実数として、 B(−a)、C(a) とおく。

Cを中心として、SをAに移す変換Fは、F(z)=(1/)e^(πi/4)・(z−a)+a

Bを中心として、AをQに移す変換Gは、G(z)=e^(πi/4)・(z+a)−a と書ける。

 このとき、合成変換GFは、

 GF(z)=e^(πi/4)・((1/)e^(πi/4)・(z−a)+2a)−a

     =e^(πi/2)・(z−a)+2a・e^(πi/4)−a=i・z+(1+i)a

で表され、SはQに移される。

 ここで、合成変換GFの不動点は、 z=i・z+(1+i)a より、 z=i・a で、点Tが不動点

となる。 よって、 GF(z)−i・a=i・z+a=i・z+i(−ia)=i・(z−i・a) より、

変換GFは、Tを中心とする回転角π/2の回転移動となる。

 したがって、 TQ=TS 、TQ⊥TS が成り立つ。  (終)


(追記) 令和5年3月6日付け

 上記では、変換の考えを用いて示したが、もちろん直接示すことも可能である。

(別解) 下図において、T(0)、A(α)、B(β)、C(γ)、S(s)、Q(q) とおく。

   

 このとき、 s=i・(γ−α)+α 、q=−i・(β−α)+α である。

さらに、 γ=i・β なので、 s=i・(i・βγ−α)+α=−β−i・α+α

よって、 i・s=−i・β+α+i・α=−i・(β−α)+α=q となるので、

 TQ=TS 、TQ⊥TS であることが分かる。  (終)


(コメント) 2つの解を比較して、(別解)の方が自然で、簡明な気がする。


問題  △ABC∽△A’B’C’とする。今、線分AA’、BB’、CC’をm:nに分ける点をそれ
   ぞれA”、B”、C”とするとき、△ABC∽△A”B”C”であることを示せ。

(解) ガウス平面において、A(α)、B(β)、C(γ)、A’(α’)、B’(β’)、C’(γ’)、
  A”(α”)、B”(β”)、C”(γ”)とおくと、

α”=(nα+mα’)/(m+n)、β”=(nβ+mβ’)/(m+n)、γ”=(nγ+mγ’)/(m+n)

等形変換F:△ABC → △A’B’C’ を、 F(z)=az+b で定義すると、

 α’=aα+b 、β’=aβ+b 、γ’=aγ+b

なので、

α”=((n+ma)α+mb)/(m+n)=(n+ma)α/(m+n)+mb/(m+n)

β”=((n+ma)β+mb)/(m+n)=(n+ma)β/(m+n)+mb/(m+n)

γ”=((n+ma)γ+mb)/(m+n)=(n+ma)γ/(m+n)+mb/(m+n)

 そこで、等形変換G:△ABC → △A”B”C” を、

  F(z)={(n+ma)/(m+n)}z+mb/(m+n)

で定義することが出来るので、 △ABC∽△A”B”C” であることが分かる。  (終)


問題  原点中心の相似回転Fと平行移動Gについて、GFの不動点 u とFGの不動点 v
    にはどんな関係式が成り立つか。

(解) 題意より、 F(z)=αz 、G(z)=z+β と書けるので、

 GF(z)=αz+β より、 αu+β=u  すなわち、 (1−α)u=β

α≠1 より、 u=β/(1−α)

 FG(z)=αz+αβ より、 αv+αβ=v すなわち、 (1−α)v=αβ

α≠1 より、 v=αβ/(1−α)

 以上から、 v=αu なので、点 v は、原点中心の u の相似回転で得られる。

また、 u−v=β すなわち、 v=u−β なので、点 v は、点 u を−βだけ平行移動して

得られる。  (終)


問題  次の問いに答えよ。

(1) z=(e^(iθ)+e^(−iθ))/2 は、両端の座標が 1 と −1 である線分を表すことを
   示せ。

(2) (1)の線分を、両端の座標が z1 と z2 である線分へ移す等形変換を求めよ。

(解)(1) z=cosθ なので、zは両端の座標が 1 と −1 である線分上を動く。

(2) 求める等形変換Fを、F(z)=az+b (a、bは複素数で、a≠0) とおく。

 F(1)=a+b=z1 、F(−1)=−a+b=z2 のとき、

  a=(z1−z2)/2 、b=(z1+z2)/2 より、

  F(z)=((z1−z2)/2)z+((z1+z2)/2)

 F(1)=a+b=z2 、F(−1)=−a+b=z1 のとき、

  a=(z2−z1)/2 、b=(z1+z2)/2 より、

  F(z)=((z2−z1)/2)z+((z1+z2)/2)  (終)


問題  裏返し等形変換Fに対して、線対称移動Gおよび原点中心の相似回転Hを適当に
    選んで、HG=F としたい。GとHをどのように定義すればよいだろうか。

(解) 与えられた裏返し等形変換Fを、F(z)=a+b (a、bは複素数で、a≠0) とおく。

 直線 z+t2=(2r)t (t は法線方向の複素数、r は原点と直線との距離) に関する対

称移動Gは、G(z)=−t2+(2r)t で、原点中心の相似回転Hは、H(z)=αz で定義さ

れる。このとき、

 HG(z)=−αt2+(2r)αt=a+b より、 a=−αt2 、b=(2r)αt なので、

 r=−bt/(2a) と書ける。r は実数なので、 −bt/(2a)=−/(2

 t は大きさ1の複素数なので、 =1/t

よって、 t2=a/(b) で、 α=−b/ と定めれば、 HG=F が成り立つ。

すなわち、 G(z)=−t2−(b/a)t2=−(a/(b))/ 、H(z)=−(b/)z

と定義すればよい。  (終)


問題  原点中心にπ/3回転してから、平行移動αを行う変換は、回転移動になることを
    示せ。

   

(解) F(z)=tz (ただし、t=e^(π/3)i ) 、G(z)=z+α より、

 GF(z)=tz+α で、不動点を z0 とおくと、 z0=tz0+α

 よって、 (1−t)z0=α

ここで、 t+=1 、t=1 なので、 z0=αt となる。

 このとき、 GF(z)−z0=t(z−z0) と書けるので、合成変換GFは、

 αt を中心として、π/3回転の回転移動を表す。  (終)

 この結果を活用すると、次の問題も即答だろう。

問題  ガウス平面において、A(z)、B(α)として、△OABを考え、その外側に正三角形
    OACを作る。次に、OB、OCを2辺とする平行四辺形COBDを作る。さらに、原点
    中心に、Bをπ/3回転させた点をEとおく。このとき、△ADEは正三角形になること
    を示せ。

    

 証明は、前問題から明らか。


反転 ・・・ 幾何問題における強力な武器ですね!

 中心がOで半径 r の円がある。Oと異なる任意の点Pに対して、 OP・OQ=r2 となる点
Qが半直線OP上にただ一つ定まる。

  

 このとき、点Pに点Qを対応させる変換は反転と言われ、点Oを反転の中心、r を反転
の半径
という。

 点Pの反転が点Qであれば、当然、点Qの反転は点Pとなる。

 反転の中心O(c)、P(z)、Q(w)とし、OP=p、OQ=q とおく。

 OPと同じ向きの大きさ1の複素数を t とすると、 z−c=pt 、w−c=qt と書ける。

このとき、定義より、 (w−c)()=pq=r2 となる。これより、

 w=c+r2/(

と書けるので、点Pに点Qを対応させる反転Fは、

 F(z)=c+r2/(

で表される。特に、単位円に関する反転Fは、 F(z)=1/ で表される。


問題  円 zz+α+1=0 (ただし、α>1) は、中心が−αで、半径が
    √(α−1)である。この円に関して、点z1の反転が点z2のとき、
     z121+α2+1=0 が成り立つことを示せ

(解) 題意より、 z1=−α+(α−1)/(2) なので、

  (z1+α)(2)=α−1

 これを展開して、 z121+α2+1=0 が成り立つ。  (終)


 反転の作図法については、「基本の作図」の「反転」で考えたが、次のように考えてもよい。

 次の図が基本となる。

    

 半直線OPとPで直交する直線を引き、円Oとの交点をA、Bとする。A、Bにおける円Oの
接線が交わる点をQとする。このとき、点Qは点Pの反転となる。

 実際に、△OPA∽△OAQ より、 OP/OA=OA/OQ なので、 OP・OQ=OA2=r2
より、点Qは点Pの反転となる。


問題  単位円Oの周上に2定点A(α)、B(β)をとる。Oと異なる点P(z)に対して、この円
    に関する点Pの反転をQ(w)とする。(Qの作図方法は、上記を参照)

  

 このとき、Pの取り方に無関係に、∠APB+∠AQB は一定であることを示せ。

(解) 題意より、w=1/ と表される。このとき、

 ∠APB=arg(β−z)/(α−z)

 ∠AQB=arg(β−1/)/(α−1/)=arg(β−1)/(α−1)

 α=β=1 なので、

 ∠AQB=arg(β−β)/(α−α)=arg(β/α)+arg()/(

ここで、 arg()/()=−arg(β−z)/(α−z)=−∠APB なので、

 ∠APB+∠AQB=arg(β/α) が成り立つ。

 この値はPの取り方に無関係な定数である。  (終)


 反転については、よく「直線」や「円」が対象となる。

 まず、直線の方程式 z+α+b=0 (α≠0、bは実数) に対して、単位円に関する
反転Fを考えてみよう。点zに対応する点をwとおくと、変換式 F(z)=1/ より、

 w=1/ すなわち、 =1/w 、z=1/ なので、直線の方程式に代入して、

 (1/)+α(1/w)+b=0 すなわち、 bww+α=0

 これは、b=0 ならば、元の直線と一致し、b≠0 ならば、円の方程式となる。

以上をまとめると、

 反転の中心(原点)を通る直線の反転は、元の直線と一致し、

 反転の中心(原点)を通らない直線の反転は、反転の中心(原点)を通る円である

ということが分かる。

 次に、円の方程式 zz+α+b=0 (bは実数) に対して、単位円に関する反
転Fを考えてみよう。点zに対応する点をwとおくと、変換式 F(z)=1/ より、

 w=1/ すなわち、 =1/w 、z=1/ なので、円の方程式に代入して、

(1/)(1/w)+(1/)+α(1/w)+b=0 すなわち、bww+α+1=0

 これは、b=0 ならば、直線となり、元の円は原点を通る。b≠0 ならば、円となり、元の

円は原点を通らない。

以上をまとめると、

 反転の中心(原点)を通る円の反転は、反転の中心(原点)を通らない直線である

 反転の中心(原点)を通らない円の反転は、反転の中心(原点)を通らない円である

ということが分かる。

 今まで、反転の定義で、「Oと異なる任意の点Pに対して」としてきたが、上記の計算でも
分かるように、非常に結果が煩雑である。ここをスッキリさせよう。

 反転の変換式 w=1/ で、z → 0 のとき、w → ∞ である。そこで、ガウス平面に

無限遠点{∞}を添加して考えると、理論的にスッキリしたものになる。

 ガウス平面に対して、∪{∞}はリーマン面と呼ばれる。このとき、ガウス平面上の
直線は、∞を通る円とみなされる。

 リーマン面の具体的なイメージは、いわゆる、球面である。

   

 ガウス平面上の原点を通る直線は、0と∞を通る大円となり、ガウス平面上の原点を通ら
ない直線は、∞を通る円となる。

 このように考えると、ガウス平面上の直線は、リーマン面では円と考えられる。

 このような一般化された円のことを、ガウス円と言う。

 ガウス平面において、

 直線の方程式は、 z+α+b=0 (α≠0、bは実数)

 円の方程式は、 zz+α+b=0 (bは実数で、|α|2−b>0)

であるが、両者を比較すると、直線と円の違いは、直線の方程式に、z を追加したものが
円の方程式になる点である。

 以上から、直線と円を総括したガウス円の方程式は、

  azz+α+b=0 (a、bは実数で、α≠0)

で与えられる。


 ガウス円という言葉を用いると、先の結果は、次のようにまとめられる。

(1) 0と∞を通るガウス円は、反転により、自分自身に移される。

(2) 0を通るガウス円は、(0を通らない)∞を通るガウス円に移される。

(3) 0も∞も通らないガウス円は、0も∞も通らないガウス円に移される。

 以上から、

 反転によって、ガウス円はガウス円に移される

と言える。

 実際に、ガウス円 azz+α+b=0 (a、bは実数で、α≠0) に対して、単位円

に関して反転を考えれば、反転の変換式 w=1/ より、

 a・(1/w)(1/)+(1/)+α(1/w)+b=0

すなわち、 bww+α+a=0 となり、ガウス円が得られる。


 0と∞を通るガウス円は、反転によって自分自身に移されるが、当然ながら、このような性
質を持つ円は無数にある。


 例えば、反転の中心をOとし反転の半径を r とすると、点Pの反転が点Qのとき、線分PQ
を直径とする円がそのような例になる。

 反転の中心をOから、線分PQを直径とする円に引いた接線の接点をTとすると、方べきの
定理から、 OT2=OP・OQ すなわち、 OT2=OP・OQ=r2 が成り立つので、接点Tは、
線分PQを直径とする円周上にある。

 したがって、反転により、線分PQを直径とする円は自分自身に移される。

 このことを、計算により追認してみよう。

問題  原点を中心とする反転によって、不変な円が存在することを示せ。

(解) 反転で不変な円があったとして、その円の方程式を、

 zz+α+b=0

とする。これに反転 w=1/ を行うと、 bww+α+1=0

 この2つの円が一致するためには、 b=1 でなければならない。

 このとき、円の方程式は、 |z+α|=√(|α|2−1) となるので、

 求める円の中心は、−α となる。

 C(−α)とし、この円と単位円の交点の一つをAとすると、

 OC=|α|、OA=1、CA=√(|α|2−1) より、 OA2+CA2=OC2

よって、三平方の定理より、 CA⊥OA となる。

 OAは円Cに点Aで接し、CAは円Oに点Aで接する。

すなわち、求める円は確かに存在し、それは、単位円Oに直交する円である。  (終)


 次に、反転の大切な性質として、

 曲線のなす角を変えない

というものがある。

 今まで話題にしてきた、平行移動、回転、相似拡大においても、もちろん、曲線のなす角
を変えない。

 曲線のなす角(交角)とは、2つの曲線L、Mの交点Pにおいて引いた接線のなす角を言う。

 2つの曲線L、Mの反転をL’、M’とする。LとMの交点Pが反転により、L’とM’の
交点P’に移されるとき、LとMのなす角とL’とM’のなす角は、異符号で絶対値は等
しい。

(証明) 以下では、単位円に関する反転で考える。

 P(z)、P’(w)とし、L、M上にPと異なる点 z1、z2 をとる。点 z1、z2 の反転を w1、w2
すると、

  w=1/ 、w1=1/1 、w2=1/2

なので、 w1−w=1/1−1/ 、w2−w=1/2−1/ から、

 (w2−w)/(w1−w)=(2)/(1)・(1/2

よって、 arg(w2−w)/(w1−w)=−arg(z2−z)/(z1−z)−arg(z1/z2

ここで、z1 → z 、z2 → z とすると、arg(z1/z2) → 0 なので、

 (P’におけるなす角)=−(Pにおけるなす角)

であることが分かる。  (証終)


問題  ガウス平面上の原点を通らない2直線

 (−i−1)z+(i−1)+1=0 ・・・ @

 (−i−)z+(i−+1=0 ・・・ A

が単位円に関する反転で移される図形@’、A’を求めよ。また、@、Aのなす角と@’、A’
のなす角を求めよ。



(解) @は、実軸上の点1/2で交わり、i−1に垂直な直線である。

 Aは、実軸上の点1/(2)で交わり、i−に垂直な直線である。

よって、@とAの交点は、(−1)(1−i)/4 で、なす角は、π/6 である。

 単位円に関する反転 w=1/ で、@は円に移される。その方程式は、

 w+(−i−1)w+(i−1)1=0 ・・・ @’

同様に、Aは、 w+(−i−)w+(i−=0 ・・・ A’ に移される。

 @’は、|w+(i−1)|=2 の円で、中心は 1−i 、半径2 である。

 A’は、|w+(i−)|=2 の円で、中心は −i 、半径2 である。

 @’、A’の原点以外の交点 (+1)(1−i) で各円に接線@’、A’を引き、そのなす

角を調べると、丁度 π/6 となる。向きも考えれば、−π/6 である。  (終)


(コメント) なす角が異符号になることは、上図から推察される。


1次分数変換

 z についての1次の有理式 F(z)=(az+b)/(cz+d) (ただし、ad−bc≠0) で表され

る変換は1次分数変換(メビウス変換)と言われる。

 ad−bc≠0 という条件から、逆変換F-1(z)が存在し、 F-1(z)=(−dz+b)/(cz−a)

である。

 ガウス平面に無限遠点を追加したリーマン面 ∪{∞} において、1次分数変換は、1対1
対応となる。

 w=(az+b)/(cz+d)=(a+b/z)/(c+d/z) において、

 z → −d/c とすると、w → ∞ で、z → ∞ とすると、w → a/c が対応する。

 1次分数変換全体は、群をなす。ただし、可換群ではない。

問題  複素数 z=x+i・y の関数 F(z)=(1−z)/(1+z) の実部 Re F(z) と虚部 Im F(z)
    を、x、y の関数で表せ。

(解) F(z)=(1−x−i・y)/(1+x+i・y)=(1−x2−y2−2i・y)/((1+x)2+y2) なので、

 Re F(z)=(1−x2−y2)/((1+x)2+y2) 、Im F(z)=−2y/((1+x)2+y2)  (終)

問題  関数 F(z)=(z+2)/(2z−1) について、次の問いに答えよ。

(1) F(0) 、F(i) の値を求めよ。

(2) F(z)=i となる z を求めよ。

(解)(1) F(0)=−2 、F(i)=(i+2)/(2i−1)=−i

(2) F-1(z)=(z+2)/(2z−1) なので、 F-1(i)=(i+2)/(2i−1)=−i  (終)

問題  関数 F(z)=(2z+1)/(3z−2) について、F2(z)を求めよ。

(解) F2(z)=(2(2z+1)/(3z−2)+1)/(3(2z+1)/(3z−2)−2)=z  (終)


問題  ガウス平面において、点−1/4+(1/2)i と点 1/2−(1/4)i を結ぶ線分をLとお
    く。1次分数変換 F(z)=1/z により、Lはどのような図形に変換されるか。

(解) xy 座標系において、線分Lの方程式は、x+y=1/4 (ただし、−1/4≦x≦1/2)と
   表される。

 z=x+i・y とおくと、 1/z=(x−i・y)/(x2+y2) なので、

F(z)=u+i・v とおくと、 u=x/(x2+y2) 、v=−y/(x2+y2) となる。

 ここで、 u2+v2=1/(x2+y2) なので、 x=u/(u2+v2) 、y=−v/(u2+v2

よって、 x+y=1/4 より、 u/(u2+v2)−v/(u2+v2)=1/4 から、

 u2+v2=4u−4v 即ち、(u−2)2+(v+2)2=8 ・・・ 中心(2,−2)、半径2の円

また、 −1/4≦x≦1/2 より、 −1/4≦u/(u2+v2)≦1/2 なので、

 u2+v2≧−4u すなわち、 (u+2)2+v2≧4

かつ、

 u2+v2≧2u すなわち、 (u−1)2+v2≧4

 以上を図示すると、線分Lは下図の実線部分(青色)に移される。

 

(終)


 上記の問題の変換 w=1/z は、 w= 、t=1/ の合成変換と考えられる。

 w= ・・・ 実軸に関する対称移動

 t=1/ ・・・ 単位円に関する反転

 このように考えれば、上図が得られる背景が納得されることと思う。


 1次分数変換では、向きも含めて等しくなるという等角性を持つ。

 w= ・・・ 実軸に関する対称移動 → 逆向き

 t=1/ ・・・ 単位円に関する反転 → 逆向き

なので、変換 w=1/z は、「逆の逆」なので、向きも含めて等しくなることが理解できる。

 実際に、1次分数変換 F(z)=(az+b)/(cz+d) において、

c=0 のとき、d≠0 で、 F(z)=(a/d)z+(b/d)  これは、等形変換で、等角性をもつ

c≠0のとき、 F(z)=(a/c)+{(bc−ad)/c2}{1/(z+d/c)} と分解され、この変換は、

平行移動、単位円に関する反転、実軸対称移動、等形変換の合成変換であることが分かる

ので、その結果として、1次分数変換は、等角性を持つことになる。


問題  ガウス平面において、直交する2直線 z=1、z=i が1次分数変換
    F(z)=(z+1)/(z−2) により移される図形を求め、等角性が保たれていること
    を見よ。

(解) z=x+i・y とおくと、

 (z+1)/(z−2)=((x+1)(x−2)+y2)/((x−2)2+y2)−3i・y/((x−2)2+y2

そこで、 F(z)=u+i・v とおくと、

 u=((x+1)(x−2)+y2)/((x−2)2+y2) 、v=−3y/((x−2)2+y2

 z=1 は、 x=1 、y=t とおけるので、

 u=(−2+t2)/(1+t2) 、v=−3t/(1+t2

これより、t を消去して、 u2+v2+u−2=0 すなわち、円 (u+1/2)2+v2=9/4

同様にして、z=i は、 x=s 、y=1 とおけるので、

 u=((s+1)(s−2)+1)/((s−2)2+1) 、v=−3/((s−2)2+1)

これより、s を消去して、(u−1)2+v2+3v=0 即ち、円 (u−1)2+(v+3/2)2=9/4

 これらを図示すれば、

   

 直交する2直線 z=1、z=i の交点 1+i の像 −(1+3i)/2 において、2円が確かに直
交しているのが見て取れる。  (終)


 1次分数変換によって、ガウス円はガウス円に移されるが、複比も1次分数変換によって
変わらない。

 実際に、1次分数変換 F(z)=(az+b)/(cz+d)によるガウス平面上の4点α、β、γ、δ

の像をそれぞれ α’、β’、γ’、δ’とすると、

 α’=(aα+b)/(cα+d)、β’=(aβ+b)/(cβ+d)、γ’=(aγ+b)/(cγ+d)、

 δ’=(aδ+b)/(cδ+d)

なので、 (α,β,γ,δ)={(α−γ)/(β−γ)}/{(α−δ)/(β−δ)} より、

(α’,β’,γ’,δ’)={(α’−γ’)/(β’−γ’)}/{(α’−δ’)/(β’−δ’)} において、

 (α’−γ’)/(β’−γ’)

=((aα+b)/(cα+d)−(aγ+b)/(cγ+d))/((aβ+b)/(cβ+d)−(aγ+b)/(cγ+d))

=[(ad−bc)(α−γ)/{(cα+d)(cγ+d)}]/[(ad−bc)(β−γ)/{(cβ+d)(cγ+d)}]

={(cβ+d)/(cα+d)}・{(α−γ)/(β−γ)}

 (α’−δ’)/(β’−δ’)

=((aα+b)/(cα+d)−(aδ+b)/(cδ+d))/((aβ+b)/(cβ+d)−(aδ+b)/(cδ+d))

=[(ad−bc)(α−δ)/{(cα+d)(cδ+d)}]/[(ad−bc)(β−δ)/{(cβ+d)(cδ+d)}]

={(cβ+d)/(cα+d)}・{(α−δ)/(β−δ)}

より、 (α’,β’,γ’,δ’)={(α−γ)/(β−γ)}/{(α−δ)/(β−δ)} となるので、

 (α’,β’,γ’,δ’)=(α,β,γ,δ)

が成り立つ。以上から、1次分数変換によって、複比は不変である。


 ここで、ガウス平面をリーマン面に拡張すると、複比の4点のうち1つは∞となる場合がある。

 その場合は、複比は極限値で考えることにする。

 例えば、α=∞ のとき、

 (α,β,γ,δ)
 ={(1−γ/α)/(β−γ)}/{(1−δ/α)/(β−δ)}=(β−δ)/(β−γ)

である。他も同様である。この拡張により、

 リーマン面において、1次分数変換によって、複比は不変である

ことが分かる。


問題  ガウス円上の異なる4点 z1、z2、z3、z4 に対して、複比(z1,z2,z3,z4)は実数
    となることを示せ。

(解) ガウス円の中心をα、半径を r とすると、 z=α+r・e^iθ

 このとき、k=1、2、3、4に対して、 z=α+r・e^iθ とおくと、

(z1,z2,z3,z4)={(z1−z3)/(z2−z3)}/{(z1−z4)/(z2−z4)} において、

(z1,z2,z3,z4

={(e^iθ1−e^iθ3)/(e^iθ2−e^iθ3)}/{(e^iθ1−e^iθ4)/(e^iθ2−e^iθ4

なので、(z1,z2,z3,z4)の共役複素数は、

{(e^(−iθ1)−e^(−iθ3))/(e^(−iθ2)−e^(−iθ3))}
  /{(e^(−iθ1)−e^(−iθ4))/(e^(−iθ2)−e^(−iθ4))

={(1/e^iθ1−1/e^iθ3)/(1/e^iθ2−1/e^iθ3)}
  /{(1/e^iθ1−1/e^iθ4)/(1/e^iθ2−1/e^iθ4

={(e^iθ3−e^iθ1)/(e^iθ3−e^iθ2)}・e^i(θ2−θ1
  /{(e^iθ4−e^iθ1)/(e^iθ4−e^iθ2)・e^i(θ2−θ1

={(e^iθ1−e^iθ3)/(e^iθ2−e^iθ3)}/{(e^iθ1−e^iθ4)/(e^iθ2−e^iθ4

=(z1,z2,z3,z4

 よって、 (z1,z2,z3,z4)は実数 となる。  (終)


問題  1次分数変換F(z)により、円Oが円O’に移るものとする。また、円Oに関して、点
    z2はz1の反転とする。w1=F(z1)、w2=F(z2)とするとき、円O’に関して、点w2
    w1の反転であることを示せ。

(解) 1次分数変換は、平行移動、単位円に関する反転、実軸対称移動、等形変換の合成
   変換であるので、与えられた問題は、単位円に関する反転について示せば十分である。

 円Oの中心を c 、半径を r とすると、題意より、円Oに関する反転は、w=c+r2/(

なので、 z2=c+r2/(1) すなわち、 (z2−c)(1)=r2 が成り立つ。

 ここで、 w1=1/1 、w2=1/2 なので、 (1/2−c)(1/w1)=r2 より、

 (c−r2)w121−c2+1=0 ・・・ (*)

 ところで、円Oを単位円に関して反転 w=1/ させると、円O’の方程式は、

 (1/−c)(1/w−)=r2 すなわち、 (c−r2)ww−c+1=0

となる。(*)より、反転の性質から、円O’に関して、点 w1 を反転させた点が w2 であること

を示す。  (終)


 1次分数変換Fによって、複比が不変であることを用いると、4点 z1、z2、z3、z に対して、
その像を、w1=F(z1)、w2=F(z2)、w3=F(z3)、w=F(z) とすると、

{(z1−z3)/(z2−z3)}/{(z1−z)/(z2−z)}={(w1−w3)/(w2−w3)}/{(w1−w)/(w2−w)}

{(z2−z3)/(z1−z3)}・{(w1−w3)/(w2−w3)}=α とおくと、

 (w1−w)/(w2−w)=α{(z1−z)/(z2−z)}

 これより、w を z を用いて表すと、

 w={(w1−αw2)z+αw21−w12}/{(1−α)z+αz1−z2

という1次分数変換w=F(z)が得られる。


問題  1次分数変換 w=(z+2i)/(2iz−1) によって、単位円の内部、周、外部はどの
    ような図形に移されるか。

(解) |w|2−1=w−1=(z+2i)/(2iz−1)・(−2i)/(−2i−1)−1

 展開して整理すると、 |w|2−1=−3(|z|2−1)/|2iz−1|2

よって、 |z|<1 (単位円の内部) のとき、 |w|>1 (単位円の外部)

 |z|=1 (単位円の周) のとき、 |w|=1 (単位円の周)

 |z|>1 (単位円の外部) のとき、 |w|<1 (単位円の内部)  (終)


問題  1次分数変換 w=20z/(5−(2−i)z) によって、単位円の内部および周はどの
    ような図形に移されるか。

(解) z=5w/((2−i)z+20) を、z≦1 に代入して、

 5w/((2−i)z+20)・5/((2+i)+20)≦1

よって、分母を払って整理すると、w−(2−i)w−(2+i)−20≦0 なので、

 (w−(2+i))(−(2−i))≦25 すなわち、 |w−(2+i)|≦5

 以上から、2+i が中心で、半径5の円の内部および周に移される。  (終)


問題  点 z が直線 iz−i=1 上を動くとき、w=(z+i)/(z−i) はどんな図形上を動くか?

(解) z=i・(w+1)/(w−1) を iz−i=1 に代入して、

 −(w+1)/(w−1)−(+1)/(−1)=1

分母を払って整理すると、 3w−w−−1=0

よって、|w−1/3|=2/3 となるので、中心が1/3で半径2/3の円周上を動く。  (終)


問題  1次分数変換 w=z/(z−i) によって、円 |z+i|=1 はどのような図形に移さ
    れるか。

(解) z=iw/(w−1) を、(z+i)(−i)=1 に代入して、

  (iw/(w−1)+i)(−i/(−1)−i)=1

分母を払って整理すると、 3w−w−=0 なので、 |w−1/3|=1/3

 以上から、1/3 が中心で、半径1/3の円に移される。  (終)


 アポロニウスの円を意識すれば、次のような別解も考えられる。

(別解) z=iw/(w−1) を、|z+i|=1 に代入して、 |iw/(w−1)+i|=1 より、

 |w−1/2|/|w−1|=1/2 なので、w は、

1/2 と 1 を結ぶ線分を、1 : 2 に内分する点と 1 : 2 に外分するを直径の両端とする円

すなわち、2/3と0を結ぶ線分を直径とする円を描く。

 この円の中心は、1/3 で、半径は、1/3 なので、方程式は、|w−1/3|=1/3  (終)


問題  1次分数変換 w=(z+2)/(z−2) について、次の問いに答えよ。

(1) ガウス平面において、点 z が点A(−2)とB(2)を直径の両端とする円周上を動くとき、
  点 w はどんな図形上を動くか。

(2) ガウス平面において、点 z が点A(−2)とB(2)を結ぶ線分を 3 : 1 に内分、外分す
  る点を直径の両端とする円周上を動くとき、点 w はどんな図形上を動くか。

(解)(1) z=2(cosθ+i・sinθ) とおけるので、

 w=(2+2(cosθ+i・sinθ))/(−2+2(cosθ+i・sinθ))

 =(1+cosθ+i・sinθ)/(−1+cosθ+i・sinθ)

 =(2cos2(θ/2)+2i・sin(θ/2)cos(θ/2))/(−2sin2(θ/2)+2i・sin(θ/2)cos(θ/2))

 =(cos(θ/2)/sin(θ/2))(cos(θ/2)+i・sin(θ/2))/(−sin(θ/2)+i・cos(θ/2))

 =−i・(cos(θ/2)/sin(θ/2))

 よって、w は虚軸上を動く。

(2) 題意より、 |(z+2)/(z−2)|=3 なので、 |w|=3

 よって、w は、原点中心で半径3の円周上を動く。  (終)


問題  点 z が、原点Oと点A(1+i)を結ぶ線分上を動くとき、点 w=i・(z−2i)+2i はど
    のような図形上を動くか。

(解) w−2i=i・(z−2i) より、点 w は、点 z を点 2i を中心に、90°回転した点である。

 よって、求める図形は、線分OAを点 2i を中心に90°回転した線分となる。

 z=0 のとき、 w=2+2i で、z=1+i のとき、 w=1+3i

 よって、 (z−2−2i)/(1+3i−2−2i)=(z−2−2i)/(−1+i) は実数なので、

 (z−2−2i)/(−1+i)=(−2+2i)/(−1−i)

よって、 (−1−i)z−(2+2i)(−1−i)=(−1+i)−(2−2i)(−1+i) より、

 (1+i)z+(−1+i)−8i=0

となる。両辺に 1−i を掛けて、 2z+2i・−8−8i=0 より、 z+i・=4+4i

 以上から、線分OAは、2+2i と 1+3i を結ぶ線分 z+i・=4+4i 上を動く。  (終)


問題  1次分数変換 w=λ(z−a)/(z−b) において、次の問いに答えよ。

(1) 点 z が、2点 a、b を通る円周上を動くとき、点 w は、どんな図形を描くか。

(2) 点 z が、|(z−a)/(z−b)|=r ( r は一定) で定義されるアポロニウスの円周上を
   動くとき、点 w は、どんな図形を描くか。

(3) 2点 a、b を通る円群と|(z−a)/(z−b)|=r ( r は一定) で定義されるアポロニウ
   スの円群は、互いに直交することを示せ。

(解)(1) 題意より、 arg{(z−a)/(z−b)}=θ または θ+π (θは一定) なので、

  (z−a)/(z−b)=±k・e^iθ=r・e^iθ ( r は実数)

  よって、 w=λr・e^iθ となり、w は原点を通る直線を描く。

(2) 題意より、 |w|=λr なので、点wは、原点中心、半径が |λ|r の円を描く。

(3) 原点を通る直線と原点中心の円は、互いに直交する。1次分数変換は等角写像なの

  で、2点 a、b を通る円群とアポロニウスの円群は、互いに直交する。  (終)


問題  点 z が点 a で接する円群を描くとき、1次分数変換 w=1/(z−a) によって、点
    w はどのような図形を描くか。

(解) 題意より、点 a で接する円群は、半径を r として、中心を a+rp (ただし、|p|=1)

とおくと、 z−(a+rp )=rt (ただし、|t|=1) とおける。

このとき、 w=1/{r(p+t)} なので、 p+t=1/(rw)

両辺の共役複素数を求めて、 =1/(r) すなわち、 1/p+1/t=1/(r

よって、 1/(rptw)=1/(r) より、 t=/(pw) なので、

 1/(rw)=p+/(pw) すなわち、 rp2w+r=p

両辺を rp で割ると、 pw+(1/p)=1/r すなわち、 pw+=1/r

これは、法線が の直線の方程式で、r の値を変化させれば直線群となる。  (終)


問題  単位円を単位円に移す1次分数変換を求めよ。

(解) 求める1次分数変換を w=(az+b)/(cz+d) (ad−bc≠0) とおく。

 |z|=1 のとき、|w|=1 なので、

 w=(az+b)/(cz+d)・()/()=1

 =1/z なので、 (az+b)/(cz+d)・(z)/(z)=1 より、

 (az+b)(z)=(cz+d)(z)

展開して、

 a2+(|a|2+|b|2)z+b=c2+(|c|2+|d|2)z+

係数比較して、 a=c 、|a|2+|b|2=|c|2+|d|2 、b=

(1) a≠0 のとき、

 =c/a すなわち、 b=d/ より、 |b|2=|c|2|d|2/|a|2 なので、

 |a|2+|c|2|d|2/|a|2=|c|2+|d|2

因数分解して、 (|a|2−|c|2)(|a|2−|d|2)=0

 よって、 |a|=|c| または |a|=|d|

 ・|a|=|c|のとき、 c=a・e^(iθ) とおける。

  このとき、 b=d/=d・e^(−iθ) より、

   ad−bc=ad−d・e^(−iθ)・a・e^(iθ)=ad−ad=0 となり、条件に反する。

 ・|a|=|d|のとき、 d=a・e^(iθ) とおける。

  このとき、 b=a/・e^(iθ) より、

 w=(az+a/・e^(iθ))/(cz+a・e^(iθ))=(z+/・e^(iθ))/((c/a)z+e^(iθ))

すなわち、 w={(z+/・e^(iθ))/((c/a)・e^(−iθ)z+1)}e^(−iθ)

 ここで、 −/・e^(iθ)=p 、−e^(−iθ)=q とおくと、 w=q(z−p)/(z−1)

 ただし、 −1+|p|2≠0 より、 |p|≠1 である。

(2) a=0 のとき、 c=0 で、また、 bc≠0 から、c≠0 なので、 d=0

  よって、 |b|=|c| なので、 b=c・t (ただし、|t|=1) とおける。

 このとき、 w=c・e^(iθ)/(cz)=t/z (|t|=1)  (終)


 上記の問題について、複比を用いた別解も考えられる。

(別解) 単位円周上の4点 1、−1、i 、w に対応する単位円周上の4点を α、β、γ、z

 とする。このとき、複比は等しいので、 (α,β,γ,z)=(1,−1,i ,w)

すなわち、(α−γ)/(β−γ):(α−z)/(β−z)=(1−i )/(−1−i ):(1−w)/(−1−w)

よって、 (1−w)/(−1−w)=i・{(α−z)(β−γ)}/{(β−z)(α−γ)} より、

 1−w=i・{(α−z)(β−γ)}/{(β−z)(α−γ)}(−1−w)

展開して整理すると、

 w(−1+i・{(α−z)(β−γ)}/{(β−z)(α−γ)})

=−(1+i・{(α−z)(β−γ)}/{(β−z)(α−γ)})

すなわち、 w(−(β−z)(α−γ)+i・{(α−z)(β−γ)})

=−(β−z)(α−γ)−i・{(α−z)(β−γ)} より、

 w({(α−γ)−i・(β−γ)}z−β(α−γ)+i・α(β−γ))

={(α−γ)+i・(β−γ)}z−β(α−γ)−i・α(β−γ)

ここで、 a=(α−γ)+i・(β−γ)

 b=−β(α−γ)−i・α(β−γ)

 c=(α−γ)−i・(β−γ)

 d=−β(α−γ)+i・α(β−γ)

とおくと、 w=(az+b)/(cz+d) となる。

 ここで、 

=()−i・(

 =(1/α−1/γ)−i・(1/β−1/γ)

 ={β(γ−α)−i・α(γ−β)}/αβγ=d/αβγ より、 d=αβγ

同様にして、

=()+i・(

 =(1/α−1/γ)+i・(1/β−1/γ)

 ={β(γ−α)+i・α(γ−β)}/αβγ=b/αβγ より、 b=αβγ

よって、 w=(az+αβγ)/(cz+αβγ)

 ここで、a=0 のとき、 w=(αβγ/c)・(1/z) と書ける。

 t=αβγ/c とおくと、|t|=1 で、 w=t/z である。

 a≠0 のとき、

 w=(z+αβγ/a)/((c/a)z+αβγ/a

 =a/(αβγ)・(z+αβγ/a)/((c/(αβγ))z+1)

ここで、αβγ/a=p 、a/(αβγ)=q とおくと、c/(αβγ)= なので、

 w=q・(z+p)/(z+1)

と書ける。ただし、|p|≠1 で、|q|=1 である。  (終)


問題  ガウス平面上において、単位円を単位円に移し、原点Oを点2に移す1次分数変換
    を求めよ。

(解) 題意より、求める1次分数変換は、

 w=q(z−p)/(z−1) (|p|≠1、|q|=1) とおける。

 z=0 のとき、w=2 なので、 pq=2 すなわち、 p=2/q

 よって、 w=q(z−2/q)/((2/)z−1)=(qz−2)/(2qz−1) (|q|=1)  (終)


問題  ガウス平面上において、3点 1、ω、ω2 をそれぞれ 0、1、−1に移す1次分数変
    換を求めよ。ただし、ω=(−1+i・)/2 とする。

(解) 複比は等しいので、 (1 ,ω ,ω2 ,z)=(0,1,−1,w)

すなわち、 (1−ω2)/(ω−ω2) : (1−z)/(ω−z)=(1/2) : (−w)/(1−w) より、

 {(1+ω)/ω)}{w/(w−1)}=(1−z)/(2(ω−z))

 w/(w−1)=(z−1)/(2ω(ω−z))

 wについて解くと、 w={1/(2ω+1)}{(z−1)/(z+1)}  (終)


(コメント) 1/(2ω+1)=(−ω2−ω)/(−ω2+ω)=(ω+1)/(ω−1) なので、

  w={(ω+1)/(ω−1)}{(z−1)/(z+1)}

とした方が美しいかな?


問題  リーマン面において、相異なる3点α、β、γをそれぞれ0、1、∞に移す1次分数
    変換を求めよ。

(解) 複比は等しいので、 (w,0,1,∞)=(z,α,β,γ) より、

 (w−1)/(−1):(w−∞)/(−∞)=(z−β)/(α−β):(z−γ)/(α−γ)

 すなわち、(w−1)/(−1):1=(z−β)/(α−β):(z−γ)/(α−γ) より、

 {(z−γ)/(α−γ)}(1−w)=(z−β)/(α−β)

よって、 1−w={(z−β)/(α−β)}{(α−γ)/(z−γ)} より、

 w=1−{(z−β)/(α−β)}{(α−γ)/(z−γ)}

  ={(α−β)(z−γ)−(z−β)(α−γ)}/{(α−β)(z−γ)}

  ={(γ−β)/(α−β)}{(z−α)/(z−γ)}


(コメント) 複比の設定として、 (w,1,0,∞)=(z,β,α,γ) とした方が計算は楽で
      ある。

 実際に、(w−0)/(1−0):(w−∞)/(1−∞)=(z−α)/(β−α):(z−γ)/(β−γ)

すなわち、 w : 1=(z−α)/(β−α):(z−γ)/(β−γ) より、

w={(z−α)/(β−α)}{(β−γ)/(z−γ)}={(β−γ)/(β−α)}{(z−α)/(z−γ)}

が直ちに得られる。


問題  実軸を単位円に移す1次分数変換を求めよ。

(解) 求める1次分数変換を w=(az+b)/(cz+d) (ad−bc≠0) とおく。

また、実軸上の4点 a、b、c、z が単位円周上の4点 1、−1、i、w に移されるものとする。

このとき、複比は等しいので、 (a ,b ,c ,z)=(1,−1,i ,w)

よって、

 (a−c)/(b−c) : (a−z)/(b−z) =(1−i)/(−1−i) : (1−w)/(−1−w) より、

 {(1−w)/(−1−w)}{(a−c)/(b−c)}={(1−i)/(−1−i)}{(a−z)/(b−z)}

よって、 (w−1)/(w+1)=i・{(b−c)/(a−c)}{(a−z)/(b−z)}

 w について解くと、

w={(−(a−c)−i・(b−c))z+b(a−c)+i・a(b−c)}
  /{(−(a−c)+i・(b−c))z+b(a−c)−i・a(b−c)}

なので、

 a=−(a−c)−i・(b−c)

 b=b(a−c)+i・a(b−c)

 c=−(a−c)+i・(b−c)

 d=b(a−c)−i・a(b−c)

 このとき、 c= 、d= が成り立ち、 w=(az+b)/(z+

 ここで、 a−b≠0 から、a≠0 、≠0 で、

 w=(a/)・(z+b/a)/(z+/

 そこで、 −b/a=p 、a/=q とおくと、 p≠ 、|q|=1 で、

 w=q・(z−p)/(z−

と表される。  (終)


問題  実軸を実軸に移す1次分数変換を求めよ。

(解) 実軸上の4点 a、b、c、z が実軸上の4点 0、1、∞、w に移されるものとする。

このとき、複比は等しいので、 ( z ,a ,b ,c )=( w ,0 ,1 ,∞ ) すなわち、

 (z−b)/(a−b) : (z−c)/(a−c)=(w−1)/(0−1) : (w−∞)/(0−∞)

より、 −(w−1)(z−c)/(a−c)=(z−b)/(a−b)

 よって、 w=1−{(z−b)/(z−c)}{(a−c)/(a−b)} より、

 w={(c−b)z+a(b−c)}/{(a−b)z−c(a−b)={(c−b)/(a−b)}・(z−a)/(z−c)

 係数はすべて実数なので、実数は実数に移される。  (終)


問題  1次分数変換が恒等変換でなければ、不動点を高々2個持つことを示せ。

(解) z を不動点として、 (az+b)/(cz+d)=z (ad−bc≠0) とおく。

分母を払って整理すると、 cz2+(d−a)z−b=0

 c≠0 のとき、解は2個あるので、不動点は2個持つ。

 c=0 のとき、 (d−a)z=b

  a≠d のとき、 z=b/(d−a) は、不動点

   また、d≠0 で、 w=(a/d)z+b/d より、 z=∞ のとき、w=∞

   よって、a≠d のとき、不動点は、b/(d−a) と∞の2個持つ。

 a=d、b≠0 のとき、 不動点は、∞の1個持つ。

 a=d、b=0 のとき、1次分数変換は恒等変換となるので、不適。  (終)


問題  1次分数変換 w=(z+i)/(i・z+1) により、実軸に関して対称な点 z、 がそれ
    ぞれ w1、w2 に移されるという。w1、w2 にはどんな関係があるか。

(解) 題意より、 w1=(z+i)/(i・z+1) 、w2=(+i)/(i・+1) なので、

 w12=(z+i)/(i・z+1)・(z−i)/(−i・z+1)=(z2+1)/(1+z2)=1

 よって、 w1=1/2 となり、w1、w2 は、単位円に関する反転の関係がある。  (終)


問題  1次分数変換 w=1/z により、1点A(1)を通る直線群はどんな図形に移されるか。

(解) 1点A(1)を通る直線群は、 z=1+r・t (r は実数で、t は大きさ1の複素数) と表

 されるので、 w=1/(1+r・t) と書ける。このとき、

 w−1=−r・t/(1+r・t)=−r・t・w から、 (w−1)/w=−r・t

 よって、 |w−1|/|w|=r となる。

このことから、w は、1、0を結ぶ線分を r : 1 に内分・外分する点を直径の両端とする円を

表す。すなわち、求める図形は、1、0に関するアポロニウス円群となる。

  


問題  1次分数変換 w=αt/(t−z) (ただし、αはガウス平面上の定点で、|t|=1)
    によって、単位円はどのような図形に移されるか。

(解) z=(α−w)t/w で、|z|=1 より、 |w|=|w−α|

 よって、単位円は、0とαを結ぶ線分の垂直2等分線に移される。  (終)


問題  ガウス平面上の円 |z|=r の内部を円 |w|=R の内部に移す1次分数変換
    を求めよ。

(解) 前述の問題から、単位円を単位円に移す1次分数変換は、

  w=q(z−p)/(z−1) ただし、|q|=1 、|p|≠1

と書ける。また、問題の(解)と同様にして、

|w|2−1=w−1=(1−|p|2)(|z|2−1)/|z−1|2

から、|z|<1 (単位円の内部) を |w|<1 (単位円の内部)に移すためには、

|p|<1 であればよい。

 よって、Z=rz 、W=Rw とおくと、 |Z|=r の内部を円 |W|=R の内部に移す

1次分数変換は、

 W=Rq(Z−rp)/(Z−r) ただし、|q|=1 、|p|<1

と書ける。  (終)


問題  次の1次分数変換によって、領域Dはどんな領域に移るか。

(1) w=1/z  D:|z|<1

(2) w=(z−i)/(z+i)  D:Re(z)>0、Im(z)>0

(3) w=z/(z−1)  D:0<arg(z)<π/4

(4) w=(z−1)/(z−2)  D:0<Re(z)<1

(解)(1) z=1/w より、 1/|w|<1 すなわち、 |w|>1

(2) z=−(w+1)i/(w−1) において、w=u+v・i とおくと、

 z=−{2v+i・(u2+v2−1)}/{(u−1)2+v2

 Re(z)>0 より、 v<0 で、Im(z)>0 より、 u2+v2<1

したがって、求める領域は、 Im(w)<0 で、かつ、 |w|<1

#換言すれば、単位円の下半分が求める領域である。

(3) z=w/(w−1) において、w=u+v・i とおくと、

 z={u2+v2−u−i・v}/{(u−1)2+v2} なので、

 tan(arg(z))=−v/(u2+v2−u)

条件より、 0<tan(arg(z))<1 なので、 0<−v/(u2+v2−u)<1

2+v2−u>0 すなわち、 (u−1/2)2+v2>1/4 のとき、

 0<−v<u2+v2−u

 すなわち、 v<0 かつ u2+v2−u+v>0

 すなわち、 v<0 かつ (u−1/2)2+(v+1/2)2>1/2

2+v2−u<0 すなわち、 (u−1/2)2+v2<1/4 のとき、

 u2+v2−u<−v<0

 すなわち、 v>0 かつ u2+v2−u+v<0

 すなわち、 v>0 かつ (u−1/2)2+(v+1/2)2<1/2

以上を図示すると、求める領域は、下図の水色部分(境界線は含まない)

  

(4) z=(2w−1)/(w−1) において、w=u+v・i とおくと、

 z=(2u−1+2i・v)(u−1−i・v)/{(u−1)2+v2} なので、

 Re(z)=(2u2+2v2−3u+1)/{(u−1)2+v2

 0<Re(z)<1 より、 2u2+2v2−3u+1>0 かつ u2+v2−u<0

前者より、 (u−3/4)2+v2>1/16 すなわち、 |w−3/4|>1/4

後者より、 (u−1/2)2+v2<1/4 すなわち、 |w−1/2|<1/2

以上を図示すると、求める領域は、下図の水色部分(境界線は含まない)

   (終)


問題  1次分数変換 w=(1+i・z)/(i+z) について、次の問いに答えよ。

(1) (w−1)/(w+1)=i・(z−1)/(z+1) が成り立つことを示せ。

(2) 1次分数変換 w=(1+i・z)/(i+z) をn回行ったとき、

 (w−1)/(w+1)=i・(z−1)/(z+1) が成り立つことを示せ。

(解)(1) w−1=(1+i・z)/(i+z)−1=(1−i)(1−z)/(i+z)

 w+1=(1+i・z)/(i+z)+1=(1+i)(1+z)/(i+z) より、

 (w−1)/(w+1)

={(1−i)/(1+i)}・(1−z)/(1+z)

=(−i)・(1−z)/(1+z)=i・(z−1)/(z+1)

が成り立つ。

(2) (1)より、w1=(1+i・z)/(i+z) に対して、(w1−1)/(w1+1)=i・(z−1)/(z+1)

が成り立つ。同様にして、

 w2=(1+i・w1)/(i+w1) に対して、(w2−1)/(w2+1)=i・(w1−1)/(w1+1)

が成り立つ。よって、(w2−1)/(w2+1)=i2・(z−1)/(z+1) が成り立つ。

同様にして、

 w3=(1+i・w2)/(i+w2) に対して、(w3−1)/(w3+1)=i・(w2−1)/(w2+1)

が成り立つ。よって、(w3−1)/(w3+1)=i3・(z−1)/(z+1) が成り立つ。

 したがって、1次分数変換 w=(1+i・z)/(i+z) をn回行ったとき、

 (w−1)/(w+1)=i・(z−1)/(z+1) が成り立つ。  (終)

#厳密には数学的帰納法を用いて示される。


アフィン変換

 w=F(z)=az+b+c (ただし、a、b、c は複素数で、a−b≠0)

によって与えられる変換は、アフィン変換と呼ばれる。

 拡大縮小、回転、平行移動などを表す変換が、アフィン変換である。

 条件 a−b≠0 により、逆変換が存在し、逆変換もアフィン変換である。

 実際に、=F(z)=z+ から、 を消去して、

 w−b=(a−b)z+c−b

 a−b≠0 なので、

 z={/(a−b)}w+{−b/(a−b)}+{−(c−b)/(a−b)}

ここで、

/(a−b)}{a/(a−b)}−{−b/(a−b)}{−/(a−b)}

=1/(a−b)≠0 が成り立つ。

 よって、逆変換もアフィン変換となる。


 アフィン変換の合成もまたアフィン変換になることは自明だろう。

 恒等変換 I(z)=z も明らかにアフィン変換である。

 以上から、アフィン変換全体は、合成を演算として、群をなすことが分かる。


問題  z=x+i・y を、w=x+i・ky (kは0でない実数) に移す変換は、アフィン変換であ
    ることを示せ。

(解) x=(z+)/2 、y=(z−)/(2i)=−i・(z−)/2 より、

 w=(z+)/2+i・k(−i・(z−)/2)={(1+k)/2}z+{(1−k)/2}

 ここで、{(1+k)/2}2−{(1−k)/2}2=k≠0 である。

 よって、z を w に移す変換は、アフィン変換である。  (終)


問題  アフィン変換によって、直線は直線に移されることを示せ。

(解) アフィン変換は、w=az+b+c (ただし、a、b、c は複素数で、a−b≠0)

直線は、z=p+λq (ただし、λは実数で、q≠0) とする。

このとき、直線の像は、w=a(p+λq)+b(+λ)+c すなわち、

 w=ap+b+c+λ(aq+b

となるが、これが直線となるためには、aq+b≠0 であればよい。

 もしも、aq+b=0 と仮定すると、 q=0 でもある。

 この2式から、=−aq/b を代入して、 を消去すると、 (−a/b)q+q=0

すなわち、 (−a+b)q=0 となり、a−b≠0 から、q=0 となる。

しかるに、これは、q≠0 であることに矛盾する。

 よって、aq+b≠0 が言えるので、

 w=ap+b+c+λ(aq+b) は直線となる。


問題  アフィン変換によって、平行線は平行線に移されることを示せ。

(解) アフィン変換は、w=az+b+c (ただし、a、b、c は複素数で、a−b≠0)

平行線は、z=p1+λq 、z=p2+λq (ただし、λは実数で、q≠0) とする。

このとき、直線の像は、それぞれ

 w=a(p1+λq)+b(1+λ)+c=ap1+b1+c+λ(aq+b

 w=a(p2+λq)+b(2+λ)+c=ap2+b2+c+λ(aq+b

となり、互いに平行である。  (終)


問題  アフィン変換によって、平行な線分比は不変であることを示せ。

(解) アフィン変換は、w=az+b+c (ただし、a、b、c は複素数で、a−b≠0)

 平行な2つの線分 P12、P34 について、P1(z1)、P2(z2)、P3(z3)、P4(z4) とおくと、

 (z2−z1)/(z4−z3)=k (kは実数) とおける。

このとき、w=az+b+c (k=1、2、3、4) とおくと、

 (w2−w1)/(w4−w3

={a(z2−z1)+b(21)}/{a(z4−z3)+b(43)}

={ak(z4−z3)+bk(43)}/{a(z4−z3)+b(43)}=k

となるので、平行な線分比は不変である。  (終)


問題  アフィン変換によって、複比は不変であることを示せ。

(解) アフィン変換は、w=az+b+c (ただし、a、b、c は複素数で、a−b≠0)

複比(z1,z2,z3,z4)={(z1−z3)/(z2−z3)}/{(z1−z4)/(z2−z4)}

ここで、w=az+b+c (k=1、2、3、4) とおく。

複比(w1,w2,w3,w4)={(w1−w3)/(w2−w3)}/{(w1−w4)/(w2−w4)}において、

(w1−w3)/(w2−w3)=(z1−z3)/(z2−z3

(w1−w4)/(w2−w4)=(z1−z4)/(z2−z4

なので、(w1,w2,w3,w4)=(z1,z2,z3,z4) が成り立つ。

 すなわち、アフィン変換によって、複比は不変である。  (終)


 次の事実は大切である。すなわち、

 1直線上にない3点を1直線上にない3点に移すアフィン変換は、ただ1つ存在する


問題  正三角形の3つの中線は互いに2:1に内分する点で交わる。これに、アフィン変換
    を施すと、どのような定理が得られるか。

(解) 正三角形ABCの3辺BC、CA、ABの中点をそれぞれP、Q、Rとし、3つの中線AP、

BQ、CRの交点をGとおく。3点A、B、Cは1直線上にないので、それぞれA’、B’、C’に移す

アフィン変換がただ一つ存在する。

 アフィン変換により、線分比は不変であるので、3点P、Q、Rは、それぞれB’C’、C’A’、

A’B’の中点P’、Q’、R’に移される。

 よって、3つの中線A’P’、B’Q’、C’R’は1点G’で交わり、互いに2:1に内分する。

 以上から、

 三角形の3つの中線は互いに2:1に内分する点で交わる

という定理が得られる。  (終)


(追記) 令和5年12月13日付け

 アフィン変換を施すことにより、面積がどのように変わるのかについて調べよう。

 多角形は三角形に分割できるので、まずは、三角形の面積について復習しておこう。

 「面積」の項で見たように、

 Oをガウス平面の原点とし、A(α)、B(β)とする。α=a+i・b 、β=c+i・d とおくと、

 △OAB=(1/2)|ad−bc| である。ところで、

 α=(a+i・b)(c−i・d)=ac+bd+i・(bc−ad)

から、 ad−bc=−Im(α) なので、 △OAB=(1/2)|Im(α)| となる。

Im(α)=(αβ)/(2i) なので、 △OAB=(1/4)|αβ| とも書ける。

 行列式を用いれば、△OABの面積は、

 

 次の問題で、上記を確認しておこう。

問題  O、A(2+i)、B(−1+2i) に対して、△OABの面積を求めよ。

 この問題の(解)は、△OAB=5/2 であったが、実際に、

 

から、正しいことが分かる。


 アフィン変換 w=az+b+c (a−b≠0) により、面積はどのように変わるだろう
か?

 一般に、ガウス平面上の3点A(α)、B(β)、C(γ)を頂点とする△ABCの面積は、

 

で与えられる。

 この△ABCにアフィン変換を施して得られる三角形の頂点を w1、w2、w3 とすれば、

 w1=aα+b+c

 w2=aβ+b+c

 w3=aγ+b+c

なので、 △w123=(a−b)△ABC と書ける。

 特に、a−b=1 のとき、面積は不変で、そのような変換は、等積アフィン変換と呼
ばれる。


問題  長径2a、短径2bの楕円の面積は、πab となることを示せ。

(解) ガウス平面上の点 z=x+i・y に、点w=u+i・v が対応するものとする。

 変換 (x ,y) → (u ,v) を、u=x 、v=ky (kは実数で、k≠0) により定義する。

 このとき、x=(z+)/2 、y=(z−)/(2i) なので、

 w=x+i・ky=(z+)/2+i・k(z−)/(2i)=((1+k)/2)z+((1−k)/2)

ここで、((1+k)/2)2−((1−k)/2)2=k≠0 なので、この変換はアフィン変換である。

 k=b/a とおくと、変換により、原点中心、半径 a の円が、長径2a、短径2bの楕円に移
される。

 よって、楕円の面積は、 k・πa2=πab で求められる。  (終)


問題  原点中心の単位円は、アフィン変換 w=2z+ によってどんな図形に移されるか?

(解) 原点中心の単位円は、z=cosθ+i・sinθ と書ける。このとき、

 w=2z+=3cosθ+i・sinθ となる。w=x+i・y とおくと、 x=3cosθ、y=sinθ

よって、 x2/9+y2=1 となり、楕円に移される。  (終)


#原点中心の単位円の面積は、πで、楕円 x2/9+y2=1 の面積は、3πで、公式通り、
 a−b=22−12=3(倍)になっている。


問題  変換 (x ,y)→(x+y ,x+ky) (kは実数で、k≠1) はアフィン変換であることを
   示せ。また、この変換が等積アフィン変換となるようにkの値を定めよ。

(解) z=x+i・y とおくと、

w=x+y+i・(x+ky)

 =(1+i)(z+)/2+(1+ki)(z−)/(2i)

 =((1+k)/2)z+((1−k+2i)/2)

ここで、 ((1+k)/2)2−((1−k+2i)/2)((1−k−2i)/2)=k−1

条件より、k≠1 なので、この変換はアフィン変換である。

 この変換が等積アフィン変換となるためには、 k−1=1 より、 k=2  (終)



  以下、工事中!